2.reasoning - Aptitude
2.reasoning - Aptitude
,· ,
Reasoning
&Aptitude
c:for GATE 2019
ancl ESE Pre 2019
Reasoning & Aptitude for GATE 2019 & ESE 2019 Prelims
© Copyright, by MADE EASY Publications.
All rights are reserved. No part of this publication may be reproduced, stored in or introduced into
a retrieval system, or transmitted in any form or by any means (electronic, mechanical, photo-
copying, recording or otherwise), without the prior written permission of the above mentioned
publisher of this book.
MADE EASY PUBL!CAT!ONS has taken due care in co!!ecting the data and providing the solutions, before
publishing this book. lnspite of this, if any inaccuracy or printing error occurs then MADE EASY PUBUCATlONS
owes no responsibility. We will be grateful if you could point out any such error. Your suggestions will be appreciated.
PREFACE
I have immense pleasure in placing this edii:ion of "Reasoning & Aptitude" before the
aspirants of Competitive Examinations. The book has been written to meet the growing
requirements of candidates appearing for GATE, ESE Prelims, UPSC-CSAT, SSC, various
Public Sector Examinations, Bank (PO), MBA Entrance Exams, Railways and Campus
Placements of Software Companies etc.
The comprehensive volume would enable the readers to acquire complete and detailed
understanding of "Reasoning & Aptitude". It covers all dimensions of Arithmetic,
Algebra, Geometry, Reasoning and Data Interpretation. My first-hand experience
of coaching the students has been a great source of inspiration and has helped me
immensely in writing this book. Preparation for Civil Services Examination taking
Mathematics as optional subject also helped me sharpen the ideas and arguments
developed here.
I am grateful to my parents and family members, who have been showering their
blessings from the very beginning. I offer my deep sense of gratitude to my Teachers,
Principals of Navodaya Vidyalayas and Professors of NIT Raipur for their blessings
and guidance. I would like to acknowledge the encouragement and useful guidance
provided by my colleagues and seniors serving in IAS, IFS, !PS and IRS etc. My publisher
Mrs. & Mr. B. Singh have been a constant source of support and encouragement. My
special thanks to the entire MADE EASY team for bringing out the book at the earliest
in the hands of readers.
Suggestions and constructive comments from the readers for the improvement of the
book are welcome.
Nem Singh
(Indian Revenue Service)
Contents
Section-A: Arithmetic
i .1 Number System ....................................................................................................... 1-17
2.3 Permutations & Combinations.. ... .. . ....... .. .. .. .. .. . .. .. .. .. . .. ... ... ....... 130-141
3.6 Tables..... ... .. .. .. .. .. .. .. .. .. .. ... .. .. ..... ........................ ... .. .. .. ... .. ... .. .. .. ... .. .. .. .. .. ....... 194-197
l Section
rith ti
Number System
In Quantitative Aptitude (QA), Number System is one of the module which is of critical importance. We can consider
this module as the back bone as well as basic foundation and building block for QA as well as for reasor,ing.
Applications of concepts of numbers can be easily found in puzzles, reasoning based questions, number series and
many more reasoning areas. This is why it is our suggestion to students to understand the concepts discussed in the
module thoroughly alohgwith understanding of applications.
Classifications of Numbers
If b = 0 i~------''---~ilH=l]
Real numbers If a = O pure
imaginary number and
~ - - - ~ -_ _ _ _ I if a" 0 then number
Integers
L
Rational numbers [p/q] form
i will be complex
Irrational number [,s p/q] Natural numiier
-ve Integers Zero (01
[p. q belongs to integers] [q "OJ
I ( +ve inegers;
I
Integers
i SUfiDS
Fractions [If Iq I ct- 1]
Transdental numbers
like [rr. e] i 11 0•1e rs neither prime Prm1e [ except 2 all
[if q = 1 or'.] r~or composite ;)rimes are odcl]
iractions
l
Improper Mixed fractions
P <lql fractions
IP I> lei I
flea! Number
Rational
crs (~ ve integ
l1ole numb
tural Numb
Our main focus in this module of numbers in on real number system. How ever in context, of imaginary number only
following property is important.
2 0 Reasoning & Aptitude fflRDE ERS!:::I
------,++,~=1
"' l0
°] RB-R-e-e--eeH-ee-i:--a-A-swe-F-fS-e-13"HeA--EelJ1-.______lm~p_ro~p_e_r_F_r_a_c_tio_n_____________~
It IpI > IqI
than fraction is improper fraction valve of improper·
Real Number System fraction is< -1 or more than(>) '1
Entire real numbers group of rational and irrational
numbers combined forms the set of real number, Mixed Fraction
which is represented by symbol ➔ R. All real Just a modified form it improper fraction.
numbers can be represented as points on a real 13 3_2
⇒
number line. Eg. 4
'--v-'
4
'-v-'
Improper fraction equivalent mixed fraction
Example 1.
There are two, 2-digit numbers ab and cd, ba is the
I I I I I I I
-5 -4 -3 -2 -1 1 2345678 another two digit number prepared by reversing the
- 0
(Negative Integers EI l Neutral [Set of positive integer I+] digits of ab, if ab x cd = 493, ba x cd = 2059, what
is value 'g' sum of (ab + cd) =?
Natural Numbers (a) 43 (b) 45
All counting numbers or set of positive integers is (c) 47 (d) 46
considered as set of natural numbers. It denoted Ans: (d)
by set [Nor I+]
Value ·g· = ab x cd is odd.
N = {1, 2, 3, 4, .... )
It means ab and cd both are odd.
Whole Number Hence there sum must be even, only one option is
Set of all nonnegative integers are considered as there which even. Hence answer is option d.
whole number; it is denoted by set
Example 2.
W = (0, 1. 2, 3, 4 "" ,)
I have multiple gift vouchers of value,
Note: If terms "numbers" is used without any
Rs. i01, 107,111,121,131, 141,151,171.
qualifier than it means natural number hence forth.
ethe I have to pick exactly 1Ovouchers to make payment
of Rs. 1121. In how many ways I can do that?
Even Numbers & Odd Numbers (a) one (b) two
(c) more than two (d) none of these
1. Even Numbers
e of All numbers divisible by 2 considered as even
Ans. (d)
✓ een numbers. Reasoning is very simple, if I'll add 10, odd numbers
Note: Property evenness is applicable their sum will be always even. Hence there is not
A number is divisible by 5 if the units digit in number LCM and HCF of Fractions
is0or5. 2
Numerator
Eg.: 50, 505, 405 etc. Fractions are written in form of . · Where
Oenom1nator
5. Divisibility by 6 denominator is not equal to zero.
A number is divisible by 6 if the number is even
and sum of digits is divisible by 3. . (H.C.F. of Numerators)
H.C.F of Fraction = (LCM of Denan1ra
. tors) 3
Eg.: 4536 is an even number also sum of digit 4 +
ti+ 3 + 6 = 18 is divisible by 3.
I (' ~A ,-.,f C,,-,nl;r,n - (LCM of Numerators)
Eg: 72, 8448, 3972 etc. L.v.lVI V I ' 'avl/VII - -,-:(H'-C-F-of-0-en-an-im-to_r.:__s)
15!::I mADE EAS!::I * Number Systems I 5
All Fractions have to be in their simplest from: 4. an bn is divisible by a + b only when 'n' is even
'it is natural number.
1 2 3
Example: Find HCF & LCM of
2,3 and 7 Ex. : a 4 - b 4 =(a 2 -b 2)(a2 + b 2 )=(a- b)(a + b)
this (a 2 + b 2 ). Hence a4 - b 4 is always divisible by
HCF = HC.F. of (1,2,3) = _1 (a+ b) but a3 - b3 will not be.
L. C. M (2, 3, 7) 42
Factors of Composite Number
~it is LCM= LCM of (1,2,3) = ~ = 6 Composite numbers are the numbers which can be
HCF of (2, 3, 7) 1 factorised into prime factors, or simply we can say that
composite number are those numbers which are not
Important Algebraic Formulae prime.
1. (a+ b)2 = a2 + 2ab + b
2 For eg.: 8 is a composite number since it can be
digit factorised into
2. )2
( a - b = a - 2ab + b
2 2
8=2x2x2
Similarly 9 is also a composite number. i e
3. (a-b)(a+b) = a 2 -b 2 9=3x3
sum 4. (a+b)2 +(a-b}2 = 2(a 2 +b 2 ) Composite number = p;-, x p~- 2 x p~- 3 .... .P~"n here, P1, P2 ,
even
5. (a+ b)2 - (a - b )2 = 4ab P3 ..... P11 are distinct prime numbers and \ , A2 ,
In in, unit digit will always be i. Sol.: Since i024 is even number unit digit will be 6.
2 7 = i 28 S3 = 125
2 8 = 256 54 = 625
After every four intervals it repeats so cycle of 2 is 2, 4, Unit digit will always be 5.
8, 6. Ex.·
6. Cyclicity of 6.
Ex.1 lJnit digit of 2323 61 = 6
Sol.:
Sol.: Here 2, 4, 8, 6 will repeat after every four interval 6 2 = 36
till 320 next digit will be 2, 4, @l. So unit digit 63 = 216
of 2 323 will be 8. 6 4 = 1296
Unit digit will always be 6.
Ex.2 Find unit digit of 12 12 x 22 22
Sol.: Unit digit of 12 12 will be 6 and 22 22 will be 4. So Ex.1 Find unit digit of 46 9 x 65
c:::o ' • ' '. ,,....c:; •
unit digit of 12 12 x 22 22 wiii be Soi.:
I ' • • I'
Unit OIQI{ Of 4v
'I . .r .. 0
IS 4 ana
I' • •
Unit Olglt OT
r
o·' IS o....... !SU
- -
7. Cyclicity of 7. Remember
71 = 7
2
7 = 49
Cyclicity table
1 :1
73 = 343
2: 2, 4, 6, 8
74 = 2401
3 : 3, 9, 7, 1
75 = 16807
4: 4, 6
76 = 117649
5 :5
77 = 823543
6 :6
78 = 5764801
ed.So 7 : 7, 9, 3, 1
Cycle of 7 is 7, 9, 3, 1
8: 8, 4, 2, 6
Ex.1 Find unit digit of 17 17 x 27 27 9 : 9, 1
Sol.: Unit digit of 17 17 is 7 and unit digit of 27 27 is 3. 0 :0
- every
So unit digit of 17 17 x 27 27 will be 7 x 3 = 21
git will
i.e. 1. □□□□
8. Cyclicity of 8.
81 = 8
82 = 64 Remainder Theorem
83 = 512
. d er of expression
. axbxc
- - [ i.e. ax bx c when
84 = 4096 Rema1n -
n
85 = 32768
So cycle of 8 is 8, 4, 2, 6. divided by n] is equal to the remainder of expression
;:iit is 6. Ex. 1 Find unit digit of 18 18 x 28 28 x 288 288 . an x bn x en [i.e. a11 x bn x c 11 when divided by n]. where
n
Sol.: Unit digit of 18 18 is 4, unit digit of 28 28 is 6, unit
ill be 6. digit of 288 288 is 6. So unit digit of 18 18 x 28 28 x a 11 is remainder when a is divided by n,
288 288 will be 4 x 6 x 6 = 144 i.e. 4. b 11 is remainder when b is divided by n. and
c 11 is remainder when c is divided by n.
) is 4 so 9. Cyclicity of 9.
91 = 9 Ex.1 Find the remainder of 15 x 17 x 19 when divided
I i.e. 8.
92 = 81 by 7.
93 = 729 15x17x19
94 = 6561 Sol.: Remainder of expression - - - - will be
7
Cycle of 9 is 9, 1.
In 9 11 unit digit will be 9 if n is odd and unit digit will 1x3x5
equal to - -
be 1 if n is even. 7
i.e. 1.
Ex. 1 Find unit digit of
On dividing 15 by 7 we get 1 as remainder
1111 + 1212+ 1313+ 14 14+ 15 1s
On dividing 17 by 7 we get 3 as remainder
Sol.: Unit digit of 1 i 11 is 1
On dividing 19 by 7 we get 5 as remainder and
Unit digit of 12 12 is 6
combined remainder will be equal to remainder
Unit digit of 13 13 is 3
Unit digit of 14 14 is 6 of~ i.e. 1.
7
Unit digit of 15 15 is 5
So unit digit of given sum will be
·. is 6 so □□□□
1 + 6 + 3 + 6 + 5 = 21 i.e. 1 .
.e. 4.
8 ® Reasoning & Aptitude fflRDE E~S!:I m
Case-Ill Ex.1 65 2 =?
1atural If a number after adding k is exactly divisible by a , b Sol.: [6 X (6 + 1)]25 = 4225
and c then that number will be. Ex.2 85 2 =?
3taken n x LCM (a, b, c) - k
Sol.: [8 X (8 + 1)]25 ⇒ 7225
Where Ex.1 Find a number which after adding 7 is divisible
Ex.3 95 2 =?
by 10, 11 and 12.
Sol.: [9 X (9 + 1)]25 ⇒ 9025
Jet 3, 4 Sol.: That number will be
nxLCMof[10, 11, 12]-7
Base System
if n = 1 then
The Number system is used to represent any number
660 - 7 = 653 Ans.
using a set of symbols (digits/letters). The base defines
tl1e number of symbols in particular base system. We
□□□□
generally work in Decimal system as there are 10 digits
(0, 1, 2..... 9). Some others systems are;
Squares of Numbers
Binary base system: 2 symbols: 0, 1
1. Squares of numbers are frequently used for calculations Octal base system: 8 symbols: 0, 1,2,3,4,5,6,7
on various types of problems. It is advisable to Hexadecimal system: 16 symbols:
;iet 2, 3
remember square of at least first thirty numbers.
e three 0,1,2,3,4,5,6,7,8,9, A= 10, B = 11, C = 12,
t2 = 1 11 2 = 121
D=13,E=14,F=15
22 =4 122 = 144
Converting any number from any Base system to
32 =9 13 = 169
2
Decimal number system:
42 = 16 142 = i96
abed efg 8 = a x B3 + b x B2 + c x B1 + d x B0
), that 52 = 25 1s2 = 225
62 = 36 162 = 256
+ex s- 1 + f x s-2 + g x s- 3
72 = 49 172 = 289 Example:
e three 8 2 = 64 182 = 324 1234.568 = 1 x8 3 +2x8 2 +3x8 1 +4x8°
92 = 81 192 = 361 +5 X 3- 1 + 6 X 3- 2
;iet3,4
102 = 100 202 = 400 = 512 + 128 + 24 + 4 + 0.625 + 0.093750
From following table we come to know that square of a = 668.718750
,e three
number always ends with 0, 1, 4, 5, 6 & 9 as unit digit.
Converting any number from Decimal to other Base
Square of a number can never have 2, 3, 7 & 8 in its
system:
unit place.
Divide the number by base and get the first remainder
On observing squares of numbers between 21 to 29 we r 1 and Quotient q 1.
~) that get following pattern. Now divided q 1 by base and get remainder r2 and
Quotient q 2 .
2 2
21 =441 29 = 8 41 Repeat the following process till we get the quotient
2 2
22 = 4 84 28 = 8 84
Je 838. 3 2 qn = 0.
23 = 5 29 27 = 7 29
2
24 = 5 76 26
2
= 6 76 Now the decimal number in base b is
2
25 = 6 25 rr/n-1 "•.rl l1 ·
3t k as
viii be Last two digits are common. Example 1:
1. (149)10 = ( )7
D
1420 = 142 X 10 = 71 X 2 X 2 X
7 • Which of these has total 24 positive factors? How many elements does S contain?
(a) 21 x 5
23 (b) 27 x
123 (a) 16 (b) 12
(c) 26 x3 4 (d) 63x55 (c) 11 (d) 13
17. Let T be the set of integers (3, i 1, 19, 27, ...... , 451, 25. In a certain base
2
459, 467) and S be a subset of T such that the sum 137 + 254 = 402 then
of no two elements of S is 470. The maximum What is the sum of 342 + 562 in that base
possible number of elements in S is ? (a) 904 (b) 1014
(a) 32 (b) 28 (c) 1104 (d) 1024
(c) 29 (d) 30
7
18. A box contains 100tickets,numberedfrom 1 to 100. 26. What is the remainder when 77 is divided by 5?
A person picks out three tickets from the box, such (a) 2 (b) 3
that the product of the numbers on two of the tickets (c) i (d) 2
yields the number on the third ticket. Which of the 3
following tickets can never be picked as third ticket?
(a) 10 (b) 12 □□□□
(c) 25 (d) 26
Solutions
19. N is a natural number, then how many values of N
1. (c)
. 6N3+3N2+N+24.
are possible such that N 1s also a
Natural Number?
Method (i) ✓3Joo + 9 + ~ using rationalization
(a) 6 (b) 7 4.
(c) 8 (d) 9 = 3✓80 + 3 xr 9-4✓5]'
20. What is the unit digit of 3953 x 27 23 x 36 12?
9+4Js l9-4✓5
(a) 2 (b) 4
(c) 6 (d) 8 _ 3Jso+ (3x9-3x4~
- 92 -(4'5)2
21. How many number of zeros are there if we multiply
~-----~H-tFTer,rfAWftt:Jm-be-rs-betweeri-0ctn&2efl-\e-.-------)+--~27~~12~F-5=------------
= 3-Jso+ -
(a) 1 (b) 2 81-80
(c) 3 (d) 4
22. A man wrote all the natural numbers starting from 1 = ✓3ff'?Sxs + 27 - 12✓5
in a series. What will be the 50th digit of the number?
~ ~hx 4x ✓S +27-12✓5
(a) i (b) 2
(c) 3 (d) 4 = ✓12✓5 + 27 - i 2✓5
23. N = n(n + i) (n + 2) (n + 3) (n + 4); where n is a = ffi=3✓3
natural number. Which of the following statement/s
is/are true? Alternative Method
1. Unit digit of N is 0.
2. N is perfectly divisible by 24.
3. N is perfect square. ✓(3fao+ 9+~✓5)
4. N is odd.
(a) 3 only (b) 3 and 4 only
3✓
80 = 3Jsi =27
(c) 1 only (d) 1 and 2 only 3
and ---=c <1
9+4✓5
24. How many factors of
✓3Jso + 9 + k = ✓3~
N = 12 12 x 1414 x 15 15 are multiple of
Thus,
K = 12 10 X 14 10 X ,5 10
(8) 2 X 4 X .5 (b) .3 x 5 x 6
(C) 8 X 7 X 4 X 5 (d) 9 x 8 x 6 x 5 = ✓3x9 = 3✓3
® Number Systems 15
2. (d) 6. (b)
x2 Use plugging in
if - = even as (x + 3}3-x3 = 117
y3
x2 = y3 even
x + 3 :2: 5 as (x + 3) 3 :2: 1i7
put X::::: 2
⇒ x 2 ⇒ even
⇒ 53 - 22 = 125 - 8 = 117
and x is integer
ed by 5? ⇒ X == 2, X +3= 5
⇒ x = even
so only xy must be even. So, sum of both numbers = 7
Alternative Method
; 3. (b) (x + 3)3-x3 = 117
1
x3 + (3) 3 + 3(x) (3) (x + 3) - x3 = 117
11
111 x3 + 27 + 27x + 9x 2 - x 3 = 1i7
⇒ 9x 2 + 27x-90 = 0
50 terms .............. 1 1 1 ⇒ x2 +3x-i0=0
40 ⇒ (x + 5)(x-2) = 0
unit digit (1 + 1...... 50 times)= 0
X = 2, -5
and carry = 5
⇒ so either 2, 5, or -5, -2
nalization tens digit (1 + 1 + .... 49 times)+ carry 5 = 4
Thus sum= 7, OR= -7
4. (d)
7. (d)
1
81Y = - - Put in prime factorization theorem.
2JX
(3)4Y = (3-3j (a) 21 5 X 23 = 35 X 75 X 23
⇒
as a:11 = an (b) 27 X (2 2 X 3) 3 = 2 13 X 33
and a;t-i,0,+1 Total factors = i4 x 4 ;t 24
⇒ m=n (c) 26 x 34 = factors 7 x 5 -:1= 24
so 4y = -3x (d) 63x55 = 32 x7fx 15x iik
4 Factors = 3 x 2 x 2 x 2 = 24
X = --y
3
8. (c)
5. (b) For remainder we have to calculate the unit digit
1 1 1 1 1 1
->-· ->--->-·
30 33 ' 30 31' 30 31'
of l 1
⇒
1 1 1 1 1
-+-+->-+-+-
1 ?11
⇒ Now,Rem =(-i) 11
30 30 30 31 32 33 4
3 1i 1
⇒ ->-+-+- =-i ⇒ -1+4=3
30 31 32 33
i 1 1 1 Thus, 3 4K + 3 gives unit
⇒ ->-+-+- Digit ⇒ 3x3x3=7
10 31 32 33
1 1 ?11
⇒ ⇒ n = 10
n 10 so, Rem~= (....... 7)
and 5 5
1 1 1 i 1 1 Thus remainder is 2. As for checking divisibilty by
OR ->-·-> ·--
31 33 ' 32 33 ' 33 - 33 5 is checked by dividing last digit of number.
1 1 1 3 N = i 23 X 132 X 14
⇒ -+-+->- = (2 2 X 3) 3 X 132 X (7 X 2)
31 32 33 33
= 27 x3 3 x7 1 x13 2
i 1 1 1 1
⇒ -+-+->-=- Number of factors
3 i 32 33 1 i n + 1
(7+ 1)(3+ 1)(1 + 1)(2+ 1)= 192
n == 10
16 @ Reasoning & Aptitude mRDE ERS!d m
9. (b) i 3. (c)
1421x1423x1425j
Rem [~] = 4, Rem [~] =5 Rem [
12
⇒ Rem [5x7x9] = Rem [35x9]
Rem= [.i+y3J = [43+53]
12 17
6 6
Rem [llx 9] = Rem [99 ] = 3
[6 + 125] 12 12
6
14. (b)
⇒
4 5 n=85 K + 39
Rem[ ; ] = Rem(i) =3 n 85K +39 34+5
Now -5 K+ - - -
17 17
10. (a)
5
N = (1 ! + 2! + 31 + 4! +.... 1000!) =5K+2+
Now we have to check only
17
⇒ Remainder is 5.
1! + 2! + 3! + 41 as after that will factorial has unit
digit as O 15. (d)
5! = 120 Number are 105, 11 L .... 195
6! = 720 and so on
Thus unit digit of (1 ! + 2! + 3! + 4! + O) for all other ⇒ Total
195 105
- +1 = 16
6
⇒ (1 + 2 + 6 + 4 + 0) 40
Unit digits of all factorial as number is odd and not multiple of 7. 20
⇒ (3)40 = (3)4k Thus in total 16 number there are 3 numbers
⇒ 3 x 3 x 3 x 3 ⇒ 1 unit digit. Hence when 21 X 5 = 125
(3) 40 + 1O, Remainder will be 1. 21 X 7 = 147
21 X 9 = 189
1i. (c) which are multiple of 7.
~ - - - - - · .8slherashouldbaoo1¥-5.wr:ir-Gi::i..&r=i01o1,k;l-G&Gemme0--------=T~hu-s--17-c6c---3~=-1
C":3:-n-u-m-cb-e·-rs---------
between any two number 21
so number may be (5x 1. 5x2 , 5x3 . ... 5xn) 16. (d)
and HFC (xi, xj) = 1 Suppose division 'd' and remainder is 'r'.
Thus, number are ⇒ 3404 1 = q 1 d + r
(5 X 1, 5 X 2, 5 X 3, 5 X 5..... ) and 32506 = q2 d + r
because (xi, xi) = ·1 ⇒ i5365 = (q 1 - q 2)d
Then only prime number will work. ⇒ d should be factor i 535
so 5 x 19 = 95 is biggest number. So, only 307 is factor of i 535 in given option.
SO 5 X 1 , 5 X 2, 5 X 3, 5 X 5, 5 X 7, 5 X 11 ,5 X 13, 5 22
17. (c)
x 17, 5 x '19, total 9 number. /3, 11, ...... 459, 467}
12. (d) This is an Arithmetic Progression
Plugging in 467 = 3 + (n - 1)8
(a) y = 10, X = 2 Total n = 59
:;.;10--2=8 X Now, (3, 407), ( 11, 459), (19,451 ) ....... .
x+ y y 59
(b) - - = 1+- Total - = 29pair
x X 2
y = 9, X = 3 All will make sum 470.
9 So we can take only 1 elements from each pair..
1+ =1+3=4 X
3 Thus total elements in T can be
(c) (x+y)=10+2=i2 X /one of (3,467), one of (11, 459) ... )
( d) None of above is true. Total 29.
e Number Systems I 17
Percentage
SL
The term 'percent' indicates the value out of hundred. It has to be understood in this manner, that a
This concept of percentage is developed to mo.kc tho
comparison of ratio easier by taking the denominator quantity is 200. Its 60% value will be
60
~ o6
100' 5' .
value as 100. times of 200. Here all these values are the Ex
The concept of percentage is very useful in reasoning multiplication factor for the quantity 200.
& aptitude and specially for data interpretation section, Multiplication factors are very important when we
where every logic that has to be used, is based on talk about the percenrage increment or decrement
percentage. of a quantity. Let us understand with some So
examples.
Calculation of Percentage
As we know that percent value is the value out of Example 1.
hundred. The percent value is calculated as A quantity 200 is increased by 75% then what will
be the new quantity?
Value x100
Total Value Solution.
The basic thing that has to be kept in mind is that One thing we can do is to find out 75% of 200 i.e.
the value in the base must be taken care of. 75
-----=-------------------------+10H----------------------
-x200 = 150
Percentage & Fraction
The value percentage can be represented in three and then the new quantity will be 200 + 150 = 350
different form each form is important for making Out we can move in other way as the quantity iflilic1lly
calculation similar. Here are some percentage values was 100% i.e. i as a multiplying factor. Now it is
given to understand different form. increased by 75% i.e. 3/4 or 0.75 as a multiplying
factor.
% form Fraction form Decimal form So the new quantity becomes
100% 1 1
50% -
1
0.5
( 1+ ¾) X 200 = 350
2
- or (i + 0.75) x 200 = 350
1 -
33.33% - 0.33
3
i These(,+¾) and (i + 0.75) are the multiplying
25% - 0.25
4 ---- factors. Let us see one more example to understand
7 it clearly.
20% -
<'
0.20
::)
- 1 Example 2.
16.66% - 0.16
6 A quantity 300 when increased by some percenrage
becomes 360. Find out by what percentage it has
Multiplication factor increased? Cc
Multiplication factor is the value by which a particular
Solution.
quantity has to be multiplied to show the final
Now we know that
changed value. It is nothing but fractional or decimal
~nn
vvv
V ~,1,
.r-. ~v,u
,jtinh1inrt FC10t,.._,
l..'t--"'.1" '~ UVLVI
n\Pill~C\/ -= 3c:o
U
form of percentage.
® Percentage I 19
360 6 Example 1.
;,~ MF= 300 = 1.2or 5 The salary of Ram was increased by 40%. But Ram's
<This shows that the multiplying factor 1 is increased performance kept on declining. Because of which
his employer decreased his salary to the salary
by 0.2 or 2 .Which is equal to 20%. before the increment. Find out what percentage
5 deduction was provided by his employer?
so net increment percentage is 20%.
Solution.
successive Percentage Change Let us assume in starting salary of Ram was x and
Successive percentage change is one percentage after the increment it became y.
change over and above another percentage change. Now his salary was again decreased frorri
Let us take one example to understand it. y to x and we need to find out this percentage
change, we can say it as
Example 1.
A quantity 300 is increased by 20%. Then it was
decreased by 10%. Find out the new quantity is
y= (1+-±Q_)x
100
how much percentage more/less than 300?
⇒ y= (1+~)x
Solution.
300 when increased by 20% becomes 300(1.2) or 7
⇒ y= 5x
300( 1+ i). Now this new quantity is decreased 5
So x =
7y
by 10%. So the final quantity will be
200 i.e.
[300(1.2)](1-0.1) or x = (1-f)y
So we can say the % decrement in the salary of
2
Ram is the percenrage equivalent of which is
0 = 350 = 300(1.08)
7
r initially
~x100% i.e. 28.56%.
low it is 7
ti plying
Example 2.
4 Rama uses rice as his daily meal. But because of
So we can understand the net result is 0.08 or
50 inflation the price on rice were increased by 10%.
increment in MF. Which is equal to net 8% increment Find out by what percentage he has to decrease
in the quantity 300. his consumption to keep the expenditure same as
So it will be very clearly understood from the previous?
example that in successive percentage Solution.
:ti plying changes, the multiplying factors are multiplied Let us assume initial price was Pand the expenditure
directly. So the point that has to be understood ·1s was 'x' then,
erstand
"In case of successive percentage changes the net Price x consumption = Expenditure
multiplying factor is the product of all the
X
corresponding multiplying factors. ⇒ initial consumption =P
:enrage Let us take another examples as:
ie it has
Comparison Leading to the Base Change Now price ii
. became ( i + -10) P == -P
100 10
Sometimes whenever the percentage comparison
occurs between two different values. The base 10(x)
11 p
X
Then final consumption= ==
change occurs. Multiplying factors are again very 11
--P
useful here. Let us understand with some examples. 10
20 ® Reasoning & Aptitude fflRDE ERS~ m
Example 5.
10
So final consumption = (initial consumption) A shop provides flat 50% discount on one shirt.
11
While another shop provides the successive
or we can say the net percent decrement in
discount of 30% and 30%. If the difference of the
10
. w1·11 b e ( 1--) .1.e. - part.
consumption
1 bills is Rs. 43. Find out the net cost of shirt.
i.
11 11 Solution.
Let the price of shirt be x
1 For the first shop the bill is
The percentage equivalent will be
11 x100 = 9.09%
= (1 - 0.5)x = 0.5x
Alternately: The net percent change For the second shop the bill is
= 0.7 x 0.7x = 0.49x
100x-ioo 0
:::: - - - 1/n = 9.091/o The difference in the bills
100 + 10 2.
= 0.01x = 43
Example 3. ⇒ x = Rs. 4300
Solution. Solution.
The net multiplying factor Net percentage increment in radius= 10%
New radius = 1. 1 r
= (1+~)(1-~)
100 100
4
So the new volume= -rc(1. 1 rr
3
'
3
= 1.2 X 0.8 = 0.96 = ( 1. 1) ( original volume)
= (1 0.04) = 1.331 (original volume)
□□□□
® Percentage I 21
Ans. (a)
:;hirt. Solved Examples Height increased =25%
;sive here x = 25 to reduce to normal % correction
,f the required is
Which of the following is the largest number?
25
(a) 20% of .200 (b) 7% of 500 = - - - X 100 = 20%
100 + 25
(c) 1300%of3 (d) 600%of7
Ans. (d) 5. A number is mistakenly divided by 5 instead of being
20% of 200 = 40 multiplied by 5. Find the percentage change in the
7% of 500 = 35 result due to this mistake.
1300% of 3 = 39 (a) 96% (b) 95%
600% of 7 = 42 (c) 2400% (d) None of these
Ans. (a)
Mr. Rajesh is worried about the balance of his Let number is 100
monthly budget. The price of petrol has increased
It is divided by 5 we get 20
by 40%. By what percent should he reduce the
Now actual result should be 5 x 100 = 500
consumption of petrol so that he is able to balance
So % change is result
nd his budget?
(a) 33.33 (b) 28.56 = 500 - 20 X 100 = 96%
(c) 25 (d) None of these 500
Ans. (b)
6. In a mixture of 80 litres of milk and water, 25% of
We know that% reduction required is
the mixture is milk. How much water should be
X added to the mixture so that milk becomes 20% of
= - - x 100 here x=40
100+x the mixture?
(a) 20 litres (b) 151itres
40
= - x 100 = 28.56% (c) 25 litres (d) None of these
140
Ans. (a)
3. In an election between 2 candidates, Ravikant gets Total mixture 80 litre
65% of the total valid votes. If the total votes were
is while 25
6000, what is the number of valid votes that the
·1 ·
M1k1s25% .
1.e. --x80= 20 1·1tre
of male 100
other candidate Shailendra gets if 25% of the total to make it 20% amount of water required to add
married
votes were declared invalid?
(a) 1625 (b) 1575 = 60 _+ X X 100 = 80%
(c) 1675 (d) 1525 80+x
Ans. (b) 6000 +1OOx = 6400 + 80x
Total votes 6000 20x = 400, X = 20
Invalid votes = 25% of 6000 = 1500
7. A landowner increased the length and the breadth
Total valid votes = 4500
of a rectangular plot by 10% and 20% respectively.
Ravikant gets 65%
1 So other candidate gets 35%
Find the percentage change in the cost of the plot
assuming land prices are uniform throughout his plot.
35% of 4500 = 1575
(a) 33% (b) 35%
4. In a medical certificate, by mistake a candidate gave (c) 22.22% (d) None of these
his height as 25% more than normal. In the interview Ans. (d)
panel, he clarified that his height was 5 feet 5 here x = i O or y = 20
inches. Find the percentage correction made by the
candidate from his stated height to his actual height. % change in area= (x + y + 100
xy )
(a) 20 (b) 28.56
1 20
(c) 25 (d) Noneofthese = 10 + 20 + OX = 32
100
22 ® Reasoning & Aptitude fflRDE ERS!:I IT
8. The length, breadth and height of a room in the shape the population drops by 5%. Find the population at
of a cuboid are increased by l0%, 20% and 50% the end of the third year if in the third the population
respectively. Find the percentage change in the increases by 20%.
volume of the cuboid. (a) 12,340 (b) 12,540
(a) 77% (b) 75% (c) i ,27,540 (d) 12,440
(c) 88% (d) 98% Ans. (b)
Ans. (d) p =10000, X = +i0%, y =-5%, Z = 20%
Let!, b, h be length, breadth and height of the cuboid Population at the end of third year
Volume v = /.b.h
Now, l, b, h are increased by 10%, 20% 50% = P(1+-=-)(1+l)(1+~)
100 100 100
respectively
20 5 10 )( 1 -5-)( 1 +
= 10000 1+-- 20)
-
v = t[1+__!Q_]xb[1+
100 100
]+h[i+ 0]
100
( 100 100 100
= 1.98 /.b.h = 12540
% change = 98%
12. In an examination, Mohit obtained 20% more marks
9. The price of sugar is reduced by 25% but inspite of than Sushant but i 0% less than Rajesh. If the marks
the decrease, Aayush ends up increasing his obtained by Sushant is 1080, find the percentage
expenditure on sugar by 20%. What is the percentage marks obtained by Rajesh if the full marks is 2000.
change in his monthly consumption of sugar? (a) 86.66% (b) 72%
(a) +60% (b) -10% (c \I 7P ~301,
I v.v ,o ( d) None of these
(c) +33.33% (d) 50% Ans. (b)
Ans. (a) Marks obtained by Shushant is 1080
Let price of sugar be x & expenditure E 120
Mohit's marks =- x i 080 = 1296
3 100
Now it is reduced by 25%. So it is - x now
4 90
expenditure of sugar is also increased by 20% i.e. Rajesh Marks, R ⇒ - x R = i 296
--------------------------------~10=0~----------
6 R = 1440
1.2 E or E
5 H~O
So quantity of sugar that can be purchased % of Rajesh Marks= - - x 100 = 72%
2000
6
-E 13. The population of a village is 5500. If the number of
= - 5-= _§_§_ = 160% of~
3/4x 5 X .\ males increases by 11 % and the number offem ales
Hence consumption increased by 60%. increases by 20%, then the population becomes
6330. Find the population of females in the town.
10. 30% of a number when subtracted from 91, gives
(a) 2500 (b) 3000
the number itself. Find the number.
(c) 2000 (d) 3500
(a) 60 (b) 65
Ans.(a)
(c) 70 (d) None of these
x is population of male
Ans. (c) (5500-x) is female population
30
x = 30% of number x
X ~12:! + (5500 - X) X120 :.:. 6030
100
100 100 °
.Now
. 91 --x=x
30
100 On calculating we get x = 3000
So female population = 2500.
130
91=-X X = 70
100 ' 14. Vicky's salary is 75% more than Ashu's. Vicky got a
raise of 40% on his salary while Ashu got a raise of
11. The population of the village of Rampur is 10,000 at
this moment. It increases by 10% in the first year. 25% on his salary. By what percent is Vicky's salary
However, in the second year, due to immigration, more than Ashu's novv?
® Percentage I 23
ion at (a) 96% (b) 51.1% i 7. 40% of 20% + 30% of 25% + 50% of 28% is
lation (c) 90% (d) 52.1% equivalent to
(a) 29.5% (b) 28.5%
Ans. (a)
Lets Ashu's salary = Rs. 100 (c) 30.5% (d) None of these
Vicky's salary = Rs. 175 Ans. (a)
Vicky's salary increased by 40% i.e.
40 20 8
40% Of 20% = - X- =- = 8%
140 100 100 100
= x 175 = Rs. 245
100
30 01/o of 25 01/o = 30 X 25 = 75 = 7 5%
Ashu's salary increased by 25% ie 125 100 100 100 .
Vicky's Salary is 120 more that Ashu's in % term
50 28
120 and, 50% of 28%=-x-
X 100 = 96 % 100 100
125
15. During winters, an athlete can run 'x' metres on one =~=14%
narks 100
bottle of Glucose. But in the summer, he can only
narks :. (40% of 20%+ 30% of 25% + 50% of 28%)
run 0.5x metres on one bottle of Glucose. How many
ntage
bottles of Glucose are required to run 400 metres = 8% + 7.5% + 14% = 29.5%.
2000.
.during summer?
18. A man's wages were decreased by 50%. The
(a) 800/x (b) 890/x reduced wages were increased by 50%. He had a
(c) 96 (d) 454/x loss of
Ans. (a) (a) 35% (b) 25%
During summer to run -5x one bottle of glucose (c) 20% (d) None of these
is required
Ans. (b)
1 Here, x = -50 and y = 50.
then to run 1 km - bottles
0.5x :. The net% change in wages
1
:. to run 400m - - x 400 = soq_ bottle
0.5X X
= (x+y+ 1-Jo)%
15% student failed in both subjects
=(10+10+ 10xi0)%=21%.
So total passed = 85% 100
/ got a
80 + 70 = 85+x, X = 65%
aise of
65% of total = 325
salary
total= 500. □□□□
24 0 Reasoning & Aptitude mRDE ERS~ n
5. Two shopkeepers sell a radio of similar brand and 13. In an examination, 30% and 35% students
type at the same list price of Rs. 1000. The first respectively failed in History and Geography while
allows two successive discount of 20% and 10% 27% students failed in both the subjects. If the
and the second allows the successive discount of
number of students passing the examination is 248,
15% and 15%. Find the difference in discounts 2
find the total number of students who appeared in ·
offered by the two shopkeepers.
the examination.
(a) Rs. 3.50 (b) Rs. 1.bO
(a) 425 (b) 380
(c) Rs. 2.50 (d) None of these
(c) 400 (d) None of these
6. The tax on a commodity is diminished by 10% and
its consumption increases by 10%. Find the effects 14. In an examination, there were 2000 candidates, oul.
on revenue. of which 900 candidates were boys and rest were
(a) 1% increase (b) 2% increase girls. If 32% of the boys 38% of the girls passed;
(c) 3% decrease (d) None of these then the total percentage of failed candidates is
ERS!::i .DE ERS!::i'! ® Percentage I 25
---!Iii:""'·
1
=( 100+15
15 1
x100)%ori3- %.
23
(b) 23- %
i3
to 729000 1. Ans.(b) 28
= 28% of 1000 = -x1000
100 = Rs . 280 .
Here, x = 20
Also, the equivalent discount of two successive
Required answer= ( x x 1oo)· % discounts of 15% and 15%
i00+ X
students
:1.phy while:'
cts. If the,
tion is 248)
15x15) 1/o or 27-%
=( -15-15+-- 30
)peared 2. Ans. (b) 100 4
Here, l = 7, and m = 28.
First number Discount on the list price of radio offered by the
second shopkeeper.
l
= -x
m
100% of second number 3 111
= 27 - % Of 1000 = - X 1000
idates, oul 4 400
j rest were;. 7 = Rs. 277.50
== x 100% of second number
Is passed~ 28 Difference in discounts offered by the two
dates is Or 25% of second number. shopkeepers= Rs. 280- Rs. 277.50 = Rs. 2.50.
26 ® Reasoning & Aptitude mRDE ERS'!:,i m
= A(1+~)(1+l)(h-z)
100 100 100
Then, A(1+~)(1+l)(1+-z ).
100 100 100
3 3
=(30+30+ 0x 0)%=69%.
100 = 729000 (Given)
8. Ans. (a)
A(1-1~00)(1- 1~00)(1- 1~00)
Since side 1 x side 2 = area
= 729000
Error% in area= (x+ y + iO0
xy )%
⇒ A= 729000 x iO0 x 100 x 100
10 90x90x90
= - 10-20-~20) %
( mo~--+--------------=--i10eeeeE}rneH-.-----------
(Here, x = 10 and y = -20) 13. Ans. (c)
= -12%, i.e. 12% deficit. Percentage of students passing the examination
9. Ans. (a) = (100- (30 + 35- 27))%
1
(a) 13 (b) 15
0. 3
duction = 30 x 100% = 23J_%
..··· 100+30 13 (c) 20 (d) 163_
.,"
3
Ans. (c) 5. If x% of a is the same as y% of b, then z% of b is
New price must be increased by
(a) yz% of a (b) xy % of a
•·.
,.
Co:~20 x 100)% 25% a
X z
12. If the numerator of a fraction be increased by i 5% 19. Ram sells his goods 25% cheaper than Shyam anc
and its denominator be dimimished by 8%, the value 25% dearer than Bram. How much percentage it
15 Bram's goods cheaper than Shyam's?
of the fraction
16 . The original fraction is (a) 33.33% (b) 50%
3 3 (c) 66.66% (d) 40%
(a) 5 (b) 4
20. Rajiv wanted to subtract 5 from a number
3 2
(c) - (d) 3 Unfortunately, he added 5 instead of subtracting
7
Find the percentage change in the result.
13. In an examination, 35% candidates failed in one (a) 300%
subject and 42% failed in another subject while i 5% (b) 66.66%
failed in both the subjects. If 2500 candidates (c) bU%
1
appeared at the examiniation, how many passed in (d) Cannot be determined
either subject but not in both?
(a) 325 (b) 1175 21. The salary of Amit is 30% more than that of Varur,
(c) 2125 (d) Noneofthese Find by what percentage is the salary of Varun les
than that of Amit?
14. The boys and girls in a college are in the ratio 3: 2. (a) 26.i2% (b) 23.07%
If 20% of the boys and 25% of the girls are adults, (c) 2i .23% (d) None of these
the percentage of students who are not adults is:
22. Ram spends 20% of his monthly income on his housi
(a) 58% (b) 67.5%
hold expenditure, 15% of the rest of books, 30% c·
(c) 78% (d) 82.5%
the rest on clothes and saves the rest. On countin~
15. The price of sugar is increased by 20%. As a result, he comes to know that he has finally saved Rs. 952(
a family decreases its consumption by 25%. The Find his monthly income.
expenditure of the family on sugar will be decreased (a) 10000 (b) 15000
by: (c) 20000 (d) None of these
-------;at-"J.G.?/c..--------1'.13}-§C>fc~----------------------
23. An ore contains 25% of an alloy that has 90% iror
(c) 14% (d) 15% Other than this, in the remaining 75% of the or,
16. A building worth Rs. 133. i 00 is constructed on land there is no iron. How many kilograms of the ore ar
worth Rs. 72,900. After how many years will the value needed to obtain 60 kg of pure iron?
of both be the same if land appreciates at i 0% p.a. (a) 250 kg (b) 275 kg 3
and building depreciates at 10% p.a.? (c) 300 kg (d) 266.66 kg
(a) 2.5 (b) 2
24. Ram sells his goods 20% cheaper than Bobby an
(c) i.5 (d) 3 20 dearer than Chandilya. How much percentage
17. A reduction of 21 % in the price of wheat enables a Chandilya's goods cheaper/dearer than Bobby's.
(a) 33.33% (b) 50%
person to buy 10.5 kg more for Rs. 100. What is the
(c) 42.85% (d) None of these
reduced price per kg?
(a) Rs. 2 (b) Rs. 2.25 25. Out of the total production of iron from hematite, a
(c) Rs. 2.30 (d) Rs. 2.50 ore of iron, 20% of the ore gets wasted, and out(
tho remaining ore, only 25% is pure iron. If the pur
18. The length of a rectangle is increased by 60%. By iron obtained in a year from a mine of hematite wa
what percent would the width have to be decreased 80,000 kg, then the quantity of hematite mined frcJf:
to maintain the same area? that mine in the year is
(a) 5,00,000 kg (b) 4,00,000 kg
(a) 372% (b) 60% (c) 11,50,000 kg (d) Nono of these
2
n(·
(c) 75 % (d) None 26. Ram spends 30% of t-iis salary on t-1ouse rent, 3v
of the rest he spends on his children's educatic~
® Percentage I 29
Purchases = 20% of x= 5.
X 5. Ans. (c)
X y
x% of a = y% of b ⇒ -a =-b ⇒ b
X 4x i 00 i 00
Balance= x-- = - .
5 5
.Transportation = 5% of 00) a-_ ( -X) a
-_ ( -X xI-
100 y y
4x 5 4x x
-=-X-=-.
5 100 5 25
:. z% of b = ( z% off a j
4x x 19x
Balance= - - - = - .
5 25 25
19x 1520 X 25
- - = 1520 ⇒
25 .
X =- -
19
- = 2000 .
6. Ans. (c)
Ans. (d}
Let total quantity of original milk= i 000 gm.
90% iror;;
80% Of X :::: 800 ⇒ 80
X = 800
Milk after first operation
>f the or(1 100 = 80% of 1000 = 800 gm.
,e ore aw
⇒ X = 800 X -100
80
= i 000 . Milk after second operation
= 80% of 800 = 640 gm.
3. .Ans. (c) Milk after third operation
= 80% of 640 = Si 2 gm.
Let, total population = x. Males = §_ x. :. Strength of final mixture= 51 .2%.
lobby an. 9
centage~\ Married males = 30% of 7. Ans. (a)
5 30 5 X
-x=-X-x=- Let A's salary= x, Then, B's = (2000 - x)
9 100 9 6
5% of A =15% of B, i.e.
Married females = ~. 5 15
x= (2000-x) or x= 1500.
6 100 100
7x
= ( -x-x100rn
%=10%. ) i 5. Ans. (a)
100 7x Let original consumption= 100 units & original pric1 1S
= Rs. 100/unit.
i 1. Ans. (c)
Original expenditure== Rs. (100 x iO0)
9 125 = Rs. 10000.
A:;;: 0 B B= c and C= SOD.
100 ' 100 100 New expenditure::: Rs. (120 x 75) = Rs. 9000.
Decrease in expenditure
iO 4 5
8 =-A.C =-Band D = -C.
9 5 4 1000
= ( -.-x100 ) % = 10%.
10 4 10000
B =- x 360 = 400, C =- x 400 = 320
9 5 16. Ans. (d)
5
and D = - x 320 = 400. 2C
4 o)n ( iO)n
·-,.------------7-29_0 00 -- = 133100x - 1- -100 -
Percentage of D = ( -
ziee
x 100 % = 80%.
)
500
Failed in both =( 15
00 x 2500 ) = 375. or
2100
x:::;:---
1
10.5x79
Failed in 1st subject only =(875- 375) ;;:: 500.
Reduced price
Failed in 2nd subject only=(1050-375) = 675.
( 7q ?1()() \
Passed in 2nd only + Passed in 1st only
= (675 + 500) = 1175.
: :;: Rs.l 1'o~ox 1~i:v79Jkg= Rs. 2/kg
E leRS ® Percentage I 31
25x90 k .
.~:i~/x (j~o:;x) xb= lb = 100 = 22 .5 g iron
P = Rs. 6713.21
·/kg
111!111111
'" ; .-·:'?;·,.,_-.:;_;:
In any organisation, the most important aspect is profit Mark up: This is the increment on the cost pricE
and loss calculation of any transaction conducted. Su before the article is sold to tho customer.
the profit/loss calculation is having a very important From these terminologies it has to be understooc
impact from business point of view. that the relation of selling price and cost price ii
While dealing with the profit/loss calculation, the nothing but a percentage increment or decremen B
situation based on profit or loss respectively. So•
following terminologies have to be understood. profit or loss is P% we can use the multiplying tactc
Terminalogies in Profit & Loss
Cost price: Cost Price (CP) of an article is the
. (1+_£_)
1.8.-100
expenditure incurred on the purchase or the
production of the article. it can be taken as the So SP= cP(1±_£_
\ 100)
I St
'INVESTMENT' on the article.
Now let us understand it with the help of som
Selling price: Selling price of an article is the examples.
revenue generated by selling that particular article.
So it is the 'REVENUE' Example 1.
Ram bought a scooter for Rs. i 0000 and sold it fc
Both CP & SP are respective quantities. They are
---frcrl'--&e-s-eil:fte--£!-lf8R-titi@S-L-e.,-lb.a-11.alu.e_ke_e.p_s. on ___ 20% loss to Rahul. Rahul invested Rs. 500 in
changing with respect to the person. and solaa.flU%profM:Tu""""3orran-:-firn:rotrtth&amet1'----E----
. )
paid by Sohan.
If A is selling a quantity to 8. In that case selling
price for A is the cost price for B. Solution.
Profit/Loss: The difference of SP & CP is the profit The CP for Rahul
value or loss value. If SP> CP, the value is profit. If 20
= iOOOOl( 1- ) Sc
SP < CP, the value is loss. 100
Percent Profit/Loss: The profit or loss as a = Rs. 8000
percentage of the 'INVESTMENT is profit or loss Net investment of Rahul on the scooter
percentage. = 8000 + 500 = Rs. 8500
Profit/Loss Value Rahul sold it for 10% profit
% Profit/loss = x 100
,
1nvestment So amount paid by Sohan
Margin: Margin is generally used in terms of
percentage. It is the profit or loss as percentage of = s5oo(, + -~) = Rs. 9350
100
the 'REVENUE' or selling price.
.. . Profit/Loss value
% Margin= - - - - - - x ,1 v
on Example 2.
Revenue Komal sold two articles at sarne selling price a·
Marked Price: It is the price of the product being at 20% profit while another at 20% loss. Find Ci
displayed on the label. Generally it is taken as MRP. his net profit or loss percent?
Discount: Discount is the rebate given to the Solution.
customer before the selling of the article. Generally Let the SP of one article be x
discount is given on marked price but to have a loss One article is sold at 20% profit so
the discount can be given on the cost price as well.
1.2 x (CP), =x
e Profit and Loss I 33
2x Example 1.
- ---2x
- 0.96 A shopkeeper sells the quantity in the same price
_ 2x(0.04)/0.96 X 100 rate for which he has bought. But he gives 20%
- 2x/0.96 less quantity to the customer. Find his profit percent.
= 4% loss
Solution.
pie 3.
Let the indicated weight be 100 gm & CP & SP be
'~se bought a bullock and a cart in rupees 6000.
1 Rs./gm
.. he sold bullock, at 20% profit and cart at 10%
~rofit and got Rs. 7000. Find out the cost price of Now quantity given to the customer
:::: (1 - 0.2) X 100
ullock.
So investment of shopkeeper
on . = 80 x 1 = Rs. 80
.L~t the CP of bullock be 'b' & of cart be 'c' The amount gained from the customer
. " Then b + c = 6000 = 100 x 1 = Rs. 100
and 1.2 b +Uc= 7000
100-80 .
By solving both the equation we get Profit%= - - - x 100 = 25% profit
80
d sold itfi b = 4000 and c =2000
s. 500 ini So CP of bullock is Rs. 4000. Example 2.
the amoui'E' Ramesh purchased a radioset at Rs. 1500 and sold it at
. xamp 1e 4.
'1'. Rs. 1200. Find loss incurred by him?
A shopkeeper gives a discount of 10% on a article
but his net profit becomes 8% still. Find out by LOSS= CP - SP
what percent he mark up the article. = 1500 - 1200 = 300.
Also in this case we can calculate
Solution.
Let the CP ce x & the mark up be P°lo 300
Loss% = --- x 100 = 20%
1500
Then SP = (1 + __§_)
100
x (As per the profit)
Thus he incurred 20% loss.
4. When S.P. and loss percentages are given True measure 100+ g
=
1
Faulty measure 100 + x
CP==( 00 )xiOO
100-Loss% iOOO iOOOxg
⇒ 950 = 950
5. If the cost price (C.P.) of m articles is equal to selling 1000x100 5
price of n article, then So, 100 + g == , g = 5 01/o.
950 19
m-n] x 100
% gain or loss= [ -n-
□□□□
7. When two different articles are sold at the same Ex.1 Find the single discount which is equivalent:
selling price getting gain of x% on the first and loss successive discounts of 50% and 40%.
of x% on the second, then the overall% loss in the Sol.: Single discount will be equal to
~---------------+-1~v~-)~2____________~Cm+n
transaction is given by
10
% ~
1moon)o=·-------~
⇒ 44%
Example 1.
Now, we will find single discount whid
A dishonest shopkeeper professes to sell his goods
at the cost price bL:1: ·__;ses faulty measure. His 1 kg equivalent to two successive discounts of 4
and 20%.
weight measures 950 gms only. Find his gain percent.
Solution: ⇒ rL44+ 20- ~~3°1%
J IUU
Here, True measure = i 000 gms ⇒ 55.2%
False measure= 950 gms Alternative:
Since the Shopkeeper sells the goods at cost price. x x 0.8x x 7x x 8 = 0.448x
So the final value is 0.448x
X = 0,
Which is reduced by 55.2%
overall gain % is given by
□□□□
EE ® Profit and Loss I 35
Ans. (b)
Solved Examples CP of 12 chocolate = Rs. 9
9
elling a watch for Rs. 495, a shopkeeper incurs CP of i chocolate=-= Rs. 0.75
12
ss of 10%. Find the cost price of the watch for Now SP = Rs. 1 Proift = Rs. 0.25
' shopkeeper.
25
~) Rs. 545 (b) Rs. 550 Profit% = 0· x 100 = 332%
0.75 3
q) Rs. 555 (d) None of these
s. (b) 5. A coal merchant makes a profit of 20% by selling
coal at Rs. 25 per quintal. If he sells the coal at Rs.
22.50 per quintal, what is his profit percent on the
irem% whole investment?
: equivale SP (a) 6% (b) 6.66%
p :: - - - - - , - X 100
· · (100 -Loss%) (c) 7.5% (d) 8%
n-:~) · 495
Ans. (d)
P =-x100 = Rs. 550 Profit % = 20%, SP=25
90
25 125
hopkeeper sold goods for Rs. 2400 and made a CP = x100 = Rs.= 20.83
120 6
fit of 25% in the process. Find his profit percent
Profit if SP = 22.50
Hf:he had sold his goods for Rs. 2040.
= 22.50 - 20.83 = 1.667
(a) 6.25% (b) 7%
(d) 6.5% 1 667
Profit%= · x 100 = 8%
Ans. (a) 20.83
0%. SP = 2400, Profit% = 25 [in fractional term this can be solved very easily]
uction of 10% in the price of sugar enables a (a) Rs. 175 (b) Rs. 200
yvife to buy 6.2 kg more for Rs. 279. Find the (c) Rs. 225 (d) Rs. 160
'ed price per kilogram. Ans. (a)
t
5 (b) Rs. 4.5 Let the cost price = Rs. x
1. 4.05 (d) None of these
Price 5% less than cost price=
9
~x
10
. (b) Selling price when sold for Rs. 7 more
Let original rate= Rs. x per kg
. New rate = 90% of = 110x+ 7
100
x = Rs. ( 1~~ x) = Rs. ~; 110 95 95x 20
Given - x + 7 - - x = - x -
, 100 100 100 100
' 279
Original quantity for Rs. 279 = - 20 7x500
X ⇒ -x=7 :. x = - - =Rs.175
500 20
10 310
New quantity = 279 x -
9x
=- X 16. A dishonest dealer professes to sell at cost price
but uses a 900 gram weight instead of a 1 kilogram
X
weight. Find the percent profit to the dealer.
(a) 10% (b) 11.11%
(c) 12.5% (d) Noneofthese
Ans. (b)
9x5 Let the cost price be Rs. x per kg.
Reduced Price =·-=Rs. 4.5 per kg
10
. 9
Then cost price of 900 gm =
f14. A man sells an article at 5% above its cost price. If 10 x
I he had bought it at 5% less than what he paid for it Hence% profit
( and sold it for Rs. 2 less, he would have gained 9
~ x----x
.,if 10%. Find the cost price of the article. :::: .
9
1O. X I 00 = 1OO
9
% = 11 11 °6
. R
abour i (a) Rs. 500 (b) Rs. 360
~- -x
ir (d) Rs. 425 (d) Rs. 400 10
Ans. (d)
□□□□
,1. Let the cost price of the article = Rs. x
~
i Price when it is bought at 5% less than cost price
5 95
erial, labou:_ =x-xx-=-x Profit and Loss
% and 10%1 100 100
~
ft G'1ven, -
105 95
t x-2-- x =95
- x x -10- 1. Cost of 3 cricket balls = cost of 2 pairs of leg pads.
u 100 100 100 '100
I Cost of 3 pairs of leg pads= cost of 2 pairs of gloves.
I 10 95 Cost of 3 pairs of gloves = cost of 2 cricket bats.
i ⇒ 100x- 2 =1ooox If a cricket bat costs Rs. 54, what is the cost of a
520
'
ri
{ x ==
2x1000
= Rs. 400
cricket ball?
(a) Rs. 12 (b) Rs. 14
~ 5
15 - A_ briefcase was sold at a prof it of 10%. If ,ts cost (c) Rs. 16 (d) Rs. 18
ls. 220
1
I
_ price was 5% less and it was sold for Rs. 7 more,
i th e gain would have been 20%. Find the cost price
of the briefcase.
2. There would be 10% loss if a toy is sold at Rs. 10.80
per piece. At what price should it be sold to earn a
profit of 20%?
I
38 @ Reasoning & Aptitude fflRDE ERS~ffl!
(a) Rs. 12 (b) Rs. i 2.96 (a) 2for a rupees (b) 1 for a rupees
(c) Rs. 14.40 (d) None of these (c) 4 for a rupee (d) 5 for a rupees
f
3. If books bought at prices ranging from 11. By selling 45 lemons for Rs. 40, a man loses 20% 19
Rs. 200 to Rs. 350 are sold at prices ranging from How many should he sell for Rs. 24 to gain 20% ir
Rs. 300 to Rs. 425, what is the greatest possible the transaction?
profit that might be made in selling eight books? (a) 16 (b) 18
(a) Rs. 400 (c) 20 (d) 22
(b) Rs. 600
(c) Cannot be determined 12. A man gains 10% by selling a certain article for,
(d) None of these certain price. If he sells it at double the price, thi 2C
profit made is
4. If the selling price of 18 articles is oquo.l to the C.P. of
(a) 20% (b) 120%
21 articles, the loss or gain percent is:
(c) 100% (d) 140%
2
(a) 16~% gain (b) 14 - o/co gain
. 13. A sells a bicycle to B at a profit of 20% and B sell1
3 7
it to Cat a profit of 25%. If C pays Rs. 1500, wha
2 did A pay for it? 21
(c) 16~% loss (d) 14-% loss
3 7 (a) Rs. 825 (b) Rs. 1000
(c) Rs. 1100 (d) Rs. 1125
5. A man sold 250 chairs and had a gain equal to selling '
price of 50 chairs. His profit percent is: 14. If the manufacturer gains 10%, the who!esa!e dea!et
(a) 5% (b) 10% 15% and the retailer 25%, then the cost of productia 22
(c) 25% (d) 50% of a table, whose retail price is Rs. 1265 is
(a) Rs. 632.50 (b) Rs. 800
6. If I purchased 11 books for Rs. 10 and sold all the
(c) Rs. 814 (d) Rs. 834.34
books at the rate of 10 books for Rs. 11, the profit
percent is 15. Two mixers and one TV. cost Rs. 7000, while two TV.! 22
(a) 10% (b) 11% am:Lamixer..c..os.lBs_9800'-----1b8...llalua.oLooe..LV.Js: ____~-
- - - - - - - - - ( c ) 21 % (d) 100% (a) Rs. 2800 (b) Rs. 2100
7. Ajay bought 15 kg ot dal at the rate of Rs. 1~ .50 per (r-) Rs 4?00 (cl) Rs. 8400
kg and 10 kg at the rate of Rs. i 3 per kg. He mixed
16. A horse and a cow were sold for Rs. 12000 eacr
the two and sold the mixture at the rate of Rs. 15
The horse was sold at a loss of 20% and the cowe
per kg. What was his total gain in this transaction?
a gain of 20%. The entire transaction resulted in 24
(a) Rs. i.10 (b) Rs. i6.50
(a) no loss m no gain
(c) Rs. 1 i (d) Rs. 27.50
(b) loss of Rs. 1000
8. Pure ghee costs Rs. 100 per kg. After adulterating (c) gain of Rs. 1000
it with vegetable oil costing Rs. 50 per kg, a (d) gain of Rs. 2000
shopkeeper sells the mixture at the rate of Rs. 96
per kg, thereby making a profit of 20%. In what ratio 17. An Article is sold at a certain price. By selling H25
does he mix the two? 2
(a) 1 : 2 (b) 3 : 2
3 of that price one loses 10%. The gain percente
(c) 3 : 1 (d) None of these
original price is
9. A dealer professing to sell his goods at cost price, uses
a 900 gm weight for a kilogram. His gain percent is (a) 20% (b) 33_:!_%
(a) 9 (b) 10 3
(b) 35% 27. A table is offered for Rs. 300 with 20% and 10% off.
5% (d) None of these If in addition, a discount of 5% is offered on cash
payment, then the cash price of the table is:
·ird of a consignment was sold at a profit of (a) Rs. 240 (b) Rs. 216
nd the remainder at a loss of 2%. If the total (c) Rs. 210 (d) Rs. 205.20
ifwas Rs. 400, the value of the consignment (in
28. A tradesman marks his goods 30% above the C.P.
ees)was:
20000 (b) 15000 1
If he allows a discount of 6 %, then his gain
(d) 10000 4
percent is:
ruit seller has 24 kg of apples. He sells a part of
'eat 20% gain and the balance at a loss of 5%. (a) 23~% (b) 22%
4
the whole he earns a profit of 10%, the amount
apples sold at loss is:
(d) None
. 6 kg (b) 4.6 kg
) 9.6 kg (d) 11.4 kg
29. What price should a shopkeeper mark on an article,
C. P. of an article is 40% of the S. P. The percent costing him Rs. 153, to gain 20% after allowing a
,t the S.P. is of CR is: discount of 15%?
; 250 ., (b) 240 (a) Rs. 224 (b) Rs.216
60 (d) 40 (c) Rs. 184 (d) Rs.162
<··•·an article is sold at 5% gain instead of 5% loss, the 30. If the S.P. of Rs. 24 results in a 20% discount on list
is r seller gets Rs. 6.72 more. The C.P. of the article is price, what S.P. would result in a 30% discount on
(a) Rs. 67.20 (b) Rs. 120 list price?
(c) Rs. 134.40 (d) Rs. 240 (a) Rs 27 (b) Rs. 21
(c) Rs. 20 (d) Rs. 9
etwoT.V.123.
~,,. A man bought an article and sold it at a gain of 5%.
ne T.V. is:* If he had bought it at 5% less and sold it for Rs. 1 31. A shopkeeper earns a profit of 12% on selling a
' less, he would have made a profit of 10%. The C.P. book at 10% discount on the printed price. The ratio
of the article was: of the cost price to the printed price of the book is:
(a) Rs. 100 (b) Rs. 150 (a) 50: 61 (b) 45: 56
:ooo eack (c) Rs. 200 (d) Rs. 500 (c) 99: 125 (d) 55: 69
the cows
:ulted in ,24. Raghu bought 4 dozen oranges at Rs. 12 per dozen
□□□□
and 2 dozen oranges at Rs. 16 per dozen. He sold
them all to earn 20% profit. At what price per dozen
. did he sell the oranges?
; (a) Rs. 14.40 (b) Rs. 16 Solutions
t
selling it~i
(c) Rs. 16.80 (d) Rs. 19.20
1. Ans. (c)
~5. The profit earned by selling an article for Rs. 900 is 3G = 54 x 2 = 108 ⇒ G = 36.
percentl double the loss incurred when the same article is
3P = 36 x 2 = 72 ⇒ P = 24
~- sold for Rs. 450. At what price should the article be
~ sold to make 25% profit? 3C = 24 x 2 48 ⇒ C = 16
j (a) Rs. 600 (b) Rs. 750 :. Cost of a cricket ball=Rs.16.
ft (c) Rs. 800 (d) Data inadequate 2. Ans. (c)
f 6 · A man sold an article for Rs. 75 and lost something.
90: 10.80 = 120 : x or
90
_
120
= ~-
J Had he sold it for Rs. 96, his gain would have been 10 80
and us~ double the former loss. The C.P. of the article is: :. x= 120x10.80 = 14 .4 0
etweigf (a) Rs. 81 (b) Rs. 82 90
I 8
(c) Rs. 3 (d) Rs. 85.50 Hence, S.P. = Rs. 14.40
l
40 ® Reasoning & Aptitude mRDE ERsm,
/
>M,~Jri"< "..
or
253
i
60
p = 1265.
/...1,-..,-..r-
l;:'OOXlbU l Rs. 800
.. ,.....,..,."\,
30 20 p =- r
0
- - " )-
0- - =
\ L.50 )
® Profit and Loss I 41
= ( 23~0~-r + ~~~) =
3y = 19600 - 7000 = 12600 or y = 4200
3 33008~-r
3.- :-. C. P. of a T. V. = Rs. 4200
2
308x _ x = 400 ⇒ 308x - 300x = 400
300 300
:. X = 300x400 = 15000
. C.P. of horse 8
2 576 5
- x = 110% of 24
New S.P. = -x, loss= 10% 20
3
576- 5x 264
or 576
20 10
5x = 528 or 5x = 48
20x
New, C.P. = , S.P. = Rs. x, or x = 9.6 kg
27
0 21. Ans. (a)
Gain = ( x _ ~ ; ) = ;; 40
C.P. = x S.P. ie. S.P. = ~C.P.
100 2
.
•·
Ga1n10 7xx -
. 01 = ( - 27x 100 ) °101 = 35°101
27 20x
= ( %x 100) % of C.P.
·
Ans. (c)
Let us consider a packet of rice marked 1 kg. :. S.P. = 250% of C.P.
( ~ "5 \
30. Ans. (b)
:. S.P. =RS.l~~ox600J=Rs.750
Let, the list price be Rs. x
26. Ans. (b) 80 X = 24 ⇒ X = 24 X 100 = 30
Let C.P. be Rs. x. 100 80 orin
2(x - 75) = (96 - x) Required S.P. = 70% of Rs.30 = Rs. 21
⇒ 3.r- 246 31. Ans.(b)
Let the C.P. be Rs. 100.
⇒ x=82
Then, S.P. = Rs. 112.
27. Ans. (d) Let the printed price be Rs. x
Cost price = 95% of 90% of 80% of Rs. 300. 90 ::a~
Then, 90% of x = 112 ⇒ -x==112 ::irn(
95 90 so 100
= ( -x--x-x300 =
\ 100 100 100 1Rs. 205.20
1
X = cI2;O100) : : I1:0
28. Ans. (c)
Let C. P. be Rs. 100. Then, marked price
1120
= Rs. 130. (C.P.): (Printed price)= 100:
9
= 900 : 1120 = 45 : 56
IHlili
Simple Interest & Compound Interest
f
l
C.I. = P[1+ 1~r-p
Where Pis principal amount
13800 = 12000 + i 800
Example 2.
An amount become double in 8 years calculate the
(. R is Rate of interest rate of interest.
i T is time duration Solution:
lase1: When interest ,s compounded annually then Let the amount be Rs. x
Jnount A will be Since it become double in 8 years so SI in 8 years
I
l
A~[1+ 1~0]
T
Now SI = P x Rx T, x = xx Rx 8
iO0 100
120
A= [1+-
R/ 2]2T :. R= 100=122%
9 8 2
100
Example 3.
. . se 3: When interest is compounded quarterly An amount become 5 times in 20 years at simple
A= [1+-
R/ 4]4T interest. Calculate the rate of interest given.
100 Solution:
se 4: When differential rate of interest is charged i.e. Principal = Rs. x
ate of interest is Total Amount = Rs. Sx. So
R1% for first year. SI= Rs. 4x
R2% for second year and
R3 % for third year then Now SI= PRT, 4 x= xxRx20
100 100
Amount = P(1 +~ ] x [1 +
100
£.g_J[
100
1+ R3 ]
100 R== 400 = 20%
20
DODO
44 @ Reasoning & Aptitude
D Solved Example: 2
4. What is the difference between the simple
on a principal of Rs. 500 being calculated at 5%
annum fOi 3 years and 4 % per annum for 4
1. Rs. 1200 is lent out at 5% per annum simple interest (a) Rs. 5 (b) Rs. 10
for 3 years. Find the amount after 3 years. (c) Rs. 20 (d) Rs. 40
(a) Rs. 1380 (b) Rs. 1290 Ans. (a)
(c) Rs. 1470 (d) Rs. 1200 P = Rs. 500, R1 = 5%, R2 = 4%,
Ans. (a) T1 = 3 years, T2 = 4 years
Here P = Rs. 1200
~--------R-=_5_'3/c_o_,T_=_3_y-ea_r_s_ _ _ _ _ _ _ _ _ _ _ _ _ _D-if-fe-r-en_c_e_=-:._M_
1_T-.:__-P-_~_ 2 = Rs. 5.
2 _·
an:
100 100
(a}
SI= PRT = 1200x5x3 = Rs.1S0
100 100 5. What is the simple interest for five years on a s1• (c)
of Rs. 700 if the rate of interest for the first 3 Ari
Amount= P +SI= 1200 + 180 = 1380
is 8% per annum and for another? yp,;:irs is 7%
2. Rs. 2100 is lent at compound interest of 5% per annum?
annum for 2 years. Find the amount after two years. (a) 400 (b) 392
(a) Rs. 2300 (b) Rs. 2315.25 (c) 352 (d) 266
(c) Rs. 2310 (d) None of these Ans. (d)
Ans. (b)
Here P = 2100, R = 5%, T = 2 years. 700 X 3 X 8 + 700 X 7 X 2 =
266
100 iO0
R lT
A=P [ 1+- . .[ 5
--'i'.IU0 1+-
]2 6. Find the compound interest on Rs. 1000 at the
1'::0J 100
of 20% per anntJm for 18 months when
21 21 compounded half-yearly.
= 2100x-x- == Rs. 2315.25
20 20 (a) Rs. 331 (b) Rs. 1331
(c) Rs. 320 (d) None of these (a)
3. Find the difference between the simple and the
Ans. (a) (c)
compound interest at 5% per annum for 2 years on
Here P = Rs. 1000, R = 20%,
a principal of Rs. 2000.
(a) 5 (b) 105 1
A r:::
T = 18 months or 1-;; years.
(
~\
.._,, '-t.J (d) None of these L
tll Simple Interest & Compound Interest I 45
tmd interest when interest is compounded Let Rs. x be amount invested at 12% Rate
y =xx12x1+(1500-x)x14 =
186
"· [ R/2]2T 100 100
= 1000 1+ 100 2 2
iQOO - x = 186 X = 1200
100 100 '
3
= 1000 [ 1+ 10 ] = Rs. 1331 10. Two equal sums were borrowed at 8% simple interest
100
per annum for 2 years and 3 years respectively. The
he principal if the interest compounded at the difference on the interest was Rs. 56. The sum
f 10% per annum for two years is R::r 420. borrowed were
s.2000 (b) Rs.2200 (a) Rs. G90 (b) Rs. 700
s. 1000 (d) Rs. 1100 (c) Rs. 740 (d) Rs. 780
;(a) Ans. (b)
ars
f =?,Cl= Rs. 420, R = 10%, T = 2 years. PRT PRT
- -1 - - -
2
..c.I=
·• ·
P[1+ ~]T
100
-P 100 100
p[ 8 X 3 - 8 X 2] = 56 p= ?00
20af 1:0)'-1] +
100 '
= p[ 1~~]
compound interest on some principal amount at 20%
per annum for 3 years in Rs. 48, then the principle
amount must be
p = 420 x 100 =Rs. 2000 (a) Rs. 550 (b) Rs. 500
21
(c) Rs. 375 (d) Rs. 400
· What is the rate of simple interest for the first 4 years Ans. (c)
if the sum of Rs. 360 becomes Rs. 540 in 9 years Here P = ?, R = 20%, T = 3 year
the rate of interest for the last 5 years is 6%? Difference Rs. 48
(aj 4% (b) 5%
R ]T
(c) 3% (d) 6% 48= Pi+-
[ - PPRT
--
Ans. (b) 100 100
P = Rs. 360, A = Rs. 540, SI = Rs. 180.
13. A sum of money doubles itself in 5 years. In how many (a) Rs. 4000, Rs.2000
years will it become four fold (if interest is compounded)? (b) Rs. 5000,Rs. 1000
(a) 15 (b) 10 (c) Rs. 3000, Rs. 3000
(c) 20 (d) 12 (d) None of these
Ans. (b) Ans. (a)
Let sum= x Let one part be Rs. x then
19.
5
X X 2 X 6 _ (6000 - X) X 3X 8
Then, 2t = x[1 + ~ ]
100 100 100
i 2.x = 144000 24.x
⇒ [1 + _B_J
iOO -
- 21/5 ".( i) 36x = 144000
X = 4000
Then, 2r = x [ 1 +
~13 17. San jay borrowed Rs. 900 at 4% p.a. and Rs. 11i
at 5% p.a. for tl1e same duration. I le had to pl
100J
Rs. 364 in all as interest. What is the time period in year: 1.
⇒ [1 + _B_J
100
= 2113 ". (i) (a) 5 years (b) 3 years
sx = x[1+ ~
1 0 r (c) 2 years
Ans. (d)
(d) 4 years
⇒
~
T
- =3
= 2Ti3
[
900 x 4T + i 100 x 5T
100 i00
l = 364
3
36 T + 55 T = 364
T = 9 yrs.
T = 4 years
Alternative:
If certain sum of money becomes 'm' times in V 18. If a certain sum of money becomes double at
years. Then it will become (mn) times in 'n x y' years. interest in i 2 years, what would be the rate of
Hence 23 in 3 x 3 = 9 years. per annum?
15. Divided Rs. 6000 into two parts so that simple
interest on the first part for 2 years at 6% p.a. may
(a) s2 (b) iO
3
be equal to the simple interest on tr·1e second part (c) i2 (d) 14
for 3 years at 8% p.a.
mADE EIRS!::j ® Simple Interest & Compound Interest I 47
11. The simple interest on a sum of money at the end of get the same amount after 2, 3 and 4 year
6. Ans. (c)
Let the sum be Rs. x. Then, Rate= (100 x 2x x _?_)% = (400)%
X 31 31
1 1
( xx 11 x ~x-- -x x 11x.I.x-- )
2 100 2 100
= 412.50 Now, sum= x, S.I.= x & Rate ( 400) o/c°.
31
22
or x = 412.50
200 . (100
Time= x - 31 ) years
- -XX
⇒ 11x = 41250 X 400
⇒ X = 4410 = 105. X X6 X 3 + .X X 8 X 5 + X X 10 X 2 =
1560
42 100 100 100
Money added = Rs. 105. or 78x = 156000
9. Ans. (a) or x = 2000
Let the investment be Rs. x, Then, 15. Ans. (c)
41 1 39 1 .
x x - x - - x x - x - = 25
180 x 40 =xx 30 or x = 240
8 100 8 100
16. Ans. (a)
⇒ 2x = 20000 ⇒ X = 10000.
Let the money lent at 8% be Rs. x. then,
10. Ans. (c) XX 8 XI+ (1550 x) X 6 X 1 =
Let sum= x, Then, S.I. = 3x 106
100 100
:. Rate= ( 100 x
xx 15
3
x)% = 20% or 2x + 9300 =10600 or x = 650.
17. Ans. (d)
11. Ans. (b) Let the sum at 5% be Rs. x. Then,
Then x x 2 x 5 _ y x 3 x 5 _ z x 4 x 5 _ 6. In how many years will a sum of Rs. 800 at 10% per
, -----------k
100 100 100 . annum compoundod semiannually bocomo
Rs. 926.10?
20
x=10k, y= k &z=Sk.
3 (a) 222 (b) 122
20
So, x: y: z =10k: k: 5k
3 (c) 223
= 30 : 20 : 15 = 6 : 4 : 3.
7. To find out the total compound interest on a sum of
□□□□ money after 5 years, which of the following
informations given in the statements P and O will
be sufficient?
Compound Interest P : The sum was Rs. 20000.
0 : The total amount of simple interest on the sum
=P+Px10x1= 11P_
□□□□ 100 10
11
p = 1320
10
Solutions 132
or P= 0x10 =Rs.1200.
~ 11
1. Ans. (d)
4. Ans. (a)
Let the sum be Rs. x. Then, Interest on Rs. 4624 for 1 year
3 = Rs. (6083.50 - 5290) = Rs. 793.50
Cl. = x ( 1+ 10 )
00 - x
1
Rate--(100x793.50) _
- - - - 0110- 6 -11010
4624x1 4
= ( 1331 _ x) = 331x. 2
1000 1000
Now, x ( 1+ x25 ) = 4624
XX 10 X 3 3x 4 100
S.I. = 100 10 17 17
or x x - x - = 4624
(C.1.)-(S.l.)=(331x _ 3x
1000 100
)=~-
1000
i6 16
Alternative:
2. Ans. (c)
or
216
x= 1270x216= 2160 _
Then, 800(1+ _2_)
100
n = 926.10
127
Thus, the sum is Rs. 2160. ⇒ (32) 2
n = 926.10 = 9261 = (~)
3
3
800
2n = 3 or n=- years.
8000 20
= Rs. 1080. 2
52 1111 Reasoning & Aptitude fflRD&: ER§Y
7. Ans. (d)
Clearly, both P and O together are needed. Using P = Rs{ 832x ~ x1)= Rs. 66.56.
1 0
and Q rate can be calculated as
Total difference = Rs. (32 + 66.56)
Rate= (i00x4000) = 4% = Rs. 98.56
5 x20000
11.Ans.(c)
Now, C.I. can be calculated.
Let the value of each installment be Rs. x.
8. Ans. (b) Then,
(P.W. of Rs. x due 1 year hence)+ (P.W. of Rs. x due
P = ( 1+ . 20 )n > 2P or (6)n
-; > 2 2 years hence) = Rs. 1100.
100
1
( + 3:~oo) + ( 1+ 3:~oo)'
---------(64-x¾ X¾)GITT~-+Gr1+-. ------------------------------+--~
X
10. Ans. (c) + 3 = 7620.
Diff. in C.I. & S.I. for 2 years= Rs. 32. (,+ :~oo)
3
S.I. for 1 year= Rs. 400.
S.I. on Rs. !JOO for 1 year= Rs.32. or 6x + 36x + 2i,6x = 1620 _
7 49 343
100x32) 294x + 252x + 216x = 7620 x 343
Rate= ( - - - % = 8%.
400x 1
7620x343
or x=----=3430.
Hence, diff. in C.I. and S.I. for 3rd year 762
= S.I. on Rs. 832
Amount of each installment= Rs:3430.
11111111111!11111
Ratio
~
similarly B will be p q part of total sum of the
Ratio is the relationship between the quantities of same
quantities.
kind. In ratio the quantities are compared as the multiple
or parts of other quantities. If ratio is a : b then a is Example 1.
called antecedent and b is called consequent. ratio of Given that for two quantities a & b, 2a = 3b. What
the quantities is expressed after removing the common will be the ratio ( 4a + 5b) : (2a + 3b )?
factor between the quantities. Solution:
Generally, the ratio is useful in comparision. If the a 3
quantities A & B are compared & their ratio comes out 2a = 3b ⇒ - = -
b 2
asp: q
⇒ a = 3K, b = 2K
We can say A/B = p/q
4a + 5b 4 x 3K + 5 x 2K
or A & B can be expressed as pK, qK respectively. so =
2a + 3b 2 x 3K + 3 x 2K
This provides a scope of comparison in terms of
22K 11
multiples of p & q.
= 12K = 6
In Equalities in the Ratio So the required ratio is 11 : 6
If a ratio is given as a : b & a quantity x is added in
both antecedent & consiquent then Example2. ·
If a : b = 3 : 5 & a : c = 6 : 7 then what is
a+x a
➔ -- >
b+x
b if a< b ... (i) a:b:c?
a+x a Solution.
➔ -- <
b+x
b if a> b ... (ii)
a:b=3:5
a+x a a:c=6:7
➔ -- = - if a= b
b+x b In this case the common quantity is
(i) and (ii) was considering x = +ve 'a' so we will make a's value as const.
If x = --ve (i) and (ii) inequalities will reverse. So we can say
a : b = 6 : i O (multiplying by 2)
Ratio as a part of total value
and a: c = 6: 7
If two quantities A & Bare in the ratio p : q then
so a : b : c = 6 : 10 : 7
A p
B = q⇒ A= pK, B = qK Example 3.
So A+ B = (p + q)K 10a 2 + ab 10
If ab b 2 = - what will be a: b
A p 3 1
So
A+B p+q (a) 2 : 3 (b) 2 : 5
(c) 3 : 4 (d) 3 : 7
or A= _P_(A+B)
p+q Solution.
~
So A wi 11 be p q part of total sum of the quantities
10a 2 + ab
2
:::::
iO
3ab-b
54 ® Reasoning &. Aptitude fflRDE ERS!::11
2 5 Solution.
⇒ X = -5 or -
2 For mean proportion 3, x, 27 are in proportion
Hence option (b) 3 X
i.e.
X 27
Types of Ratio
⇒ x = 3 x 27 or x = 9
2
If two quantities A & B have their ratio as
So mean proportion of 3 & 27 is 9
A : B = P : q then
For third proportion 3, 27, x are in proportion
e Duplicate ratio of A : B is P2 : q 2
3 27
11 Triplicate ratio of A : B is P3 : q 3 i.e.
27 X
11 Sub-duplicate ratio of A : B is Jp: Jq ⇒ 3x = 272
11 Sub-triplicate of A : B is VP: efq ⇒ X = 243
a-b c-d
Proportion b- = -d- (dividendo rule)
Proportion is comparison of two equal ratio when
the two ratios are equal then all the quantities
a+b c+d ___
a-b - c-d
comprising the ratios are called in proportion i.e. (componendo-dividendo rule)
a C b d
b = d then a, b, c & d are in proportion & 'd' is - = - (invertendo rule)
a C
Case 2. 4K 2
By adding water K + =
If the investment time is same for all the persons 10 1
⇒ 4 K = 2 K + 20
i.e. t 1 : t2 : t3 = 1 : 1 : i, then the profit will be shared
⇒ 2 K = 20 or K = 1O
in the ratio of their investment i.e. the ratio of profit
So amount of alcohol =4 x i O = 40 litre
will be
p1: 1-\: i·\ Example 3.
The present ratio of ages of Ram & Shyam is 5 : 4.
Example 1.
i 8 years ago the ratio of their ages was 16 : 11. The
A, B, C invest rupees 10000, 15000 & 20000
present sum of their ages is
respectively in a business for 2 years each what
(a) 90 years (b) 105 years
will be the ratio of their profit?
(c) 80 years (d) i 10 years
Solution.
Ans. (a)
Since the investment time is constant. So profit will Let the ages be 5 K, 4 K
be shared in the ratio of their investment i.e. ratio
5K-18 16
of profit .ik~=w = 11 ⇒ K = ·jo
= 10 K : 15 K : 20 K
So sum of the ages = 50 + 40 = 90
= 2: 3: 4
Hence option (a)
Case 3.
Example 4.
When the investment is variable we have to look for
The income of Ram and Shyam are in the ratio 2: 3
the effective investment value for the profit sharing.
and their expenditures are in the ratio 3 : 5. If each
_:__ _ _~-c-:c--:-r:c-:r------------------ssia~v7EelSs:RRs:-muo-menRam7s1ncome
Example 1. is
A, B, C invested rupees 10000, 15000 & 20000 (a) 8000 (b) 6000
respectively. Next year A & 8 invested 5000 more (c) 5 000 (d) 4ooo
while C withdrew 10000 rupees. What will be the Ans. (d)
ratio of their profit after 3 years? Let income be 2x, 3x
and expenditure ue 3y, 5y
Solution. 2x - 3y= 1000
Here we will see the effective investment value of 3x - Sy= 1000
A, B & C respectively. So x = 2000
Effective Investment Value of A So income is 4000
= 10 K + 15 K + 15 K Hence option (d)
= 40 K
EIV of B = i 5 K + 20 K + 20 K □□□□
= 55 K
EIV of C = 20 K + 10 K + 10 K
= 40 K
So ratio of profit will be Solved Examples
40 K : 55 K : 40 K
=8:11:8 1. Divide Rs. 500 among A, B, C and D so that A and B
together get thrice as much as C and D together, B
Example 2.
gets four times of what C gets and C gets i .5 times
A mixture contains alcohol and water in the ratio
as much as D. Now the value of what B gets is
4 : 1 by adding 10 litre of water the ratio becomes
(a) 300 (b) 75
2 : 1. Find out the amount of alcohol in the mixture
(c) 125 (d) Noneofthese
fflRDE ERS'!:::;11 ® Ratio, Proportion & Variation 57
= 6:4:3 =6:4:3 1 7
12 It is given that 12.x + 10x x
2 4 x = 75
+
280
= - x 6 = Rs. 420.
ITI Practice Exercise: I
4
1 . Amit, Nitin and Ravindra entered into a partnership.
6. Ans. (c) Amit invested Rs. i 6000 for 9 months. Nitin invested
We have, a : b = 7 : 5, c : d = 4 : 3 and x=6. Rs. 12000 for 6 months and Ravindra invested
bx(c-d) Rs. 8000 for 12 months. At the end of a year there
The present age of Mahesh = d b
a - C was a profit of Rs. 26000. Find the share of Nitin in
= 2ix13-8x31 =
5 5. A and B started a business with initial investments
(8+31)-(21+13) . in the ratio 5 : 7. If after one year their profits were in
9. Ans. (b) the ratio 1 : 2 and the period for A's investment was
Ratio of alcohol and water i 2 : 5 7 months, B invested the money for
Let their qualities be i 2x and 5x respectively. (a) 6 months (b) 2'
After adding the litres of water ratio becomes 4: 3 (c) 10 months (d) 4 months
12x 4
= 6. A, B, Center into a partnership with shares in the
5x+ 14 3
7 ratio ?_:-=!:.: ~- After 4 months, A increase his share
X = 2 2 3 5
7 by 50%. If the total profit at the end of one year be
Quantity of alcohol = 12 x = 42 litres.
2 Rs. 21600, then B's share in the profit is
(a) Rs. 2100 (b) Rs. 2400
□□□□ (c) Rs. 3600 (d) Rs. 4000
@ Ratio, Proportion & Variation I 61
7. Bis a sleeping partner and A working. A puts in Rs. Then, profit of A : profit of B : Profit of C
5000 and B puts in 6000. A received 12.5% of profit :::: C1 X t1 : C2 X t2 : C3 X t3
for managing the business and rest is divided in = 3x x 2y : 5x x 3y : 9x x y
proportion to their capitals. A's share of profit in a == 6: 15: 9 or, 2: 5: 3.
total profit of Rs. 880 is 4. Ans. (b)
(a) Rs. 350 (b) Rs. 400 We have, P 1 : P2 : P3 = 4 : 5 : 6 an~ t 1 : t2 : t3 == 2 :
(c) Rs. 420 (d) Rs. 460 3: 6.
8. A starts business with a capital of Rs. 1200. Band P P P 4 5 6
Required ratio = -1 : _1_ : -1. = - :- :-
C join with some investments after 3 and 6 months, t1 t2 t3 2 3 6
respectively. If at the end of a year, the profit is or, 12 : 1O : 6
divided in the ratio 2: 3: 5 respectively, what is B's Thus, A, Band C invested their capitals in the ratio
investment in the business? 12:10:6.
(a) Rs. 2400 (b) Rs. 1800 5. Ans. (c)
(c) Rs. 3600 (d) Rs. 6000 Let investments of A and B respectively be 5x and
7x and period of B's investment bey months.
□□□□ (5x) X 7 I
Then, (?x)xy= 2 ⇒ y=10.
Solutions
6. Ans. (d)
1. Ans. (c)
7 4 6
Here, C 1 = 16000, C2 =12000, C3 = 8000, Given ratio = - : - : - = 105: 40: 36
2 3 5
t 1 = 9, t2 = 6, t3 = 12 and P = 26000.
Nitin's share in the profit Let the initially invest Rs. 105, Rs. 40 and Rs. 36,
respectively.
C x t xP
::: - - - ' " 2
- - - -2- = - - - - Ratio of investments
= [105 X 4 + (150% Of 105) X 8]: (40 X 12):
12000 X 6 X 26000 (36 X 12)
= ------------
16000 X 9 + 12000 X 6 + 8000 X i 2 = 1680 : 480 : 432 = 35 : 10 : 9
8. Ans. (a)
or, x = 42 000x 3 = Rs. 9000.
2x7 Profit ratio of A, B and C is
(1200 X 12): (x X 9): (y X 6) = 2: 3: 5
3. Ans. (a)
Ratio of capitals of A, B and C are 3 : 5 : 9. Let the 1200x 12 9x
capitals of A, Band C be 3x, 5x and 9x, respectively.
⇒
2
=
3
Ratio of timing of their investments are
x = Rs. 2400.
2 : 3 : 1. Let A, Band C invest their capitals for 2y,
1111111111111'1!111111
3y and y months, respectively.
Average Alternate way of Calculation of Average
Let us say that a group contains n numbers P1' P2'
Average is a number which represents the general
P3 .... Then take a value which can be assumed as
characteristics of a set of numbers. It is a very effective
the mean value. Take the deviations of mean value
way of representing the entire group by a single value.
'x' from the given values. If deviations are Q 1' Q 2'
Average of the group is defined as:
03 .... 0 0 . Then the average will be
Sum of all items in the group
A verage = ---------------'=---=- 01 + 02 + 03 + .... +on
x+--~----''-----"'-
No. of items
n
Example 1.
Basic Points related with Average
6 Students have their weights as 60, 62, 63, 67, 69,
@ !f the value of each item is increased by the some
7 4 kg. Find their average weight.
value P, then the average of the group or items will
also increase by P. Solution:
Let us assume the mean value as 67, then average
Example 1.
will be
Let us take a group of 5 numbers 25, 30, 15, 35, 40
and each number is increased by 3, then find out 67+ -7-5-4+2+7
------he-oowave.r:agAc---------------------- 1 . 1 - - - - - - - - - - - - · - - · - · · · · · · -~
Solution:
This sort of average is called as weighted average Mixture & Alligation technique
and weighted average is calculated as When it comes to the mixing of two different
quantities. Weighted average comes into the picture.
average= If two solutions of concentration c 1 and c2 are mixed
in the ratio m : n. Then the weightage concentration
where n1, n2 , n3 ... are the number of elements in the cwill be
groups. While A 1 , A 2 , A3 .... are the average of the
mxc 1 +nxc 2
groups. C=
m+n
Instead of exact value of numbers of elements we
or we can rearrange the formula as
can also take the ratio of the number of elements.
Example 1.
The average marks of students of class A is 37
while that of class B is 23. If the number of students This is called as alligation formula. It can also be
in class A & B are 42 & 63 respectively then find understood with pictorial representation.
out the average marks of all the students taken SI
(I cone.)
nd
(II cone.)
together.
Solution:
2x37 +3x23
'•~/'' c (Weighted avg.cone.)
Average=
2+3
(·: since 42:63 ==2:3)
/~
74+69 =
143 = 28.6 (!In d Cone. - Weighted avg. Cone.): (Weighted avg. Cone. - I• Cone.)
= m n
5 5
The weighted average has wide range of
So it can be understood as the alligation technique
applications. The weightage need not be
provides us the ratio of mixing directly. Alligation
necessarily the number of elements. It can be any
technique has to be used when the weighted
characteristic of group that affect the group's
average is given & weighted average formula has
standing in the total. We will see some examples
to be used when weighted average is not given.
here.
Let us understand it better with some examples.
Example 2.
Example 1.
If two alloys having the ratio of copper & gold in the
How much water has to be added in 60 litre of 70%
ratio 3 : 4 and 2 : 5 are mixed and 14 kg of first
milk solution. To make it as 40% milk solution.
alloy is mixed with 21 kg of the second alloy. Find
out the new ratio of gold and copper in the mixture. Solution:
The concentration of water will be 30% and 60%
Solution:
respectively.
The ratio of mixing = 14: 21 = 2: 3
So this will be the weightage for the mixing. x litre ----+ 100% 30% -11> 60 litre
4 5
It is
7 and
7.
30 40
So the amount of gold in the mixture
So ratio of mixing is 3 : 4
2x4 3x5
-7- + -7- __23
X 3
==
2+3 - 35 ⇒ -60 - 4
So the ratio of gold to copper is 23 : 12. ⇒ x = 45 litres
64 e Reasoning & Aptitude fflRDE ERSW
== ix15+4x5
1+4
= 35 = %
5
7 D Solved Example
Example 3. 1. The average age of 24 students and the principal is
Ramesh deposits some amount in BIS bank at 7%
15 years. When the principal's age is excluded, the
interest rate p.a. & some part of amount in IBU bank,
average age decreases by 1 year. What is the age
at 9% p.a. & he gets Rs. 850 on Rs. 10000 yearly.
of principal?
Find out the amount invested in BIS bank.
(a) 38 (b) 40
Solution: (c) 39 (d) Data inadequate
850 out of 100000 means the net interest rate Ans. (c)
Average age of 24 students
= ~ x 100 = 8.5%
10000
sum of ages of 24 students
So BIS IBU = ------'----------
7~ /9 ⇒
14
24
/~
0.5 1.5
:. sum of ages of 24 students = 24 x 14
= 336 years
So the amount invested are in the ratio 1 : 3 for BIS Let the Age of principal be x year
&IBU Then, average age of 24 students and the principal
1
So the amount in BIS= - - x 10000 sum of ages of 24 students & principal
1+3 =------'--------------
25
= Rs. 2500
Example 4. ⇒ 15 = 336+ X
25
A Jar contains solution having milk and water in the
ratio 3 : 2. If out of 100 litres, 10 litre is taken out & ⇒ x = 25 x i 5 - 336 = 39 years
is replaced with water in one cycle. Find out the
amount of water after 2 cycles. 2. The average weight of 3 men A, Band C is 84 kg.
Another man D joins the group and the average now
Solution: becomes 80 kg. If another man E, whose weight is
Out of 100 litre solution.
3 kg more than that of D, replaces A then the average
3 weight of B,C,D and E becomes 78 kg. The weight of
Milk = - x 100 = 60 litre
5 A is
(a) 70kg (b) 72 kg
2 (c) 79 kg (d) 78 kg
Water = - x 100 = 40 litre
5 Ans. (c)
Now percent of solution which is being replaced Sum of weights of A,B and C
with water =3 x 84 = 252 kg
10 sum of weights of A.B. C and D
= -x100 = 10% = 4 x 80 = 320 kg
100
so amount of milk after 2 such cycles weight of D = 320 - 252 = 68 kg
=60x(i--0.i)(i 0.i) :. weight of E = 68 + 3 = 7i kg
= 48.6 litre Now, sum of weights of A,B, C, D and E
Net amount of water = 320 + 7i = 39i kg
= 100 - 48.6 = 51.4 litre sum of weights of B,C, D and E
= 4 x 78 = 312 kg
□□□□
:. weight of A= 39i -312 = 79 kg
66 ® Reasoning & Aptitude fflRDE ERS~
Then, 5x-3::::: 5x~ 15o+y 15. The average temperature of 1st , 2nd 3 rd December
5 was 24.4°C. The average temperature of the first
⇒ 5x - 15 = 5x - 150 + y two days was 24°C. The temperature on the 3rd of
⇒ y = 135 kg December was
(a) 20°C (b) 25°C
12. The average age of the Indian cricket team playing
(c) 25.2°C (d) None of these
the Nagpur test is 30. The average age of 5 of the
Ans. (c)
players is 27 and that of another set of 5 players,
Let the temp. on 3 rd of December be x C. Then
totally different from the first five, is 29. If it is the
captain who was not included in either of these two
groups, then find the age of the captain.
24.4 == 2 X 24 + X "' 25.2 oc
3
68 ® Reasoning & Aptitude fflRDE ERSY
16. The average of 20 results is 30 and that of 30 more 4. The average monthly salary of a staff of 9 persons
results is 20. For all the results taken together, the is Rs. 2450. One member of the staff whose monthly
average is salary is Rs. 2650. Find the average salary of the
(a) 25 (b) 50 remaining 8 persons of the staff.
(c) 12 (d) 24 (a) Rs. 2425 (b) Rs. 2625
Ans. (d) (c) Rs. 3025 (d) Rs. 2825
The required average
= 20 X 30 + 30 X 20 = 1200 = 24 5. 3 years ago the average age of a family of
50 50 5 members was 17 years. With the birth of a new
baby the average remains the same even today.
17. The average temperature on Monday, Tuesday and
Wednesday was 4 i °Candon Tuesday, Wedne::;Jc1y Find the age of the baby.
and Thursday it was 40°C. If on Thursday it was (a) i years (b) 3 years
exactly 39°C, then on Monday, the temperature was 1
(c) 2- years (d) 2 years
(a) 42°c (b) 46°c 2
(c) 23°C (d) 26°C 6. A batsmen in his "17th innings makes a score of 85,
Ans. (a) and thereby increases his average by 3. What is
The sum of temp. on Tuesday and Wednesday
his average after the i 7 th innings? He had never
=3 X 40 - 39 = 81°C been 'not out'.
The sum of temp. on Monday, Tuesday, and
(a) 47 (b) 37
\Nednesday = 3 x 41 - 81 = 42°C
(c) 39 (d) 43
3 kg more than that of D, replaces A, then average 18. The average age of a committee of 8 members is
weight of B, C, D and E becomes 79 kg. The weight 40 years. A member, aged 55 years, retired and he
of A is was replaced by a member aged 39 years. The
(a) 70 kg (b) 72 kg average age of the present committee is
(c) 75 kg (d) 80kg (a) 39 years (b) 38 years
(c) 36 years (d) 35 years
12. The average of marks obtained by i 20 candidates
was 35. If the average of marks of passed
□□□□
candidates was 39 and that of failed candidates
was i 5, the number of candidates who passed the
examination is
(a) 100 (b) 1iO
Solutions
(c) 120 (d) i50 1. Ans. (d)
Let the original expenditure = Rs. x
13. In a class, there are 20 boys whose average age is
decreased by 2 months, when one boy aged 18 Original average expenditure= ~
years in replaced by a new boy. The age of the new 35
boy is x+42
(a) 14 yrs. 8 months New average expenditure = ~
(b) 15 years
(c) 16 yrs 4 months X + 42 _
⇒ -X - - -- - -1
I ⇒ .\• -_ 420
(d) i7 yrs. iO months 35 42
:. Original expenditure = Rs. 420
14. The average daily wages of A, B and C is Rs. 120.
If B earns Rs. 40 more than C per day and A earns 2. Ans. (a)
double of what C earns per day, the wages of A per The total time taken can be calculated as shown
day is below:
(a) Rs. 80 (b) Rs. i 20
Distance Speed Time
(c) Rs. i 60 (d) Rs. 100
25km 500 km/hr 5 hrs.
15. With an average speed of 40km/hr a train reaches 1200 km 400 km/hr 3hrs.
its destination on time. If it goes with an average
500km 250 km/hr 2hrs.
speed of 35 km/hr, it is late by 15 minutes. The total
Total 4200 km 10hrs.
journey is
(a) 30 km (b) 40 km Total Distance
(c) 70 km (d) 80 km Average speed= Total Time
l!lllllllfllllill
or x = 35 x 40 = 70
' 5x4
In real life situations we come across practical problems of Example 2.
accomplishing the given project in prescribed time limit. A can finish a work in iO days & B can finish the
Since efficiency of different person is different, the same work in i 2 days. In how many days, they will
management has to take proper note of it before distributing finish the task working together.
the task to subordinates or executives. We discuss all such
Solution.
problems under the heading 'Time and work'.
Given:
In Time & Work, the problems are based on real life A takes 10 days
situations to allot the work between the subordinates B takes 12 days
based on their capacities. Since we can not assume the same capacity here.
The basic understanding of the problems comes by So we will assume the work. Let us assume the
quantizing a work. We will start understanding the work is 60 units.
quantization with the following example. 60
So capacity of A = = 6 units/day
Let us assume a person finishes 2 units of the work in 10
one day. So for finishing 60 units he will take 30 days. Capacity of B = ~~ = 5 units/day
So to find out the net time we should know the capacity
Combined capacity of A & B = 11 units/day
of the person & the net amount of work to be finished.
,zun,me & work problems depends upon one particular SotrmETTaKerroy-/\·& ~ · ~ ays
1i 11
relation i.e.
Example 3.
Work
Capacity = -T.- A finishes a task in 30 days. But with the help of B,
1me
he finishes the same task in 12 days. How much
So to find out any variables we need to know the other time B alone will tako to finish the task.
two variables. If they are not given they can be assumed
based on the situation. Let us start with a simple example. Solution.
l_et the work be 60 units
Example i. A Capacity =2unit 30 days
5 Persons finish one work in 7 days. In how many
A+ B Capacity =5unit 12 days
days 7 persons will finish the same amount of work.
B Capacity = 3unit ?
Solution.
Here we don't know how much amount of work, is 60
to be finished & what is the capacity of the
So the time taken B =
3 = 20 days
individuals. So we will assume the capacity.
Let us assume 1 person finishes i unit in 1 day Alternate ·work
Sometimes the availability of the person is limited.
⇒ 5 persons will finish 5 units in 1 day
in that case they work in a particular sequence i(
⇒ 5 persons will finish 35 units in 7 day can be understood with the following example. ·
⇒ The net work is 35 units
Example 1.
Now taking 7 persons,
A can finish a task in 12 days while B can finish a tac
they will finish 7 units/day in 15 days. In how many days both of them will finis
So time taken to finish 35 units = 35/7 = 5 days. the work if they work on alternate days started by'/;!,;
ffllRDE ERS~ @ Time & Work / 73
Solution. Solution.
Let work = 60 units Let the work = 60 units
A Capacity =5unit/day 12 days A Cap.=+6 units/day
iO days
Constructs
Now since they work on alternates day, the work Cap.=-3 units/day
C destructs
20 days
distribution will be in following manner for individual
6+5--3=8 units/day
day A+B+C 8 units/day
As all of them are working together.
5 + 4 + '--v--'
..........,,...
5+4+5 + 4 + .. .. .
'--v-'
= 60
. 60 1
So we can understand that in 2 days they finish
So time taken =- = 7-days.
8 2
9 unit and the work is being completed as a multiple
of 9 unit. Concept of Wages
So 9 x 6 units will be finished in 12 days. If the work is finished by more than one persons
So tl1e 60 units of work can be allocated as then the wages for them will be distributed among
them based 011 the amount of work done by them.
Work So
allocation 9 x 6 units + 5 units + 1unit = 60 units Ratio of wages = Ratio of work finished
1 (A+B) 1A 1B Now if all the persons are working for same no. of
Time
days then the ratio of work = ratio of capacities,
allocation 2x6days 1day 1/4day=13)'.jday
i.e. ratio of wages= ratio of capacities.
The inlet pipe always does the positive work while Example 2.
the outlet pipe always does the negagive work. Let A tank can be filled completely in 6 hours but
us see some problems. because of the leak at its bottom, it takes 2 more
hours to get filled. In how much time the leak can
Example 1.
empty a full tank?
Pipe A can fill a tank in 10 hours while pipe B can
empty a completely filled tank in 15 hours. How Solution.
much time it will take to fill a 2/3rd filled tank Pipe Cap. 4 litre/hr
6 hrs
completely by both of the pipes.
Cap. 3 litre/hr
Pipe+ Leak 8 hrs
Solution.
Tank 24 litre
Take volume ;: ; : 30 litres
Leak capacity== 1 litre/hour
Cap.=31itre/hr
A Fill
10 hours 24
Time = - = 24 hrs
Cap.= -21itre/hr
1
B Empty 15 hours
Example 3.
Capacity of (A + 8) = 3 - 2 = 1 litre/hr A monkey climbs on a greeced pole. In 1 minute it
Amount of water to be filled goes up by 3 meter while in the next 1 minute it comes
down by 2 meter if the height of the pole is 50 meter.
= (-½) x 30 = 20 litre In how many minutes the monkey will be at the top?
20 Solution.
So the time taken = - = 20 hrs.
1 When monkey will reach@ the height of 47 m. In
the next minute, it will reach at the top. So we will
Efficiency Comparison find out how much time the monkey takes to reach
When two person's efficiencies are compared then @ the height of 47 m.
in that case, the basic work and time calculations in first minute monkey moves= +3 m
become much easier. In this sort of case, we will is second minute monkey moves = -2 m
eaoteTo assume the capacities ct1rectly. Fo~r~---s-o-in_2_m_in_u_t_e_s_m_o_n__,.k_e_y_e_f_fe_c_t_iv_e_l_y_m_o_v_e_s----J1--·-
example: = i m
A Cap.3unit/ day
) 5 days Example 5.
Ram can finish 1/2 of a task in 9 days while Shya
Cap.1unit/ day
B > 15 days can finish ,;4 th of the same task in 3 days. Ho
Work 15 unit many days they will take to finish the work together
(A+ B)'s capacity= 4 unit/day
Solution.
15 3 Ram finishes 1/2 work. in 9 days
Time taken == - = 3- days
4 4 He will finish full work in 18 days
0 Time & Work I 75
Solution:
Here Man x Day = constant
mo x 50 = 5000
Initially 100 workers are working for 10 days So they
!1 b1 h1 M 0 H
1 1 1
can finish = =~--
l2 b 2 h2 M2 0 2 H2
100 x 10 = 1000 (Man.Day)
Now 4000 man - Day is still 1·emaining which has to
be finished by 200 workers Concept of Efficiency
So, 200 x Day= 4000
Efficiency of persons performing same task may not
4000 bo some. It may be differ<::ml for different persons. If
Day = - - = 20 days
200 efficiency is high a person can complete the work much
So total it requires iO + 20 = 30 days to complete faster than stipulated time period. Similarly if efficiency
the task. is low, it requires more time to finish the same task than
that of stipulated time.
Case-Ill.
We will observe following examples to have a glimpse
Let iOO workers can finish the task in 50 days. 100
of problems based on concept of efficiency.
workers started the task and working for 20 days.
Now because of unforeseen situations 80 workers Ex.1 A is three times as efficient as B and finish the
have to left the work Find total number of days task 32 days ahead of B. Find the number of
required to complete the task. days required to finish the task if both are 1
y
(a) 22( )days (b) 25(i)days
. .
remaining wor k = -3 part
4
3
B finishes
(c) 5(~}days (d) 12(; }days. 4 part in 10 days
2
. . . 10 40
And. (a) B f1n1shes 1 part 1n = days
314 3
Let man completes m part in a day and woman together they can finish it in
completes w part in a day then
1 1
4m+ 3w =
1
also ... (i) 1
--+-
3
=
1- = 8 days
6 20 40 8
1
5m+ 7w =
4 ... (ii) 5. Twenty workers can finish a piece of work in 30 days.
After how many days should 5 workers leave the
After simplifying we get
job so that the work is completed in 35 days?
5
20 m + 15 w = - (a) 5 days (b) iO days
6 (c) 15days (d) 20days
20 m + 28 w = 1
Ans. (c)
1 Man x Day = Man . Day
So 13 w = -
6 20 X 30 = 600
1 Let 5 workers leave after x days then
w=- ... (iii)
78 20x + 15 (35-x) = 600
From (i) and (iii) we get 5x + 525 = 600
5 5x = 75, X::: 15
m=-
156
6. Subhash can copy 50 page in 10 hours; sub hash
5 1 7
m+W= - + - = - and Prakash together can copy 300 pages in 40 hours.
156 78 156 In how much time can Prakash copy 30 pages?
. . 156 d 2 (a) 13h (b) 12h
So 1t require - ays = 22 -
7 7 (c) 1 i h (d) 9 h
78 ® Reasoning & Aptitude ffiRDE ERS!::11
88 144 54x12
(a) -day (b) -days x = --- , x = 8 days
17 17 81
. 12x8
72 together they will tnko = - - = 4.8 Jays.
(c) -days (d) 8 days 12+ 8
17
11. Raju is twice as Vijay. Together, they finish the work
Ans. (b)
in 14 days. In how many days can Vijay alone do
(1/2) i the same work?
Priya's days work =
8 =i6 (a) 16 days (b) 21 days
Preeti's days work (c) 32 days (d) 42 days
13. A can do a work in 1O days and B can do the same 16. A cistern is normally filled in 5 hours. However, it
work in 20 days. They work together for 5 days and takes 6 hours when there is leak in its bottom. If the
then A goes away. In how many more days will B cistern is full, in what time shall the leak empty it?
finish the work? (a) 6 h (b) 5 h
(a) 5 days (b) 6.5 days (c) 30 h (d) 15 h
1 Ans. (c)
(c) 10 days (d) 8- days
Ans. (a)
In 5 days the can complete
3
In one hour i part is filled now leak can empty in x
hour then
i ] x5 = -part
1+ - 3
[-
10 20 4 1 1 1
--- = --
5 X 6
1
Remaining is
4 part 1 1
14. 15 men could finish a piece of work in 21 0 days. But 17. Pipe A and B running together can fill a cistern in 6
at the end of 100 days, 15 additional men are minutes. If B takes 5 minutes more than A to fill the
employed. In how many more days will the work be cistern, then the time in which A and B will fill the
complete? cistern separately will be respectively?
(a) 80 days (b) 60 days (a) 15 min, 20 min (b) 15 min, 10 min
(c) 55 days (d) 50 days (c) 10 mir., 15 min (d) 25 min, 20 min
80 I ® Reasoning & Aptilude mRDE ERS~
or
x(x+ 5) =6
2x+5 1. 10 men can complete a piece of work in 15 days
X = iO, X + 5 = i5 and 15 women can complete the same work in 12
days. If all the 1Omen and 15 women work together,
18. There are two pipes in a tank. Pipe A is for filling
in how many will the work get completed?
the tank & Pipe B is for emptying the tank. If A can
fill the tank in 10 hours and B can empty the tank in 2 1
(a) 6 days (b) 8
15 hours then find how many hours will it take to 3 3 days
completely fill a half empty tank? 2
(c) 7 days (d) None of these
(a) 30 hours (b) 15 hours 3
(c) 20 hours (d) 33.33 hours
2. A is thrice as good as B and is therefore able to
Ans. (b)
finish a piece of wmk in 60 days less than B. Find
1 i 5 1 the time in which they can do it, working together.
---=-=-
10 15 150 30
So it takes 30 hours to fiii the tank and 15 hours to (a) 22~ days (b) 222 days
4 2
fill half the tank
(c) 24 days (d) None of these
19. There are three Taps A, Band Cina tank. They can
fill the tank in 10 hrs, 20 hrs and 25 hrs respectively. 3. Ramesh takes twice as much time as Mahesh and
At first, all of them are opened simultaneously. Then thrice as much time as Suresh to complete a job. If
after 2 hours tap C is closed and A and B are kept working together, they can complete the job in 4
-------rt::tftfltrtg:-After-ttw-4thttooi,i"ap-&fs-a+so-closecr:--'fh=e---~daysrtber::\.-the-tlme-taker::i-b-~acl+Qf_tJ:i@A+~a+at@lY---'--:
remaining work is done by Tap A alone. Find the to complete the work is ;
percentage of the work done by Tap A by itself. (a) 36, 24 and 16 days
(a) 32 % (b) 52 % (b) 20, 16 and 12 days
(c) 75 % (d) None of these (c) 24, 42 and 18 days
Ans. (d) (d) None of these
1?0
work in ·-
xx ' x- 43 days and Ajay and Sunil can
X
2 3
== - - - - - - - days
X X X X complete the work in 10 days.
xx-+-x--+xx-
2 2 3 3 Sanjay alone can complete the work in
x3/6
.1•e• - x days 120 X 10
- or. -
x2 · 6 13 == 120 days.
. X
10-~20
G1ven : - == 4. :. x == 24 13
6
24
Ramesh takes 24 days. Mahesh takes
2 or
1
~ 2
of the work is done by San jay in \ C!. = 1o
2
24 days.
12 days and Suresh takes = 8 days to finish
3
the work. 6. Ans. (b)
Bansal, Gupta! and Singha! together can finish the
4. Ans. (b)
work in 4 days.
1 Man can complete the work in 5 x 2 = i Q . d e y s . - - - - - - - - - - - - - - - - - - - - - -
1 woman can complete the work in 4 x 3 = 12 days. 24
Bansal and Gupta together can do it in days.
And 1 child can complete the work in 5 x 3 = 15 5
days. Gupta and Singha! together can do it in 8 days. 1
1 man, 1 woman and 1 child, working together,
can complete the work in
Therefore, Bansal alone can complete the work in
j
xyz d
xy X 4 "1
= - - days == - - davs
(8 l
- - - - ays y-x · \8-4) ,
xy+yz+zx
[Here, x = 4 and y = 8 days
== 10 x 12 xi 5 == 4 days
Also, Singha! alone can complete the work in
10x12+i2xi5+15x10
l
5. Ans. (b)
Ajay, sunil and Sanjay, working together, can
complete the work in
= f244 x4J days
(x:_Y)days= --x4
5
. 2xyz \
== (
xy + yz + zx
·-J (,7,31/S
24
Here,x=4and y= =24days.
( 2 X 10 X I 5 X 20 )\
5
l
== iO x 15 + 15 x 20 + 20 x 10
Bansal and Singha/ can complete the work in
J
Work done by all of them together in 6 days [Here x = 24 and y = 6 days.
-
6xi3
- - !.8. -
120
13
20
Gupta alone can complete the work ~C~- ~ 12.
fflRDE EAS!:11 0 Time & Work I 83
7. Ans. (a) Work done by a man and boy are in the ratio
2: 1
We have M 1 == 24 D 1 = 27, W 1 = i, t 1 = 7,
M2 = 14, D2 = ?, W2 = 1, t2 = 9, 12. Ans. (b)
M1D1t1W2 = M2D2t2W1 1 1 7
(A + B)'s 2 days' work + = Evidently, the
⇒ 24 X 27 X 7 XI :::: 14 X 0 2 X 9 XI 9 7 36
⇒ D2 = 36 days. work done by A and B during 5 pairs of days
Work done in 8 x 2 = i6 days simultaneously, in how much time the tank will be
67x8 67 filled?
= 600 = 75 (a) 7 hours 30 minutes
67 8 (b) 6 hours 40 minutes
Work left = 1 - = (c) 8 hours 30 minutes
75 75
On i 7 th day(A + B) will work and they will finish (d) None of these
8
5. A cistern is normally filled in 8 hours but takes
150
2 hours longer to fill because of a leak in its bottom.
8 8 8 4 If the cistern is full, the leak will empty it in
Work l e f t = - - - = - = -
75 150 150 75 (a) 35 hours (b) 45 hours
On 18th day (A + C) will work and they will finish (c) 40 hours (d) None of these
2. A tap can fill a tank in 25 minutes and another 9. A cistern is provided by two taps A and B. A can fill
can empty it in 50 minutes. Find whether the tank it in 20 minutes and B in 25 taps minutes. Both the
will be filled up or emptied and in how many taps are kept open for 5 minutes and then the second
minutes? is turned off. The cistern will be completely filled in
(a) Tank is filled up in 50 minutes another
(b) Tank is emptied in 25 minutes (a) 11 minutes (b) 10 minutes
(c) Tank is filled up in 25 minutes (c) 15 minutes (d) 12rninutes
(d) None of these
10. A cistern has two taps which fill it in 12 minutes and
3. Two taps A and B can fill a tank in 1O hours and 15 15 minutes respectively. There is also a waste pipe
hours, respectively. If both the taps are opened in the cistern. When all the pipes are opened, the
together the tank will be full in: empty cistern is full 20 minutes. How long will the
(a) 8 hours (b) 6 hours waste pipe take to empty a full cistern?
(c) 5hours (d) Noneofthese (a) 8 minutes (b) 1O minutes
I
(c) 12 minutes (d) 16 minutes
4. Two pipes can fill a tank in 10 hours and 12 hours,
respectively. While a third pipe can empty the full
tank in 20 hours. If all the three pipes operate □□□□
mRDE ERS'!::ll @ Time & Work I 85
Solutions ⇒ x(x-10)=12(x+x-10)
1. Ans. (a) ⇒ x2 - 34x + 120 = 0
Here, x = 2 and y = 3
or (x - 30)(x - 4) = 0
Part of the cistern filled in i hour
x = 30 or x = 4
1 1 i i 1
=---= - - = - The faster pipe takes 30 hours to fill the reservoir.
x y 2 3 6
Total time taken to fill the cistern = 6 hrs. 7. Ans. (b)
As pipe 'A' is 3 times faster than pipe 'B'. So if 'A'
2. Ans. (a) takes 'x' min, 'B' will take '3x' min. to fill tank.
Here, x = 25 and y = 50 According to question 3x - x = 2x = 32, x = 16,
Part of the tank filled or emptied in 1 minutes
4
1 1 1 1 1 3x=-.
::: =---=- 8
X Y 25 5Q 50
Both pipe together can fill tank in
Which is positive, therefore the tank will be filled.
1
Total time taken to fill the tank
1 1
= 12 min.
= 50 minutes -+--
i6 48
3. Ans. (b) 8. Ans. (a)
Here, x = iO and y = 15. As the pipes are operating alternately, thus their
The tank will be full in
. . b. 1 1 5
2 minutes JO 1s -+- = -
=(_!'j_)hours 4 6 12
x+y
5
= (iOx 1~) hours or6 hours. In the next 2 minutes the pipes can fill another
12
10+15
part of cistern. Therefore, in 4 minutes the two pipes
4. Ans. (a)
which are operating alternately will fill
Here, x = iO, y = 12 and z = -20
The tank will be full in 5 5 10 5
- +- =- = - part.
12 12 12 6
= ( -xxyxz)
- - - hours
xy-yz-zx
The part of the cistern left unfilled = 1- ~ = 2
6 6
iQ X 12 X -20
=( --- - - - - - - )' hours
10 X 12 - 12 X 20 - 20 X 10 1
Pipe A can fill
4 of the cistern in 1 minutes.
1
= ( ;) hours or, 7 hours 30 minutes 1
Pipe A can fill
6 of the cistern in
5. Ans. (c) 1 2
4x- = minutes
Here, x = 8 and y = 8 + 2 = 10. 6 3
The leak will empty the cistern in Total time taken to fill the cistern
2 2
= ( -xy)
- hours 4+ =4 min.
y-x 3 3
~ 11) ~
3 th0 f11II t:::rnk in 6 minutes, A and Baro J,;opt open for
= (i I 2~ == ( 20 - -2~) = 1 5 minutes in the begining and then C is also opened.
In what time is the cistern emptied?
Waste pipe can empty the cistern in 1Ominutes.
(a) 30 min. (b) 33 min.
1. Two pipes can fill a tank in 10 hours and 12 hours 8. A leak in the bottom of tank can empty the full tank
respectively while a third pipe empties the full tank in 8 hours. An inlet pipe fills water at the rate of 6
---+rt--2&--rt&tirs-:-H--a-lt--the4hre-e--pi-p es operate lrrers per m1nute~-Wnen-fnefank is full, the inlet is
simultaneously, in how much time the tank will be opened and due to the leak, the tank is empty in 12
filled? hours. How many liters does the cistern hold?
(a) 7 hrs (b) 8 hrs (a) 7580 (b) 7960
(c) 7 hrs 30 min. (d) 8 hrs 30 min. (c) 8290 (d) 8640
2. An electric pump can fill a tank in 3 hours. Because 9. A cistern has two taps which fill it in i 2 min. and 15
of a leak in the tank, it took 3½ hours to fill the tank. min, respectively. There is also a waste pipe in the
The leak can drain out all the water of the tank in: cistern. When all the three are opened, the empty
(a) 10:30 hrs (b) 12 hrs cistern is full in 20 minutes. How long will the waste
0) 21hrs ~) 24hrs pipe take to empty the full cistern?
(a) 8 min (b) 10 min
3. If two pipes function simultaneously, the reservoir (c) 12 min (d) 16 min
will be filled in 12 hours. One pipe fills the reservoir
1O hours faster than the other. How many hours it 10. Two pipes A and B can fill a cistern in 12 minutes
takes the second pipes to fill the reservoir? and 16 minutes respectively. If both the pipes are
(a) 25 hrs (b) 28 hrs opened together, then after how much time B should
0) 30hrs ~) 35hrs be closed so that the tank is full in 9 minutes?
1
4. 12 buckets of water fill a tank when the capacity of (a) 3 (b) 4min.
each bucket is 13.5 liters. How many buckets will
2 min.
1 3
be needed to fill the same tank, if the capacity of (c) 4 min. (d) 4-
2 4 min .
each bucket is 9 liters?
(a) 8 (b) 16
□□□□
(c) 15 (d) 18
fflRDE ERS~ ® Time & Work I 87
Solutions 1
Now, part is emptied in 1 min
60
1. Ans. (c)
Net part filled in 1 hour ¾part will be emptied on ( 60x¾}
= 7 hrs. 30 min.
Part filled in 2 hours = *= i
2. Ans. (c) . . part = (1- 1) = 2
Rema1n1ng
Work done by the leak in 1 hour 3 3
(A+ B)'s 7 hour's work= 2/3
=(i-~)= ;1 (A+ B)' s i hour's work = 2/21
Average Speed
For calculating the average speed the first thing
that has to be wiped out from the mind is that the
Time taken for first distance = _Q_
averge speed is not the average value of different· 90
speeds. It is the ratio of the total distance travelled
20
& the total time taken to cover that distance. Time taken for second distance= = J2..
60 30
Total distance
Average speed= - - - - - Net distance = D + 2D = 3D
Total time
fflROE IERS~ ® Time, Speed & Distance / 89
30 30 ⇒ 200 + L0 = 500
So average speed = 0 0
= 0+30 or L0 = 300 m
-+--- ---
90 30 90 Rivers & Boats
30 270 For the case of boats and river the condition is
=
40
x 90 ==
4 = 67.5 km/hr similar as of the trains problems. Only thing to
understand is
Relative Speed
0 Upstream ➔ Moving against the river flow
Relative speed is very good application part of TSO.
• Downstream ➔ Moving along the river flow
Relative speed is defined as the speed of one
moving body with respect to the another moving If U0 is speed of object and Ur is the speed of river
then
body.
If two bodies are running with the speed a and b for upstream ➔ relative speed is U 0 - Ur
then the relative speed of one body when for downstream ➔ relative speed is U0 + Ur
• They are moving on same plane (FOR is same)
Example 1.
( a - b) for same direction
A person covers a distance downstream in 3 hrs.
( a + b) for opposite direction
While returns back in 9 hrs. If the speed of river is
• They are moving as one on another
10 m/s find out speed of fv1an.
(As man on an moving escalator)
( a + b) for same direction Solution.
( a - b) for opposite direction Upstream speed = UO - 10
The basic funda of using relative speed in the Downstream speed = U0 + 10
problems is make one body as constant and let
Since the distance covered are same
the other bodies run with the relative speed.
(U 0 - 10)9 = 3(U 0 + 10)
Problems on Trains ⇒ 9U 0 = 3U 0 + 30 + 90
When a train crosses any body it has to cover its ⇒ 9U 0 = 120
own length apart from the length of other object so U0 = 20 m/s
relative distance will be the sum of length of train
Circular Tracks
and length of the object if LT is length of train, L0 is
When talking about circular track the first thing to
length of the object UT is speed of train and U0 is
speed of object then the time taken for the crossing be kept in mind is the time interval between any
T :::: LCM of
s
fDa' -D}
1- b
Soiution.
----+ t hrs. -+ 4 hrs
A p B
9 hrs +-- t hrs +--
Let us assume that train takes 't' time to reach at
Case 2. 'P', then
When more than two persons are running then
Ts 10 8 prn
Son .:.c -::.:--5
. T. 2 10 am
□□□□
3. Rajdhani Express travels 650 km in 5 hand another
940 km in i Oh. What is the average speed of train?
(a) 1590 km/h (b) 63 km/h
(c) i06 km/h (d) 126 km/h
Solved Exampls
Ans. (c)
Total Distance Travelled
1. Walking at 3/4 of his normal speed, Ankur is A verage spee d = - - - - - - - - -
Total Time taken
16 minutes late in reaching his office. The usual time
650 + 940
taken by him to cover the distance between his home = = i06 km/h
and his office is 15
(a) 48 minutes (b) 60 minutes 4. A car travels from A to B at V 1 km/h, travels back
(c) 42minutes (d) 62 minutes from B to A at V2 km/h and again goes back from A
Ans. (a) to Bat V2 km/h. The average speed of the car is:
Speed x Time = Distance (a) 2V1V2 (b) 2V1V2
SxT=D V1+ 2V2 V2 + 2V1
Ans. (c) The whole journey took 5 hours. What is the distance
Total Distance he covered on the car?
Average speed:;:; ~.~Total Time - (a) 12 km (b) 30 km
(c) 15 km (d) 6 km
3D 3 _ 3VN2 Ana. (a)
- D ·~ Q + o = . T; 2 . ""' 2v; +v; Let Distance be D km
V1 V2 V2 V1 V2
5. Narayan Murthy walking at a speed of 20 km/h o(~. 1 +2);;;: s
. 12 3
reaches his college 1o minutes late, Next time he
5 12
increases his speed by 5 km/h, but finds that he Is D
. =e • >< : ""' 12 km
5 .
still late by 4 minutes. Wh~t is the distance of his
college from his house, 8. Two tains A and B start simultaneously in the opposite
(a) 20 i\m (b) 6 km direction from two points Aand B and arrive at their
(c) 12 km (d) None of these destinations 9 and 4 hours respectively after their
Ans. (d) meeting each other. At what rate does the second
Let distance be D km train B travel If the first train travels at 80 km per hour.
D . 10 (a) 60 km/h (b) 100 km/h
=""T+= ... (i) (c) 120 km/h (d) None of these
20 60
A.ns. (c)
4
Als o g
25
:;:; T + 60
-
"'
('1i)
Dx~"'~
100 10 Here
- - - - -~- 1_ ? ? - , ; , - H } k m - - - - - - - - - - - - - - - - - - - - - - - -
10 80 J4 80 2
6. A motor car does a journey in i 7.5 hours, covering s2 -:':' Jg Sg 3
the first half a.t 30 km/h and th~ second half at s2 ""' 120 km/hour
40 km/h, Find the distance of the journey.
(a) 684 km (b) 600 km 9. A journay of i~? km tak~s 2 hours less by a fast
(c) 120 km (d) 540 km train than by a slow train. If the average speed of ,
Ans. (b) the slow train be i 6 kmph less than that of fast train,
Here Total time;;;: 17.5 hours what is the average speed of the faster train,
let total Distance be 2 D km (a) 32 kmph (b) 16 kmph
D D (c) 12 kmph (d) 48 kmph
then~+=~ i7,5 A
· 30 40 . ns. (d}
•·.
let speed of fast train be S km/hour then
D [. 3··.10·· + _410:. ] ;,; i 7.5
192 192
---~1c2 ... (i)
7 8=16 S ..
DX 126 :s; 17,5
This type. ot questions can be dlrectiy solved by
D 1_7_,~~ i ~9-
;:\a
going through option.
7 Hea.re using equation (i) and putlng options,
D,;,; 300km We get S ;;;; 48 km/hour.
Total Distance;;; 2D 600 km
80 15 4 15
--=-- ⇒ -::::--
60 SB -5 3 S8 -5
'10 6
--+--=4hour ... (i)
S8 + 1 SB -i SB= 16.25 ktr1/h
Going by option and (i) we get, 21. in a stream, B lies in between A and C such that it
SB = 4 km/hour is equidistant from both A and C. A boat can go
from A to B and back in 6 hr 30 minutes while it
18. Two trains are running on parallel lines in the same
goes from A to C in 9 hr. How long would it take to
direction at speeds of 40 kmph and 20 kmph
go from C to A?
respectively. The faster train crosses a man in the
I
(a) 3.75 hr (b) 4 hr
second train in 36 seconds. The length of the faster
(c) 4.25 hr (d) 4.5 hr
train is
® Time, Speed & Distance I 95
60
stoppage it will take = 48 minuts
75
Let AB = D km then So, stoppage= 12 minutes
BC= D km
24. A boat rows i 6 km up the stream and 30 km down
AC= 20 km
stream taking 5 h each time. The velocity of the
D D i3 current
---+ = ... (i)
S8 -Ss S8 +Ss 2 (a) 1. 1 km/h (b) 1.2 km/h
(c) 1.4 km/h (d) i.5 km/h
... (ii) Ans. (c)
i6
- - - = 5 hour
D 9 Ss -Ss
So - - - = ... (iii)
S8 -Ss 2
from (i) and (iii) we get, = 5 hour
9 D 13
-+---= 16 = 5S 8 - 5 Ss ... (i)
2 S8 + Ss 2
30 = 5 S8 + 5 Ss ". (ii)
D 14= iOSs
---=2
Ss +Ss Ss = 1.4 km/hr
to go from C to A it takes
25. A man can row 30 km upstream and 44 km
2D downstream in i O hours. It is also known that he
= 4 hours
S8 +Ss can row 40 km upstream and 55 km downstream in
13 hours. Find the speed of the man in still water.
22. Two trains are travelling in the same direction at
(a) 4 km/h (b) 6 km/h
50 km/h and 30 km/h respectively. The faster train
(c) 8 km/h (d) 12 km/h
crosses a man in the slower train in 18 seconds. Find
Ans. (c)
the length of the faster train.
(a) 0.1 km (b) 1 km 30 44
+ = 1o also
(c) 1.5 km (d) 1.4 km S8 -Ss S 8 + S3
Ans. (a)
t = LT+ Lo
ST -So
50 - 30 = 20 km/hr = 50/9 m/s 1
Let---= x and - - = y
Ss -ST Ss +Ss
now 18 = ⇒ LT = ·100 m Then
5019
30x + 44y = iO ... (i)
23. Without stoppage, a train travels at an average
speed of 75 km/h and with stoppages it covers the
40x + 165 = 13 ... (ii)
26. In a stream that is running at 2 km/h, a man goes Time, Speed & Distance
i O km upstream and comes back to the starting
point in 55 minutes. Find the speed of the man in
still water.
(a) 20 km/h (b) 22 km/h I Practice Exercise: I
(a) 65 minutes (b) 60 minutes to pass the other completely. If they are moving in
(c) 70 minutes (d) None of these opposite direction, they pass each other completely
in 3 seconds. Find the speed of each train.
8. A car starts from A for B travelling 20 km an hour.
(a) 42 m/sec., 38 m/sec.
1 . (b) 38 m/sec., 36 m/seo.
1 hours later another car starts from A and·
2 (c) 36 m/sec,, 42 m/sec.
travelling at the rate of 30 km an hour reaches 8 (d) None of these
what should be his speed to cover the remaining 7 (64 + x) = 16x or 9x = 448
distance in the remaining time? 448
(a) 16 km/hr (b) 8 km/hr
x = - = 49.78 km/hr.
9
(c) 12 km/hr (d) 14 km/hr
5. Ans. (a)
21. A train 110 metres in length passes a man walking
at the speed of 6 km/hr. against it in 6 seconds. The Rakesh's speed _Jr;_ !IB _4
speed of the train in km per hour is Suresh's speed - ff; - .Jg -3
(a) 60 km/hr (b) 45 km/hr
(c) 50 km/hr (d) 55 km/hr 3
Suresh's speed =
4 Rakesh's speed
□□□□
3
= x16 = 12km/hr
4
1. Ans. (b) 30 45
Here, T1 = -, T2 =-, T,= 2, s 1= 40
Let the length of the train be x m. 60 60 °
:. Total distance covered by the train= (x + 150)m
S2 = 60 and S3 = 70.
Speed of the train = 60 km/hr Tr-1e average speed of the car
5 50 S1T1+S2 T2+S3 T3
= 60 x-= -m/sec =-----
18 3 T1+ T2+ T3
Since, Distance= Speed x time
30 45
40 X - + 60 X -
+ 70 X 2
50
x + 150 = x 18 = 300 = ----"6--"'0-- - -6:c...:0;___ _
3 30 + 45 +2
--------------------------~u--·my----
or, x = 300-150 = 150 m.
= 150 m.
Length of the train = 63 km/hr.
. . . Change in time
=~X4=15m.
⇒ 0 ng1na 1time=
--1
(b )
4 a
Distance 15 /
Spee d = - - - m sec 20
Time 2 = -.- -. = 60 minutes
'
r
\U
t- 1 1
/
8. Ans. (c)
4. Ans. (c) Here, difference in speed = 30 - 20 = i o.
Let the speed on the return journey be x km/hr.
Difference in time= 22+
2
,22 = 4.
2x 64x x
64+.x And product of speed= 20 x 30 = 600.
We have,
®
.
Time, Speed & Distance I 99
s -s 42-28
Stoppage time/hr = - 1- -2- = - - -
s1 42
1
=
3 hour= 20 minutes = ( "130 + 1 i0)(60-3)
x = 38 m/sec.
2 60 3
11.Ans.(b)
Here, L 1 = 300 m, L 2 = 200 m, 15. Ans. (d)
s 1 = 90 km/hr and s 2 = 60 km/hr Time from 7 A.M. to 8 A.M. = 1 hour.
Therefore, time of their meeting
5
s 1- s2 = 90 - 60 = 30 km/hr = 30 x i8 m/s
= (d+ s2T) hr= (100 +25 x
S1 + S2 20+25
1) hr
L1 + L2 300 + 200
Time taken = =
30x~ = 2 hrs 47 min. after 7 A.M.
18
16. Ans. (b)
= 500 x 18 = 60 sec.
30x 5 Distance between Mumbai and Pune
17.Ans. (b)
⇒ = 135 + 135
18 Let the required speed by x km/hr.
x+x
270 Then, 2 x 64 xx = 56
or, x = x m/s 64+x
2 18
100 I @ Reasoning & Aptitude fflADE ERSW
= (1+ l)kmi.e.] kms. upstream at 2 kmph, his speed in still water is:
(a) 4 km/hr (b) 2 km/hr
(c) 3 km/hr (d) 2.5 km/hr
7 2 minutes. The speed of the man in still water is: Speed upstream = .2.i km/hr,
2 3
(a) 2 km/hr (b) 3 km/hr Speed downstream = 14 km/hr
(c) 4 km/hr (d) 5 km/hr
Speed of the current
10. A man can row 5 kmph in still water. If the river is
running at 1 kmph, it takes him 75 minutes to row to
a place and back. How far is the place?
(a) 3 km (b) 2.5 km
14 2
(c) 4 km (d) 5 km = -krn/hr == 4-km/hr
3 3
5. Ans. (b)
□□□□
Speed downstream= (iO + 6) km/hr
== 16 km/hr.
Time taken to cover 80 km downstream
Solutions
1. Ans. (a)
1
=( ~~) hrs = 5 hrs.
Speed in still water=
2 (6 +2) kmph 6. Ans. (b)
= 4 kmph. Let the distance between A and B be x krn
2. Ans. (a) Speed downstream = 6 kmph,
Speed upstream = 2 kmph.
1
Rate upstream == ( ; ) kmph = 5 kmph. . . Distance A B = 6 km.
7. Ans.(d)
1 Speed upstream= 7.5 kmph,
Rate downstream = ( ~ ) kmph == 7 kmph.
Speed downstream = 10.5 kmph.
Speed of stream =
1
(7-5) kmph
.
. . Total time taken = (105_ + 105)
_ hrs
2 75 10 5
= 1 kmph. = 24 hours
8. Ans. (a)
3. Ans. (c)
Suppose he moves 4 km downstream in x hours.
Rate upstream = ( ; x 60) kmph = 10 kmph Th~n, speed downstream = (;) km/hr, speed
2
Speed of stream= 3 kmph.
Let speed in still water be x km/hr upstream= (;-)km/hr.
102 I >\II Reasoning & Aptitude fflRDE ERS~
~
(c) 5 sec (d) 7.5 sec
Speed downstream = (~ x x 60) kmph
4 15
6. A train 150 m long moving at a speed of
= 6 kmph 25 metres per second overtakes a man moving at
1 5 metres/sec in opposite direction.The train will pass
Speed in still water=
2 (6 + 4) kmph the man in
= 5 kmph (a) 5 sec (b) 6 sec
~
rate of 2 kmph and 4 kmph and passes them
Required time ( 275 x ) sec.
completely in 9 and iO seconds. respectively. The 1 0
length of the train is
= 7.5 seconds.
(a) 72 m (b) 54 m
(c) 50 m (d) 45 m 2. Ans. (d)
13. Two stations A and Bare 110 km apart on a straight Speed ( 72 x ~) m / sec = 20m/sec.
line. One train starts from A at 7 a.m. and travels 1
towards B at 20 kmph. Another train starts from B Time = 60 sec.
at 8 a.m. and travels towards A at a speed of
700 + X
25 kmph. At what time will they meet? . = 60 <=:> 700 + X = 1200
20
(a) 9 a.m. (b) 1O a.m.
⇒ X = 500 m
(c) 11 a.m. (d) 10.30 a.m
3. Ans. (d)
14. Two train are running in opposite directions towards
Total distance covered = (300 +900)m
each other with speeds of 54 kmph and 48 kmph
= 1200 m.
respectively. If the length of the train is 250 m and
Time taken= 1 min. 12 sec= 72 sec,
they cross each other in 18 seconds, the length of
the other trains is:
(a) 145 m (b) 230m
Speed=
(
n
1200)
m/sec.
6. Ans. (a)
220
Speed of train relative to man = 6 or 250 + 5x = 660
250+5x
= ( 25 + 5) m/sec ( 18 )
= 30 m/sec
Time taken to pass the man or x = 82 kmph
5 11. Ans. (a)
= (~;)sec = 5 sec.
Let the speed of train C be x kmph.
Speed of B relative to C = ( 120 - x)
7. Ans. (d)
Relative speed= (45-40)kmph = Gkmph. = [(120-x)x ~]rn/sec
1
m/sec = l Jm/sec
5\ 1
25\
= ( 5x
18
J 18 = ( 60~; 5x) m/sec
Total distance covered = Sum of length of trains
Distance covered= (iOO + 200)m = 300 m.
= 350m
300
= 120 ⇒ 5400
:. . taken= ( 350 x 18) sec.= 252 sec.
Time
25
(60~; 5x)
8. Ans. (c) = 120(600-Sx) ⇒ x = 111.
Relative speed = (30+42) kmph = 72 kmph
12. Ans. (c)
== ( 72x
1~ }m/sec = 20 m/sec
2 kmph = ( 2 x ~) m/sec
Distance c_overed in crossing each other
1
~---~=-.-12u±-114+n1_;;;__24o_ - ~ - - - - - - - - - - ~- - - - - - ~ , - -~ - - - - - - ~ . . - - - - -
5 0
240) = - m/sec and 4 kmph == - m/sec
Required time = ( sec == 12 sec. 9 9
20
Let the length of the train be x metres and its speed
9. Ans. (a) by y m/sec.
Speed of the train relative to man
= 66 kmph.
Let the speed of the train be x kmph. 9y- 5 == x and 10 (9y-10) = 9x
Then, relative speed= (x + 6) kmph. 9y - x = 5 and 90 y - 9x = i 00
x + 6 == 66 or x = 60 kmph.
On solving we get x = 50.
10. Ans. (d) Length of the train is 50 m.
Let the speed of the second train be x kmph
'13. Ans. (b)
Relative speed = (x + 50) kmph
Suppose they meet x hours after 7 a.m.
Distance covered by A in x hours
= [(x+SO)x ~] m/sec. = 20x km
1
Distance covered by B in (x - i) hours
=(
\
25~;·~ 5x) m/sec.
/
= 25 (x-1)km
:o 20x + 25 (x -1) = 110
or 45x=135orx=3.
Distance covered= ('108 + i 12) = 220 rn.
So, they meet at 10 a.m.
® Time, Speed & Distance I 105
or x = 200. 1500)
Speed = ( - - m/sec
120
Length of first train = 200m.
Let the length of platform be y metres.
Speed of the first train = ( 25 x 18) kmph = 45 kmph.
2 5
( 5) 18
25
= 48 x - m/sec = -m/sec.
3 llllilllllllllllil
Section
LAWS OF INDICES Example:
(1) am x an== arn+n (i) x =y or
Example: (ii) X =0, y =0
➔ 2 4 xi 3 =i 7
-m 1
➔ 3sx37=315 (8) a =-
am
am m-n Example:
(2) - = a
an
(i) 4-2 = _i2 =_2_
Example: 4 16
(5) (~r ~~
Example:
=
(3)
(i)
(ii)
n§"_ = r,fa
~2x5=~x%
'R/7x 13 = lft X lffj
Vb %
(i) (]r=!:=1~ Example:
(i) J2 = lf2.
3 3
5) 5 _- 125 ~5 Vs
ef{-
(ii) (8 83 - 512 Vi
(6)a == i O (ii) 3 = if0;
Here a may be any number
Example: (4) ( efaf = ~
(i) 5° = 1 Example:
(ii) 1000° = 1
(7) ax= aY (i) ( v5-)5 = ½-5 = m
If and only if Condition
(ii) (f/4)3 = ifi3 = o/64
i 08 I ® Reasoning & Aptitude fflADE EAS~
1 20
(5) ~~ = mnifa = a mnp (4A) = 45 = 1024
Example:
20
(i) ~f~
i
= 1Q2x3x4 =
1
1Q24
(5)~) . = 5 = 625 4
1 1 Hence, f4 >Pefs
(ii) ~o/25 = 253x5 = 2515
5. Simplify, ✓8 + ✓2
✓8- ✓2
□□□□
✓8+ ✓2 ✓ 8+ ✓ 2
Sol.: ✓8_✓ 2 x ✓8 + ✓2
D Solved Examples (Multiply numerator & denominator by conjugate of
denominator)
Sol ..
.(R)-%
343
= ( z5 )-¾ = (3-)sxi
l3s/ 3, 6. Simplify ~
~+~
1 2
\/a +1-va -i
Sol.:
1)12
43
1
= 3x - 5 = - X - 3 [ = 4 4 = 256 I
j
I
1
1 1 \12
I
= 4x = 2 ⇒ x=-
ls j = =,
(
4 3
l
2 5 25
-2 1 1 ' '
4 == == Is written
42 16
1
= 22x-1 ==
=: 2 -3x+9 i == a
[ ·: am -m] as log
416 == -2
103 == 1000 ⇒ IOQ10 1000 = 3
= 5x == I 0, X =2 35 = 243 ⇒ 1093 243 = 5
9. If 2x == 3Y = 6-z then (.:!_+.:!_+.:!_)is equal to? Properties of Logarithms
X y Z
(a) lo9a (m x n) =lo9am + lo9an
Example 1.
So, 2 = k/2, 3 == kh, 6 = k - Yz 10910(15) == 109103+ 10910 5
We know that 2 x 3 = 6 this gives
1/ -x 1 1 1 1/
(b) log 3 ( ~ )=log 3 (m)- log 3 (n)
klxxk/Y=k z=-+-=--
x y z Example 2.
1 1 1
==-+-+-=0
X Y Z log 2 ( %) == log2 6 - log2 5
10. Arrange j2, 3/4 and~ in ascending order
(c) log 3 (mn)=nlog 3 m
Sol.: ✓2 = 2½
1/ 1' Example 3.
V4 == 413 ' ef6 = 5/4
Now raise the power of given surds by LCM of 2, 1095 625 = log5 54 == 41095 5
3, 4 that is 12 1
(d) log 3 n(m) = -log3 (m)
n
(212) 12 = 26 = 64 Example 4.
12
(4l3) = 44 = 256
12
(5¼) == 63 = 216
(e) log b = lognb
Ans: ✓2, ~. </4- a logna
l
1
log 3 8 = - -
2 3 = 8 is similar as log 28 == 3 log83
11 o I ® Reasoning & Aptitude fflRDE ERS!::ll
a=i6½
a= 4
Remark:
7. log3 ✓3 = 2, find the value of a
(1) When base is not mentioned it is taken as 10. 6
(2) Logarithms to the base 10 is known as common • r;:; 1
Sol.. loga _,3 = -
logarithms. 6
(3) logaa = 1
Example 7. = loga 3h = 2
6
log 55 = i
log 22"" 1
(i) loga 1 = 0
8. If log27 x + log 9 x + log 3 x = ii, find the value of x
[a may be any natural number.]
(j) alo9ax = X
Sol.: Iog 27 x + log 9 x + log 3 x = 11
1 1
50Q; 3 = 3 =
3 1093 x +
2
log3 x + log 3 x = 11
3'093 7 = 7 11
= 6IOQ3X=11
□□□□ 1
= -IOQ3 X = 1
6
= IOQ3 X = 6
Solved Examples = X = 36 = 729
9. Solve log3 n- log 3 4 = 2
1 . Find the logarithm of 32 to the base 2
.:____ _ ___"_.:__:___::_::~~"'-==~_:::.:___c:c=__=--''-'-""---==--""'----------1s~o1.: n=2
IOQ3 n - IOQ3 4 = log3..::.
5
Sol.: log2 32 == log2 ( 2 ) 4
n
= 5 IOQ2 2 = 5 X 1 = 5 log3 - = 2
4
2. Find the value of 210925 or ~ = 32 n = 3 2 x 4 = 36
4 '
Sol.: aloga X =X
So 2092 5 = 5 x2 y2 z2
10. Find the value of log-+ log-+ log-
yz zx xy
3. Find the value of 32 + 1% 5
6. If log15a::: 2'
2
Find a Sol.: logJ ~'.~I== log7 -~f"
. \_ J40}
- . '
bOL: IOQ15 a= 1 = 1097 7-3 = -31097 7 = -3
2
® Surds, Indices & logarithms I 111
t: r•-b) t: rb-o)
i 3. If log-Js x == 3 find the value of x
1%3 7. ( ~ J(b+c-a) =?
1
Sol.: log-Js x == ; , x = ( 2%)
(a) xabc (b) xa+b+c
X == 2¾ x l% = 25: : 32 (c) xab+bc+ca (d) 1
gn x3 5 x(27)3
4. If - - - - - = 27, then n equals: Solutions
3x(s1)4
i. Ans. (c)
(a) O (b) 2
(c) 3 (d) 4
(-ls/3 = (sv2t3 = J}xi) = 81/6
5. If ( -13}5 x 92 = 3a x 3,,/3, then a equals:
(a) 2 (b) 3
(c) 4 (d) 5
2. Ans. (c)
6. If x, y, z are real numbers, then the value of:
213 413
✓x- 1 y)y- 1 z.✓z-1x is
(2~6r +(2~ r
112 I * Reasoning & Aptitude fflRDE ERS!:11
=6
(3x-32) + 3(3x-34) = 62 + 34 = -36 = -4 . 7. Ans. (d)
81 9 Given Exp.
3. Ans. (d) _ (b-c)(b+c-a) (c-a)(c+a-b) (a-b)(a+b-c)
-x .x .x
Jin= 64 => 2n 12 =64 = 2 6
.
:. 2n-= 6 or n = i 2.
:. x +2 = O or x = - 2.
9. Ans. (b)
or 22x-1 = _1_
gx-3
or 32n+5+9 = 33 X 31 X 316
2n+ 14 = 20
:. 2x-1=9-3x or 5x-10 orx=2.
or 2n =6 or n = 3.
11.Ans.(d)
Let ax = by = c2 ::: k,
Then a= k 1/x, b=k 1/Y and C=k 1/z.
or a=(~-f)=s.
12. Ans. (d)
6. Ans. (d)
aoc=
' 4
1.
□□□□
fflADE EASY ® Surds, Indices & Logarithms I 113
Surds, Indices & Logarithms 10. If log 2 [1ogilogzr)] = 1, then xis equal to:
(a) 512 (b) 128
(c) 12 (d) O
Practice Exercise: II
11. If log 10 2=0.3010, then log 2 10 is:
(a) .3322 (b) 3.2320
1. The value of log 343 7. is: (c) 3.3222 (d) 5
5. If log 4 x + log 2 x
(a) 2
(c) 8
= 6, then x is equal to
(b) 4
(d) 16
16. The value of [ log
(p/q)
1
x + log
(q/r)
x + log
1
(r/p)
x is:
1 l
(a) 3 (b) 2
6. If log 2 = 0.30103, then the number of digits in
(c) 1 (d) O
5 20 is:
(a) 14 (b) 16 17. If log 2 = 0.3010 and log 3 = 0.4771 then the value
(c) 18 (d) 25 of log 4.5 is:
(a) 0.6532 (b) 0.7727
7. The value of logilog 5 625) is:
(c) 0.3266 (d) None of these
(a) 2 (b) 5
(c) 10 (d) 15
□□□□
8. (lo9:iaxlo9cbxlog8 c) is equal to:
(a) 0 (b) 1
(c) abc (d) a+b+c
Solutions
1. Ans. (a)
Let log 343 7 = m.
equal to: m
Then, (343) = 7 ⇒ ( 73'm
) =7
(a) 1 (b) 2
3 ⇒ 73m =7
(c) 3 (d) -
2
ii 4 I "" Reasoning & Aptitude fflRDE EASY
⇒ 3m= 1 ⇒
i
m=-. or 2Y = 4 = 22. So, y = 2.
3
:. log2 (logs 625) = 2.
1
IOQ343 7 = -. 8. Ans. (b)
3
(3x3-)
3 = logabe a+ logabe b + logabe C
=2 = 2 2 = 4.
= log81:ic(abc) = 1.
4. Ans. (b)
10. Ans. (a)
IOQJ2 x = 0.8 ⇒ x = (32)°" 8
log 2[Iog3 (log 2 x)] = 1
⇒ lo93 (1og2 x) = 2
5. Ans. (d)
⇒ log 2 x = 32 =9
1094 X + 1092 X = 6 ⇒ -IGg-.x.-.:;..-IG@; = 6 ⇒ x=i'=512
log4 1092
11.Ans.(c)
logx lo9x
2
log +
2 1092
= 6 ⇒ 31ogx = 121og2 1 1
log210 = 10910 2 = 0.301 O = 3.3222
or logx = 4Iog2 ⇒ logx
12. Ans. (c)
4
= log2 or x=~ = 16.
log10 5 = 10910 ( ~) = log101O-log10 2
6. Ans. (a)
20 1
log5 = 2OIog5 = 20 x [1og( ;)] = 1-109102 = (1-0.3010) = 0.6990.
13. Ans. (d}
= 20x [lo910-log2) = 20x [1-0.3010) log 927 + log 8 32
= 20 x 0.6990 = 13,;dOO log 32 33 + log2:i 2 5
Characteristic == 13.
3 5
:. Number of digits in 520 is 14.
-IOQ3 3 + -IOQ2 2
2 3
3 5 19
7. Ans. (a) So...:..+-=-:
'2 3 6
Let log5 625 = x. Then, 5x
-- F;')c::; -
.._,~.._,, -
t::;4 nr
-.,.,,VI
v
.I\,
-- A
-,-
14. Ans. (c)
log 5 (.r2 + x)-1og 5 ( x + 1) = 2
@ Surds, Indices & Logarithms I 115
log, [t\1)l 2 =
= logx ( ~) + logx ( ~) + logx ( &)
logx(Ex.9.. x_r:_) = log1 = 0.
or log5x === 2 or x::::52 = 25. q r p
= 6 [ 2x 1 i +(6-1J
2 4
'(-3)1
2
n
-§_[_iJ-~]
- 2 2 2
= 2 [a+T,1]
=-6
• If in an AP mth term is 'n' & nth term is 'm' then
So Sn= %(a+ Tn] = %[2a+(n-i)d] (a) The common difference of such AP will be '-1'
(b) The (m + n)lh term of the AP will be 'zero'
Properties of Arithmetic Progression (c) The first term of the AP will be '(m + n -1 )'
• Application of numbers which are in AP
Example 2.
(a) If three numbers are in AP then they can be
Talk about any AP of common difference
assumed as a - d, a, a+ d
- i.
(b) If four numbers have to be assumed
7, 6, 5, 4, 3, 2, i, 0
then they can be a - 3d, a - d, a + d, a+ 3d
Its 2nd term is 6
(c) If five numbers have to be assumed then they
and 5th term is 2
will be (a - 2d), (a - d), a, (a + d),
s
then th term will be 'O'
(a+ 2d) and the first term is 6 + 2 - i = '7'
If in an AP sum of n terms is 'rn' and sum of 'm' @ If same number is added or subtracted from each
terms is 'n' then sum of (m + n) terms will be "-(m terrn of the AP. The new series will again be an AP.
+ n)"
Example 3.
Example 1.
If 2, 6, 10, 14, i8, .... is an AP and a number 5 is 1·
For an AP sum of 2 terms is '4' and sum of '4' terms either added or subtracted, then the new series
is '2'. Find out the sum of 6 term of that 'AP. "
formed are
® Progression I 117
Basics of HP
□□□□
th
The n term of a harmonic series can be calculated
only by finding out the nth term of the arithmetic series
obtained by inversing the harmonic series and then
again reversing the term.
Solved Examples
Example.
Find out the nth term for the series
1. If in an AP 1Qth term is 86 and 86 1h term is 1O, then
1 1 1 what will be the 10Qth term?
5 ' 11' 17 ' 23 ' ..... (a) 1 (b) 3
The terms are reversed then the AP obtained is (c) 4 (d) -4
5, 11, 17, 23, .... Ans. {d)
1 a+ 9d = 86
So nth term of HP is n_ ⇒
6 1 -76d = 76 or d = -1
a+ 85d = 10
Basic Means
substituting the values
For the given terms there are three types of means
-----~--Ar~tflmet-fe-me ~g~ a=5.~d~~~------~
1h
"' geometric mean So i 00 term T100 = 95 + ( 100 - 1)(-1)
e Harmonic mean ⇒ TIOO = --4
hence option (d)
Arithmetic Mean
Alternate: For this series from the property of AP.
For n terms a 1 , a 2, a 3 , a4 , ...... an the AM will be
We can say that 96 th tArm is O and
a1 + a2 + 83 + ...... an d = -1 so 100th term= -4.
n
2. Given that a function S ={2, 3, 4, ..... 2n + 1} and X
Geometric Mean is average of all the odd integers of S while Y is
Geometric mean is the term which when included, average of all the even integers of S, find out what
is (X - Y)
makes all the term in GP. Let us say there are two terms
(a) i (b) n 1
a and b then GM will be in such a way that a, GM, be will
be in GP (c) n (d) None of these
Ans. (a)
So GM= F,Sj
3 + 5 f 7 + .... 2n + 1
or it can be written as X=
No. of terms
" GM(fortwoterms=(a.b) 112
If there are n terms a1 , a2 , a3, a 4 , ..... an then 3+ (2n+ 1)
== =n+2
>I/) GM= (a1 .a2,a3, ......... an)iln 2
Harmonic Mean y == 2+4+6+ .... 2n
Harmonic mean is the term when included between No. of terms
the terms make all the terms in harmonic progression. 2+2n
== --=n+,
4
The harmonic mean for the two terms a and b is
2
® Progression I 119
So X - Y = 1 (a) 5 (b) 6
Hence option (a) (c) 4 (d) 3
Alternate: Difference of 2 consecutive terms i.e.
3. Find the 15th termofthesequence20, 15, 10, ....... .
odd term and even term == 1 so difference of avg.
(a) -45 (b) -55
of odd and avg. of even = 1
(c) -50 (d) O
i.e. X - y = 1.
4. A number 15 is divided into three parts which are in
3. The number of terms between 30 to 530, which are
AP and the sum of their squares is 83. Find the
divisible by 11 are
smallest number.
(a) 32 (b) 35
(a) 5 (b) 3
(c) 36 (d) 46
(c) 6 (d) 8
Ans. (d)
The first term divisible by 11 = 33 5. The sum of the first 16 terms of an AP whose first term
The last term divisible by 11 = 528 and third are 5 and 15 respectively is
528-33 (a) 600 (b) 765
No. of terms = - - - + 1 = 46 (c) 640 (d) 680
11
Hence option (d). 6. A boy agrees to work at the rate of one rupee on the
4. Split 74 into four parts in such a manner that all first day, two rupees on the second day, four rupees
parts are in AP and the product of first part and last on the third day and so on. How much will the boy
part is 2 less than the product of second part & get if he starts working on the I st of February and
third part. finishes on the 20 th of February?
1. Ans. (b)
ar6 = 8 ar3
3
r = 8, r=2
In series 20, 25, 30 ............... 730.
Now ar 4 = 48
a= 20, d = 5
a X 24 = 48, a=3
n th term is 130
⇒ 20 + (n - 1) 5 = 130 8. Ans. (a)
⇒ 5n = 115, n = 23 a, ar, ar 2
nowa+ar+ar2 =14 ... (i)
2 2 2 2 4
_:_______2_._A_n_s_.~(c_,_)_ _ _ _ _ _ _ _ _ _ _ _ _ _ _ _ _ _ _ _a_ls_o_a__+_a__r_+----'-'a--'-r__=-=8-'-4------~·••iiLL--~
Here a+ 7d = 39 ... (i) only suitable combination is 2, 4, 8
a+ 11d = 59 ... (ii) So largest term is 8
So, 4d = 20
9. Ans. (a)
d=5
Between 300 and 500 the first term divisible by 7 is
a+ (7 X 5) = 39
301 and lasl lerm divisible by 7 is 49/
So, a= 4
So, here a= 301, d = 7
3. Ans. (c) L =a+ (n - 1) d
AP 20, 15, 10 497 = 301 + (n-1) X 7
Here a= 20 ⇒ 7n = 203
d=-5 ⇒ n=29
th
15 term will be a + 14d which is equal to
10. Ans. (c)
20 + (-5) X 14 = -50
It is given that
4. Ans. (b) ar3 = 1/3 ... (i)
Let numbers are
a - d, a and a + d
ar9 = 243
From (i) and (ii) we get
... (ii) I
now it is given that r6 = 729
r=3 .. (iii)_1:
sum = 15 & sum of there square is 83 i.e.
3a=15
2
(a - d) + a + (a + d) 2 2
= 83
... (i)
... (ii)
From (i) and (iii) we get
~ V '{3 -
1
-, ~- -
II
3· a-
3a 2 + 2d 2 = 83
~"V -
7b + 2d 2 = 83
81
j
second term will be
' 1
27 Ii
E EIRS~ ® Progression I 121
2
-
4 Solutions
(a) (b)
5 5
1. Ans. (a)
3 1
(c) (d) -
5 5 ~. k,.!?. k are in A.P.
3 8
11
10. Evaluate ~::{2+3j)
j=1
k- ~ = _!?_ k - k ⇒ Sk ~ 2k = ~2
3 8 8 3
11 11
⇒ -11 k = -2 ⇒ k = ~.
(a) 22+~(3 -1) (b) 11+~(3 -1) 8 3 33
2 2
2. Ans. (c)
10
(c) 22+%(3 -1) (d) None of these Let a be the first term and d, the common difference
of an A.P.
a7 =a+ 6d
11. The common ratio of a G.P. is -~ and the sum to
5 a11 =a+i0d 7a 7 =11a 11
7(a+6d)=11 (a+10d)
infinity is SO. Find the first term. ⇒ 7a + 42d = 11 a + 11 Od
9 ⇒ 4a + 68d = 0
(a) 14 (b) 16 ⇒ a+17d=0
(c) 12 (d) 10 ⇒ a1s =0
II
15. If a be the first term of a G.P., I the nth term and P = 32m
the product of first n terms then P =
Distance covered during the 3rd second
(a) (alt12 (b) (a - l)n12 = 28 m.
(c) (a + l)n12 (d) None of these The distance covered form an A.P.
= 36 + 32 + 28 ..... ih which
I
□□□□ a ::c: 36, d ± -4
Distance covered in 8 th second
® Progression I 123
125
n ⇒ -.-::::-
3
152=125r3 +125
525 = -[3 + 39] ⇒ 1050 = n(42) r +1 152'
2
3 3 27
1050 ⇒ 125r = 27
'
⇒ r = -125
or, n=--:;:::25
42
putting n = 25 in (i), we get
7. Ans. (b)
Let a be the first term and r the common ratio
a5 = 2 ⇒ ar 4 = 2 ... (i)
Now, product of first 9 terms 80 9
⇒ 8= X-=16
= a x ar x ar 2 x ...... ar 8 9 5
= 8 s r1+2+ ..... +s = 8 s r36
Hence, the first term is i 6.
= (ar 4 )s = 2s = 512.
12. Ans. (c)
8. Ans. (c)
Let a be the first term and r be the common ratio of
G.P.
We have a 3 = (a 1) 2 ⇒ ar 2 = a 2
⇒ r2 = a ... (i)
Also, a 2 = 8
⇒ ar = 8 ... (ii)
Multiplying (i) and (ii) we get
ar3 = 8 x a
I
i 24 I @ Reasoning & Aptitude fflRDE EASY
~iii
3 i6 5 i6 4 1
=-X---X-=---
4 i5 i6 i5 5 3
Practice Exercise: Ill
12-5 7
=--=--
15 15 1. If the sum of the 6 th and the 15 th elements of an
13. Ans.( b) arithmetic progression is equal to the sum of the
th th
The given sequence is 6, 4, 3, ...... which is H .. P. 7 , i 0 and 12th elements of the same progression,
The sequence of reciprocals of its terms is then which element of the series should necessary
be equal to zero?
1 1 1
6,4,3° ....... which is anA,.P (a) iQlh
st
(b) 8th
(c) 1 (u) hJone of these
i 1 i 1
8. If b -; = a-b, then a, b, C form a/an
DODD 15
= 2(200- 70] = 975.
126 I @ Reasoning & Aptitude fflRDE EASY
5. Ans.(b)
2ac 1 1 1 .
⇒ b =- - ⇒ -,-,- are in AP.
Sn = ~ [ a + (n - i) d J a+c ab c
2 ⇒ a, b, c are in H.P.
⇒ n[a+2nd-d] = 3[f(a+nd-d)]
Sum of the cubes of first n natural numbers
⇒ d=~
i+n
2
3n n(n + 1)
:. Mean = -----'--'--
S -[a+3nd-d] 4
~ = ....,2=------
sn ~[a+nd-d] 11. Ans. (a)
2
Sn= Sum of n terms of an AP.
3[a+ 3na - ~ ]
= 1+n i+n = 6. =%[2a+(n-i)d]
na a
a+---
i+n i+n where a= first term,
d = common difference
6. Ans. (b)
Let the three numbers be a, ar, ar2 , where r is +~5+7 + ... + n terms =
7
common ratio. 5 + 8 + 11 + ... + 10 terms
2
a+ar+ar = 28 and a3r3 = 512 f[2x3+(n--i)x2]
⇒ =7
ar = 8 ⇒ a + ar 2 = 20 iO
[2 X 5 +(10-i)x 3]
⇒ 8r
2
- 20r + 8 = 0
2
1 n(2n+ 4)
⇒ r = 2, r=- ⇒ =7
2 370
2
if r = 2, a = 4. Therefore, the three numbers are ⇒ 2n + 4n - 2590 = 0
4, 8, 16 ⇒
2
n + 2n - i 295 = 0
2
7. Ans. (b) ⇒ n +37n-35n-1295=0
The sum of the squares of the first n natural numbers ⇒ n(n+37)-35(n+37) = 0
is ⇒ (n-35)(n+ 37) = 0
n(n+ 1)(2n+ 1) ⇒ n == 35.
6
Put n = 15, we have, 12 + 22 + 32 + 42+ ... + 152 12. Ans. (c)
= 15(15+:(30+ i) = 1240.
Sum of the first n natural numbers = _n(_n_+_i)
2
8. Ans. (c) Sum of the squares of the first n natural numbers
2 1 1 n(n+ 1)(2n+ i)
-=-+- =
b a c 6
rDADE EASY ® Progression I 127
1))
n(n + 1) = _!_(n(n + 1)(2n +
2 5 6 l!~i Practice Exercise: IV
⇒ 2n + 1= 15 ⇒ n = 7.
1. lffor an AP m th , nth , pth terms are a, b, c respectively
13. Ans. (c) then c(m - 1) + a(n - p) + b(p - m) will have the
values as
1
(a) O (b) 2
3. Ans. (d)
12. Three numbers P, q, rare in GP if P & q are increased
Common difference = (b - a)
by 4 and r is decreased by 4 then they forms and 2a = a + (n - 1)b -·a
AP. If sum of the numbers is 56 then highest among
a
P, q, r will be ⇒ --=n-1
(a) 24 (b) 18
(c) 36 (d) 32 b
⇒ n= - -
b-a
13. Find the sum to infinite term of the series
1( b ) 3ab
1 1 1 1 ? b _ a [ 2a + a] = sum = 2 (b _ a)
- + - + - + - + ....
56 72 90 110
4. Ans. (d)
1 1
(a) 8 (b) - a2
7 s-- - -
1-a2
1 1 ⇒ s(i - a2) = a 2
(c) 6 (d) 28
⇒ s - sa 2 = a2
s
14. Let {1, 2, 3, .... 541 }. Find out how many APs can be ⇒
2 -
a = 1+s
formed by taking the element of S starting from 1
and ending at 541. Which have atleast 3 elements S )1/2
(a) 12 (b) 13
⇒ (
a= t+S
(c) 23 (d) 24
5. Ans. (b)
15. Find the sum to first 10 terms of the series Let speed be x and the car covers distance in AP.
12 + 35 + 7f) + 1 i 7 + ... Then the distance covered in
(a) 2595 (b) 2575 3 rd sec. = x + 2d = 25
(c) 2945 x + 6d = 65 so d = 1O and x = 5.
So the distance covered in
□□□□ 15th sec. = 5 + 14 x 1O = 145 m.
® Progression I 129
6. Ans. (c)
T11 = s 11 - s 11 _ 1 ={n 2 + 2n} - {(n - 1 )2 + 2(n - 1)} = 100(1+ i+ i+ . .)-so
= (2n - 1) + 1 = 2n
T11 _ 1 = 2(n - 1) = 2n 2
Common diff. = 2n - (2n - 2) =2 = 100[-;J-50 = ·10om
i--
7. Ans. (d) 3
Applying A.M. 2 G.M 12. Ans. (d)
&
1 1 1 1]
[ a2 +"ti+ c2 + d2
4 2
(-1a2 ._1b2 ._1c2 .J_)1/4
d2
p + q + r = 56
by solving the eq. we get
... (iii)
p = 8, q = 16, r = 32
So _1_(a2 + b2 + c2 + d2).(_1_ + _1_ + _1_ + _1_) >1
16 a2 b2 c2 d2 - 13. Ans. (d)
or the min. possible value= 16 1 1 1
The series=-+-+--+ ....
7.8 8.9 9.10
8. Ans. (b)
In the A.P. all the multiples of 3 will be common with
1 1 1 1 1 1
= 7-8+8-9+9-iO+ ...
G.P. and since the common diff. is '1' every term
So the required
will be there in consecutive 3 terms.
1 i
100 sum== --0 =-
7 7
So net common terms =
3 = 33 .
14. Ans. (c)
9. Ans. (d) In this case 54i = i + (n-1)d
So the possible value of n will be the number of
T
2
= T9 ' ⇒ T2 =21 factors of 540.
Tg'
So no. of factors of 540 = 3 x 4 x 2 = 24
s(2.2_1) __ (2.2-1) T2
So - in this we can not include 'I' as n - 1 = 1.
s'( 29 _ 1) - (2.9-1)T~
Son= 2.
s 3 1 3 Which is not acceptable. So net possibilities
⇒ _3 = - X - = -
s;? 17 1 17
= 24-1 = 23
1 = (n)(n+1)(2n+1) + (n)(n+1) +n
This will be a infinite G.P. with common ratio as 6 5
3 6 2
For n = 10
So net distance covered
5
S 10 = (10x11x21)+ x10x11+10
=
50 50 50 50 50 50
5 0 + - + - + - + - + - + - + ... 2
3 3 9 9 27 27 s10 = 2595
j, j,
up down l!illlll!IIIIIUII
Being an area as a measure of students reasoning ability, For each male among couple has a choice of 39
permutations & combinations becomes one of the shake hands, because he has not do l12u1clsl1ake
important chapter from exams point of view. will his spouse & for each of single boy there is
choice of 40 handshakes.
Fundamental Principle of Counting Hence total no. of handshakes.
== 20 X 39 + 20 >< 40 = 20 X 79 :::: 7580
Multiplication Rule
If a task A can be done in 'm' ways & a second Permutations
task B can be done in 'n' ways, the number of ways Number of ways of arranging particular things in
of doing both the task one after another is 'm >< n' certain order is known as permutations. For example
ways. for three things (a, b, c) can be arranged in multiple
ways in a straight line, total number different
Additive Rule
arrangements is called permutations.
If task 'A' can be done in 'm' ways & another task
'B' can be done in 'n' ways then total no. of ways of a,b, b,
C a, C C, a, b]
doing either of task. [ a, c, b b, c, a c, b, a Total six ways
= (m + n) ways
These two rules are basic building block to np = . lD
parmutatlons.&...combinations..p.r:incLpJ.es.------~--r-1.cn~. .-:-~r________________
Distinct permutations of 'r' things taken out of n
Some Basic Examples things taken at timings= npr·
1. lnabuildingthereare6floors.lnhowmanyways5 The terms permutation & arrangement are
persons can get down at different floors such that synonymous and can be used inter•changeably.
exactly one person is allowed at any floor. ( order in
which person are getting down is not important) Combinations
Combinations is basically selection of a group of
Soiution. certain things out of given set. In combination order
For 1st person there are 6 choices of floor. Now for of things is not important.
nd
2 person only 5 choices are left (because one of Combinations of 'r' things out of 'n' distinct things is
the floors is occupied by 1st person)
Progressing with same logic nc : : np, = ID
Total no. of ways (BecauS? all five to get down, r lr In- r x ~
hence multiplicative rule)
6 x 5 x 4 x 3 x 2 "" 7?0 ways Special Notes:
At times, question may not explicitly state whether
2. In a party there are 20 cuuples, 20 boys (single) &
20 girls (single) are invited. If handshake between we have find out permutations or combination but l.':•·-,.••._
Couples
I\IVl'-AIVV
20
IL::1 lc.c Females
20
name 123, 132 order is important hence permutation I
W illbe used. m
Singles 20 20 i!
® Permutation & Combination I 131
7 7 as = 9 C 2 ways.
same pattern will be applicable 10 th place, 100th (In a grid of 8 x 8, there will 9 horizontal lines and 9
place, 1000 th place, 10000th place digit vertical lines)
Hence sum of all numbers unit place digits will be Hence total number of rectangles are
1260
= -7-( 1+ 2 + 3 + 4 + 5 + 6 + 7]
=
9
C2 X
9
C2 = ( g; g; B)
8
) X( = 36 X 36
Example 10.
How many natural number solutions are possible Solved Examples
for equation below?
X + y + Z + W:::: 20
(a) 23C3 (b\I 22c
2 1. In the above question how many numbers can be
(c) 19C3 (d) None of these formed when repetition is allowed?
Ans. (c) Solution: We have to form two digit number i.e., we
In this case again partion rule is applicable, such have to fill two place out of 4 numbers & any number
that each of x, y, z & w should get at least one thing. can be used any number of times this can be done
Hence no. of ways= 20- 1C 4 _ 1 = 19C 3 in 4 x 4 = 16 ways.
Hence option (c) 2. How many words can be formed by using letters of
Example 11. word MADAM?
Solution: In MADAM we have
There are 15 people to be seated around a round
2-M's
table. In how many ways we can arrange them such
that 2 A's j
134 I ® Reasoning & Aptitude mRDE EASY
3. or the different words that can be formed from letters 8. Out of i5 points in a plane, 5 of which are collinear.
of word MOBILE, how many begins with Mand ends How many straight lines can be formed?
with E? Solution: If all 15 points were non collinear then the
Solution: ~DD DD~ answer would have been 15C2 . However, in this case,
since the 5 collinear points have also been counted.
M and E are fixed at the start and end positions,
These would have been counted as 5 C 2 whereas
Hence, we have to arrange B, I, L, 0 among
they should have been counted as 1. We need to
themselves (1e in four places), this can be done in
adjust by reducing the count by ( 5C 2 - 1).
4! ways.
Hence, the answer 15 C 2 - ( 5 C 2 1) = 96
4. Six boys and 4 girls wanted to enjoy a movie. How
many ways of sitting arrangement can be possible 9. How many triangle can be formed by 18 points if no
for them if girls want to sit together? three among them are collinear.
(here X is symbolic notation that all girls are sitting 10. In the above situation how many triangle can be
together) formed if 5 points are collinear.
equation (2) Can be arranged in 7! ways Solution: The triangles will be given by
Now girls can be arranged among themselves i.e., 18C - 5C == 806 ways
3 3
4! ways so total number of sitting arrangement will
be 7! x 4! ways
-C--- ------5.--~R--t-he-aee-ve~qtre-sttonitm:t-rrum oor-orways of - --- --
arrangement if no two girls sit together.
Solution: This Cc:tr1 be done in following manner
- 81-B2-B3-B4-Bs-86-
First of all, boys can be arranged in 6! ways, now [!] Practice Exercise: I
vacant seats between them will be filled by girls.
This can be done in 7P 4 ways so total aliangement 1. How many numbers of 3 digits can be formed with
= 7P4 X 6! the digits i, 2, 3, 4, 5 (repetition of digits not
allowed)?
6. How many 5 digit numbers can be formed by using
(a) 125 (b) i20
0, 1, 2, 3, 4, 5, 6, 7 only once?
(c) 60 (d) i50
d 1 d 2 d 3 d4 d 5
Solution: DD DD D 2. How many numbers between 2000 and 3000 can be
box d 1 cannot be filled with zero so only 7 numbers formed with the digits 0, 1, 2, 3, 4, 5, 6, 7 (repetition
can be filled in first box rest can be done in of digits not allowed)?
(a) 42 (b) 210
7 x 7 x 6 x 5 x 4 = 5880 ways
(c) 336 (d) 440
7. How many different sums can be formed with the 3. In how many ways can a person send invitation
following coins, selecting any number of coins from cards to 6 of his friends if he has four servants to
5 rupees, 1 rupee, 50 paisa, 25 paisa, 10 paisa, distribute the cards?
1 paisa.
Solution: A distinct sum will be formed by selecting (c) 24
(b) 46
(d) None of these
I
either 1 or 2 or 3 or 4 or 5 or all 6 coins. II
fflRIOE ERS~ ® Permutation & Combination I 135
4. In how many ways can 7 Indians, 5 Pakistanis and 14. How many distinct words can be formed out of the
6 Dutch be seated in a row so that all persons of the word PROWLING that start with Rand end with W?
same nationality sit together? (a) 8!/2! (b) 6!2!
(a) 3! (b) 7!5!6! (c) 6! (d) None of these
(c) 3! 7! 5! 6! (d) "18!
15. How many 7-digit numbers are there having the digit
5. How many straight Iin.es can be formed from 8 non- 3 (three times) and the digit O (four times)?
collinear points on the X-Y plane?
(a) 15 (b) 33 x 4 4
(a) 28 (b) 56
(c) 30 (d) None of these
(c) 18 (d) 19860
6. In how many ways can the letters of the word PATNA □□□□
be rearranged?
(a) 60 (b) 120
Solutions
(c) 119 (d) 59
1. Ans. (c)
7. In the above question, how many words would be Three digits can be formed in 5 P3 ways that is
there which would start with the letter P?
(a) 24 (b) 12 51
= 60
(c) 60 (d) 18 (5-3)!
6. Ans. (a)
51 Practice Exercise: II
Th .1s can b e d one .in - ways
2!
51
= 60
2! 1. There are 15 buses running between Delhi &
Mumbai. In how many ways can a man go to mumbai
7. Ans. (b)
and return by a different bus?
In PATNA, if P is fixed so we have only four letter
(a) 280 (b) 310
ATNA in which there are two A's. (c) 240 (d) 210
4!
So -·· = 12 ? . A tP.;:ic:her of a class wants to set one question from
' 2!
each of two exercises in a book. If there are 15 and
8. Ans. (c) 12 questions in the two exercises respectively, tr·1en
11
C2 x 2! Ways which is equal to 110. in how many ways can the two questions be
selected?
9. Ans. (a)
(a) 160 (b) 140
In Allahabad we have 4 A's that is only one vowel
(c) 180 (d) 120
and 4 consonants (B, D, H, L)
So any four combination. 3. A code word is to consist of two distinct English
AB, AD, AH, AL are only 4 possibilities so. alphabets followed by two distinct numbers between
1 and 9. For example, CA23 is a code word. How
10. Ans. (b)
many such code words are there?
In EQUATION there are five Vowels A, E, I, 0 and U
(a) 615800 (b) 46800
and three consonants N,Q, T so the possible
(c) 719500 (d) 410800
arrangements will be 4! x 5! ways
4.
There are 5 letters and 5 directed envelopes. Find
11. Ans. (c) the number of ways in which the letters can be put
~------,0-ta+·oomeeref-stfafgltts-Hfte·~-----------.,.,;1n::.to=the envelopes so that all are not put in directed
25 7
= C2 - C 2 + 1 = 280 envelopes?
12. Ans. (b) (a) 129 (b) 119
Total triangle formed will be equal to (c) 109 (d) 139
25
C3 - 7C3 = 2265
5. How many diffE:m:mt ni1mbers of two digits can be
i3. Ans. (a) formed with the digits 1, 2, 3, 4, 5, 6; no digits being
i5-1C11-1 X 11! repeated?
14
⇒ C10 X ii! = i001 X 11 ! (a) 40 (b) 30
(c) 35 (d) 45
14. Ans. (c)
6. How many three-digit odd numbers can be formed
from the digits i, 2, 3, 4, 5, 6 when
/1 is R & 18 is Wrest boxes can be filled in 6! ways (i) repitition of digits is not allowed
(ii) repitition of digits is allowed?
15. Ans. (a)
(a) (i) 60, (ii) 108 (b) (i) 50, (ii) 98
dl d2 dJ d, <ls d6 d, (c) (i) 70, (ii) 118 (d) (i) 80, (ii) 128
□□□□□□□ 7. How many numbers are there between i 00 and i 000
d 1 is 3 because we have to form 7 digit number now
in which all the digits are distinct?
we have rest six boxes d 2 , d 3 , d 4 , d 5 , d 6 , d 7 to be
(a) 548 (b) 648
filled, this can be done in
61 (c) 748 (d) 756
--·- = i 5 ways
8. If (n + i)!=6 [(n-i)!], find n
21 X 4! .
(a) 6 (b) 4
□□□□ (c) 8 (d) 2
® Permutation & Combination I 137
i
= 720-120 = 600.
⇒ (n+ 1). n. [(n- 1)!] = 6[(n-1)!] 13. Ans. (a)
The required number of three-digit numbers= The
⇒ n2 + n = 6 ⇒ n2 + n - 6 = O
permutations of the '10 objects 0, 1, 2, 3, 4, 5, 6, 7,
⇒ (n-2)(n+3)=0 8, 9 take 3 at a time, with the condition th8t Ois not
in the hundred's place.
Either n - 2 = O or n + 3 == 0
= P(i 0, 3) - P(9, 2)
⇒ n = 2 or n = -3
10! 9!
n being natural number, son :;t-•3 7! 7!
n = 2. i0x9x8x7! 9x8x7!
= 7! 7!
9. Ans. (c)
The word EOUATIOi\J has exactly 8 letters which are =i0x9x8-9x8
all different. = 7 20 - 7 2 = 648
® Permutation & Combination I 139
A's are together while in the rest they are not together. be selected?
q-p = 2520- 720 = moo. 3. A box contains 10 balls out of which 3 are red and
the rest are blue. In how many ways can a random
15. Ans. (b) sample of 6 balls be drawn from the bag so that at
Out of letters in the word 'BHARAT' two letters, that the most 2 red balls are included in the sample and
is, A's are alike. no sample has all the 6 balls of the same colour?
61 (a) 105 (b) 168
.
Number of permutations= == 360 .
12. (c) 189 (d) 120
Number of words in which B and H are never 4. A cricket team of 11 players is to be formed from 20
together. players including 6 bowlers and 3 wicket keepers.
= Total number of words - number of words in which The number of ways in which a team can be formed
B and H are together having exactly 4 bowlers and 2 wicket keepers is
2 x6! 5! = 2.6!5! 6. Seven points lie on a circle. How many chords can
be drawn by joining these points.
= 10.9.8.7.6! = 3 x 7 = 21 _ (a) 22 (b) 21
2.6!5.4.3.2.1
(c) 23 (d) 24
17. Ans. (c) 7. Ten different letters of an alphabet are given. Words
The require number of ways= C(10, 5) with 5 letters are formed from these given letters.
Then the number of words which have at least one
= 10! = 10.9.8.7.6 == 252.
5!5! 5.4.3.2 letter repeated is
(a) 69760 (b) 30240
□□□□ (c) 99748 (d) None of these
140 I III Reasoning & Aptitude fflRDE EASY
using the numerals 0, 1, 2, 3, 4 and 5 without There are 6 bowlers, 3 wicket keepers and
repetition. The total number of ways in whir.h this i 1 batsmen in all. The number of ways in which a
can be done is team of 4 bowlers, 2 wicket keepers and 5 batsmen
(a) 211 (b) 216 can be chosen.
(c) 22i (d) 311
== 6C4 X 3C2 X 11C5
8. Ans. (b)
⇒ n2 - i in + 8n - 88 = O
4! x 2 ways, i.e. 24 x 2 = 48.
⇒ n(n-11)+8(n-11)=0
9. Ans. (b)
⇒ (n- i 1)(n+8) = 0
= 8C x 8 C = 8x7x6 x 8x7x6
3 3 ⇒ n=ii.
3x2 3x2
=56x56=3136. 12. Ans. (b)
All permutations formed with i, 2, 3, 4, 5 (sum= 15)
10. Ans. (a)
will be divided by 3.
7C3 x 6C2 + 7C4 x 6C1 + ?Cs x eco There are 5 ! = i 20 such permutations. Such numbers
= 7
C3 x
6
C2
7
+ C3 X
7
6 + C2 X 1 can also be formed using Oand i, 2, 4, 5. There are
4 x 4! such numbers, i.e. 96. (Factor of 4 for four
=7x6x5x6x5+7x6x5xG+7x6 positions of O and 4! for different permutations of
3x2x1 2x1 3x2x1 2x1 these four numbers)
= 525 + 210 + 21 = 756. :. Total of such numbers = 120 + 96 = 216.
i 1. Ans. (a) 13. Ans. (c)
Let the number of sides be n. 7 6
C2 X C2 = 315.
14. Ans. (c)
SC _SC _5x4_ 10
2 -2x1- ·
⇒ n2 - 3n - 88 =0 3 -
lllllllll!l!il
Introduction 2. Playing card: A pack (or deck) of playing cards
The probability has its origin in the problems dealing has 52 cards, divided into four suits:
with games of chance such as gambling, coin tossing, (1) Spades (2)Clubs
die throwing and playing cards. In all these cases the (3) Hearts (4) Diamonds
outcome of a trial is uncertain. These days probability Each suit has 13 cards
is widely used in business and economics in the field (a) Nine number cards 2, 3, 4, 5, 6, 7, 8, 9, 10
of prediction for future. (b) An Ace, a king, a Queen and a Jack are known
as face cards
Lots of questions based on coins, die, playing cards
Ace-A King-K
and alphabetical arrangements are asked in various
Queen - 0 Jack -J
public sector examination.
Spade and clubs are black faced cards while Hearts
and Diamonds are red- faced cards.
Theory The King, Queens and Jacks are called court cards.
Probability means the chance of occurrence of an event. 3. Unbiased coin - coin having two faces head (H)
In layman terms, we can say that it is likelyhood that and Tail (T}
----~s=o=m=et~h=ingJ.bat is defined a~ aD e\Lents w i J l . . i l l . j [ j f f i L t 1 0 ~ - - - - - - - - - - - -
occu mathematically,
Some Basic Concepts
Probability (P) =Number of favourable events Random experiment
Total Number of events An experiment whose out come has to be among a set
for example probability of getting an even number after of events that are completely known but whose exact
throwing a die can be calculated as. out come is unknown is mndom experiment.
(P) = {2, 4, 6} = ~ = _! Eg: (Throwing of a dice, tossing of a coin)
(1,2,3,4,5,6) 6 2
Sample Space
Were (2, 4, 6} are even numbers & hence favorable out This is defined in the context of a random experiment
comes and {"1 ,2,3,4,5,6} etc. all 6 possible outcomes. and denotes the set representing all the possible
Similarly, we can calculate probability of getting a outcomes of the random experiment.
number divisible by 3 out of 9 single digit number in
following manner Probability Eg: 1 . Sample Space when a coin is tossed is Head
(H) or Tail (T)
__ Number divisible by 3
2. Sample space when a dice is thrown is ( 1, 2, 3,
All nine single digit number
4, 5, 6)
_ {3, 6, 9) 3 1
- {1, 2, 3, 4, 5, 6, 7, 8, 9} = 9 ⇒ 3 Event
The followings remark may be important for learning The set representing the desired out come of a random
this chapter on probability experiment is called event. Event is subset of a sample
1. Die: A die is small cube used in games of chance. space for example: Probability of getting a number
On its six faces dots are marked numbering (1, 2, divisible by 5 in a single throw of a die if odd numbers
3, 4, 5, 6) are obtained.
® Probability I 143
Here sample space is (1, 2, 3, 4, 5, 6), event is (1, 3, 5) Example: The probability that in two throws of a dice
favourable outcome is (S)only. we get a total of 7 or more given that in the first throw of
1 the dices number 4 had occured.
So probability= { {S} } =
1, 3, 5 3 Concept of AND and OR
Non Event Whenever we use AND as the natural conjunction joining
The outcome that is opposite of the desired event is two separate parts of event definitions, we replace the
the non-event. AND by the multiplication sign.
Note: If the event occurs, the non-event does not ocur
Example: It probability of passing in an exam is 1/2 for
and vice versa.
A and probability of passing the same is11/3 for B then
Impossible Event probability of passing of A and Bis P(A)x P (B)
An event that can never occur is an impossible event. . 1 1 1
The probability of an impossible event is 0. I.e.: 2X3=6
Eg. (Probability of occurance of 7 when a dice with 6 Whenever we use OR as the natural conjunction joing
faces numbered 1-6 is thrown). two separate parts of the event definition, we replace
the OR by their addition sign.
Mutually Exclusive Events
A set of event is mutually exclusive when the occurence Example: If we have the probability of A winning a race
of any one of them means that the other event cannot as 2/3 and that of Bas 1/6 then probability that either A
occur. or B wins a race is given by
Eg: 1. If head appears on a coin tail will not appear
. 2 1 5
and vice versa P(A) + P(B) = + = .
3 6 6
2. If 5 occurs in a single throw of a die then (1, 2,
3, 4, 6) will never appear.
□□□□
Equally likely Events
If two events have the same probability or chance of
occurence they are called equally likely events. Solved Example
Example: In a throw of a dice, the chance of 1 showing
on the dice is equal to 2, is equal to 3, is equal to 4, is 1. In throwing a fair dice, what is the probability of
equal to 5, is equal to 6 appearing on the dice. getting the number '3'?
1 1
Exhaustive Set of Event (a) 3 (b) 6
A set of events that includes all the possibilities of a
sample space is said to be an exhaustive set of an 1
(c) - (d) 12
events. 9
Example: In a throw of a dice the number is less than 2. Find the chance of throwing at least one ace in a
four or more than or equal to four. simple throw with two dice.
1
Independent Events (a) 12 (b) 3
An event is described as such if the occurence of an
11
event has no effect on the probability of ocurence of (c) 4 (d) 36
another event.
3. Find the chance of drawing 2 blue balls in succession
Example: If the first child of a couple is girt there is no
from a bag containing 5 red and 7 blue balls, if the
effect on the chances of the second child being a girl.
balls are not being replaced.
Conditional Probability 3 21
Probability of the occurence of an event A given that (a) 13 (b) 64
event B has already ocured. 7 21
This is denoted by P (A/B) (c) 22 (d) 61
144 I ® Reasoning & Aptitude fflRDE ERS!!:11
4. If a card is picked up at random from a pack of 9. Two fair coins are tossed. Find the probability of
52 cards. Find the probability that it is obtaining
(i) a spade. (i) 2 Heads
1 1 2
(a) - (b) (a) 1 (b)
9 6 3
1 1
(c) - (d) - 1 1
4 3 (c) (d)
2 4
(ii) a king er- queen. (ii) 1 Head and i Tail
3 2
(a) (b) 1
B 13 (a) - (b) -1
4
7
(c) (d)
52 169 1 2
(c) (d)
2 3
(iii) 'a spade' or 'a king' or 'a queen'
(iii) 2 Tails
21 5
(a)
52 (b) 13 1
(a) 1 (b)
4
19
(c) - (d) None of these
52 2 1
(c) (d) -
3 2
5. Three coins are tossed. What is the probability of
getting 2 Tails and 1 Head 10. In rolling two dices, find the probability that
1 3 (i) there is at least one '6'
(a) - (b)
2 8 11 22
(a) (b) -
2 3 36 36
-c- - (u
3 4 15 29
(c) - (d) -
36 36
6;• For the above question, the probability that there is (ii) the sum is 5
at least one tail is :
1 i
(a) - (b)
2 7 4 9 if
(a) - (b)
3 8 1 i
I ,.J)
(c) \U
l
3 1 2 6 j
(c) - (d) - '
8 2 11. From a bag containing 4 white and 5 black balls a )
7. A bag contains 3 green and 7 white balls. Two balls
are drawn from the bag in succession without
man draws 3 at random. What are the odds against I'
il
these being all black?
I
replacement. What is the probability that both are
white? (a)
37
5
(b) -
37
5
;
I
1 5 11 '13
(a) - (b) (c) - (d)
7 11 37
13
7 7 12. Two balls are to be drawn from a bag containing 8 Jl
(c) - (d)
11 15 grey and 3 blue balls. Find the chance that they will
both be blue.
8. What is the probability of throwing a number greater
than 2 with a fair dice?
2 2
(a) -
5
1
(b) -3
55
1-
I
'k'l!
:jlj
(a) -r, (b) - i
0 5
11 i4 ]
r,
0 (c) (d) -45
(c) (d) -
5
'15 -1
<iii
sil
fflADE EASY ® Probability I 145
13. A bag contains four black and five red balls. If three (iii) one is white and one is black.
balls from the bag are chosen at radom, what is the i3 15
chance that they are all black? (a) (b)
24 24
1 1 11 1
(a) 2i (b) 20 (c) (d) -
21 2
2
(d) - □□□□
(c) 23 9
So P = -
8
7 c: : ;:
5
9
C 10 5
84 = 42 out of 42 cases only 5 are
22100 5525
1 3
Practice Exercise: I (a) (b)
4 4
1
i. Three coins are tossed. Find the probability of no (c) - (d) None of these.
2
heads.
3 1 8. In a simultaneous throw of two dice, find P (A or B)
(a) (b)
8 8 if A denotes the event 'a total of '11 and B denotes
1 the event' 'an odd number on each die'.
(c) (d) None of these
2
11 1
2. A coin is tossed three times. Find the chance that (a) 36 (b) 4
head and tail show alternately.
3 1 5 '1
(a) (b) (c) 18 (d) 6
8 4
1
(c) - (d) None of these 9. A and B are mutually exclusive events of an
8
experiment. If P ('not A') = 0.65, P(AuB) = 0.65 and
3. In a single throw of two dice, find the probability of P(B) = P, find the value of p.
getting a total of 3 or 5. (a) 0.70 (b) 0.30
1 2 (c) 0.63 (d) 0.35
(a) - (b)
3 3
10. The probabilities that a student will recive an A, B,
1 5 C or D grade are 0.30, 0.38, 0.22 and 0.01,
(c) - (d) -
6 6 respectively. What is the probability that the student
will receive at least B grade?
4. In a single throw of two dice, what is the probability (a) 0.38 (b) 0.42
of a doublet? (c) 0.68 (d) None of these
1 5
(a) (b) 11. A card is drawn from an ordinary pack and a gambler
6 6
bets that it is a spade or an ace. What are the odds
1
(c) - (d) against his winning the bet?
9 i8
(a) 9 : 4 (b) 4 : 9
5. What is the chance that a leap year, selected at (c) 5 : 9 (d) 9 : 5
random will contain 53 Sunday?
12. A problem in Statistics is given to four students A.
i 2
(a) - (b) - 8, C and D. Their chances of solving it are
7 7
1 1 '1 1 . .
3 4 -, -, -, and - , respectively. What 1s the
(c) (d) 3 4 5 6
7 7
probability that the problem will be solved?
6. Find the probability that in a random arrangement
of letter of the words "UNIVERSITY" two 'I's do not
1 2
(a) 3 (b) 3
come together.
4 1 4
(a) (b) (d) None of these
5 5 (c) 5
3 2
(c) (d)
5 3 Direction for Question No. 13- i 4: An urn contains 25
balls numbered 1 to 25. Suppose getting an odd
7. An interger is chosen at random from first two
numbered ball is considered a 'success'. If two balls
hundred natural numbers. What is the probability
are drawn from the urn with replacement.
that the integer chosen is divisible by 6 or 8?
i 48 I ® Reasoning & Aptitude fflRDE EAS~
Oiiection for Question No. 15-18: A husband and 21. Find the probability that in a random arrangement
wife appear in an interview for two vacancies in the same of the letters of the word DAUGHTER, the letter 0
post. The probability of husband's selection is 1/7 and occupies the first place.
that of wife's is 1/5. 1 i
(a) - (b) -
8 4
15. What is the probability that only one of them will be
selected? 3 1
(c) - (d) -
8 2
2 1
(a) 7 (b) 35
□□□□
24 11
(c) 25 (d) 35
(a)
2
(b) -
i Required probability= ~ =2 .
7 35 8 4
-<-<
24 I I
(c) (d) 3. Ans. (c)
35 35
A total of 3 or 5 may be obtained in 6 ways, viz.,
19. A man speaks truth in 80% of the cases and another
(1, 2), (2, i), (1, 4), (2, 3), (3, 2), (4, 1).
in 90% of the cases. While stating the same fact,
what is the probability that they contradict?
No. of exhaustive cases= 6 x 6 = 36.
Probability of getting a total of 3
( ) 37 (b) 1r3
6 1
a 50 ;:;0
or 5 = 36 = 6.
® Probability I 149
= Total number of words Number of words in which = P (A)+ P(B) - P(A and B)
1-.!=~ =
6 4
1--X-= 1 - - = - ,
24 ii
6 6
7 5 35 35
Since the events are independent, the probability
that all the four students fail to solve the problem is 19. Ans. (b)
Let the two men be A and B.
2 3 4 5 1
-X-X-X-=-. A: A speaks truth
3 4 5 6 3
B: B speaks truth
:. The probability that the problem will be solved
P(A) =
80
⇒ P(A) = 1-P(A) = 1- 80 = 20
=1-.!=~- 100 100 100
3 3
P(B) =
90
. 100
⇒ P(B), = 1- P(B), = 1- 100
9
o = J_Q_
100
13. Ans. (a)
Required probability
. 13
Success: Getting odd number P = 25 = P(A)P(B) + P(B)P(A)
Practice Exercise: II city is 50%, then what is the probability that it rains
on exactly 3 days in a 5-day period?
(a) 8/125 (b) 5/16
1. The Probability of raining on day 1 is 0.2 and on (c) 8/25 (d) 2/25
day 2 is 0.3. What is, the probability of raining on
8. The probability that an event A happens in one trial
both the days?
of an experiment is 0.4. Three independent trial of
(a) 0.2 (b) 0.1
the experiment are formed. The probability that the
(c) 0.06 (d) 0.25
event A happens at least once is
2. A bag contains 5 red balls and 8 blue balls. It also (a) 0.934 (b) 0.784
contains 4 green and 7 black balls. If a ball is drawn (c) 0.548 (d) 0.343
at random, then find the probability that it is not
9. A number is chosen at random among the first 120
green.
natural numbers. The probability of the number
(a) 5/6 (b) 1/4
chosen being a multiple of 5 or 15 is
(c) 1/6 (d) 7/4
(a) 1/5 (b) 1/6
7 (c) 1/7 (d) 1/9
3. If the probability that A will live 15 years is
8 and
9 10. From a pack of 52 playing cards, two cards are
that B will live 15 years is
10 , then what is the drawn together at random. Calculate the probability
probability that both will live after 15 years? of both the cards being Kings.
1 (a) 1/15 (b) 25/57
(b) 63
(a) 20 80 (c) 35/256 (d) Noneofthese
1 11. From a box containing 60 standard and 40
(c) 5 (d) None of these
substandard article, two articles are chosen one by
4. The probability that a student is not a swimmer is one. What is the probability that one of them is
1/5. Then the probability that out of the five students, standard and the other substandard?
exactly four are swimmers, is
60 40
(a) 100 X 100
(a) sc4(if (2) 16
(c) 33 (d) 24%
(c)
5
c{i)(if (d) None of these
6 6 2. Ans. (a)
(c) 19 (d) 23
The probability that the ball drawn is of green colour
None= n
Total sample sets= 3
Clearly 4 regions are then (x, y, z & n) Here X, Y, Z are different 3 attributes hence
Region x can be referred as (a) Total number of regions = 2 3 = 8
(a) z = things that belongs to both attribute
(b) n(A) = x + w + u + p, things belong to only
A&B
A= X
(b) x = things that belong to A but not to B
(c) n(B) = u + p + v + y, things belong to only
(c) y = things that belong to B but not to A
B=y
(d) n = represent region of thing which neither
(d) n(C) = w + p + v + z, things belong to only
belongs to A nor to B.
C=z
Further observations (e) Things belong to exactly one of attribute
(a) If things belong to exactly one attribute tr1en =x+y+z
region= x + y (f) Things belong to attribute (A & B) both
(b) If things belong to at least one of the
n(A n B) == u + p
attribute = exactly one + exactly two
(g) Things belong to attribute (B & C) both
n(A u B) = x + y + z.
n(B n C) = v+ p
(c) Total sample(µ): [(Things belongs to atleast
(h) Things belong to attribute (A & C) both
one)+ (things belong to none of attribute.))
n(A n C) = w + p
µ=x+y+z+n
(d) Things belong to both attribute (i) Things belong to only attribute (A & B)
n(A n B) = Z both= u
(e) Things belong to attribute A (j) Things belong to only attribute (B & C)
n(A) == x + z both= v
l
154 111 Reasoning & Aptitude fflRDE ERS~
(k) Things belong to only attribute (A & C) 13. B & C both = n(B n C) = k + h + o + I
both= w 14. B & D both= n(B n D) = m + j + o + I
(I) Things belong to all the three attributes 15. C and D both = n(C n D) = m + o + I + f
n(A n B n C) = p 16. Only to (A & B) both = e
(m) Things belong to all the three attributes 17. Only to (A & C) both= g
n(A n B n C) = p 18. Only to (A & C) both = i
(n) Things belong to none of the attributes = n 19. Only to (8 & C) both = h
(o) Things belong to at least one of the attributes 20. Only to (8 & D) both = j
Total sample- n = µ- n 21. Only to (C & D) both= f
= Exactly one + Exactly two + 22. To exactly two out of four
Exactly three =e+f+g+h+i+j
=x+y+z+u+v+w+p 23. (A, B & C) all three
n(A u B u C) = n(A) + n(B) + n(C) = k + 0 = n(A n B n C)
- n(A n B) - n(B n C) - n(C n A) 24. (B, C & D) all three
+ n(An BnC) = I + 0 = n(B n C n D)
(p) Things does not belong to A= n + z + u + y 25. (A, B & D) all three
(q) Things belong to A or
= m + 0 = n(A n B n D)
B=x+u+y+w+p+v 26. (A, C & D) all three
(r) Things be!ong neither l\ nor B = (z + n)
= n + 0 =n(A n C n D)
Venn diagrams with 4-attril:mtes 27. Only to (A, B, C) all three = k
28. Only to (8, C, D) all three = I
29. Only to (A, B, D) all three ::::: m
30. Only to (A, C, D) all three = h
31. To exactly three of four attributes
--l-------'-'-l------'-'--+-1,----l----------~-'-P,-,{,,.--r-+--r+l--4--1-----------
D
Solved Examples
Direction (Q. i to Q. 5): These examples are based on
None= p
the following diagram.
Total sample = m
'1. n(A) = a + e + g + k + n + o + j + m
2. n(B) = b + e + h + k + I + o + j + m
3. n(C) = c + f + h + I + k + o + g + n
4. n(D) = d + f + j + I + m + o + j + n
5. Only A= a
6. Only 8 = b
7. Oniy C = c "-.__/z
8. Only D = d
9. Things belong to exactly one of attributes
=a+b+c+d Total sample = First 50 natural numbers.
10. A & B == n(A n B) = e + k + o + m Circle X- represent even numbers in sample.
11. A & C both = n(A (') C) = g + k + o + m Ciicle Y-repiesent odd numbers in sample.
12. A&Dbothc::n(AnD)=n+o+i+m Circle Z-represent prime number in sample.
roADE EASY ® Set Theory I 155
Solution : Following venn-diagram can be drawn. Direction (Q.6 to Q. 10): In summer a survey was
conducted in a colony to know how many houses are
X equipped with fans, AC's & coolers 224 houses have
coolers. 10% of the houses have coolers & fan. 42%
houses have AC's. 44% of the houses have fans. 10%
of houses have none of three. 22% have only fans &
12% of the houses have AC's & coolers.
Sol.
Since maximum data 18 is % terms we can
comfortably work by assuming sample size to be
1 . How many numbers belong to X only? 100%.
(a) 25 (b) 24 And we can draw following venn-diagram.
(c) 23 (d) 22
Ans. (b)
AC
Only even number but not prime number hence 24 42%
elements, option (b).
X None
10%
8. How many total houses were surveyed? 1 . The number of girls who like Cricket is
(a) 640 (b) 768 (a) 140 (b) 170
(c) 600 (d) 800 (c) 190 (d) 230
Ans. (d)
2. The number of boys who like both games is
Total houses surveyed 800
(a) 20 (b) 40
Hence option (d)
(c) 60 (d) 100
9. How many houses do not have fan?
3. How many boys like none of the games?
(a) i76 (b) 352
(a) 0 (b) 90
(c) 448 (d) 288
(c) 170 (d) 190
Ans. (c)
Over all 44 % houses have fans 4. How many students like Hockey?
Hence 56% don't have fans. (a) O (b) 320
(c) 160 (d) 240
800
Hence 56 x - == 448
100 5. How many girls like at least one of the game?
Hence option (c) (a) 270 (b) 360
(c) 290 (d) 230
10. What is the ratio of number of houses that have
exactly 2 out of 3 appliances to those who have at Direction (Q.6 to Q. 8): During the annual day
most one appliance. celebrations of a college 225 students participated. Out
(a) i 3 : 35 (b) 10: 35 of them 100 played Cricket, i 05 played Hockey, 95
(c) 3 : 7 (d) 4 : 7 played Football, 20 played none of these three, 45
Ans. (a) students played exactly two of the above three sports,
No. of houses having 2 out of 3 == 26% of population 20 student played all three game.
No. of houses having at most one appliance= 70% 6. If 45 students played only Cricket, then find the
·------1-,ati.@-=-+3~5;,___ _ _ _ _ _ _ _ _ _ _ _ _ _ _n_u_m-,-b_e_ro-:f:-s-tu-d~e-n--=ts-w,-h,-o-pl~a-ye_d_F_o_o:._tb_a_ll_&_H_oc_k_e_y___,_
Hence option (a) only
(a) 15 (b) i0
(c) 20 (d) 25
m Practice Exercise: I
7. If 20 students played both Cricket & Football but
not Hockey, find the number of students who played
only Hockey
(a) 50 (b) 55
Direction (Q.1 to Q. 5): The following table gives partial (c) 45 (d) 60
information about the number of students of a school
8. If 50 students played Hockey but not Football, find
who like Hockey & Cricket. There are 800 students in
the number of students who played Cricket or Hockey
the school.
(a) 110 (b) 120
(c) 135 (d) 140
Hockey Cricket Both Total
Boys 100 300 Direction (Q.9 to Q.13): In an apartment, a survey was
Girls 170 conducted among 600 people to find out readership of
3 magazines India today, Business today and Front line. ;
Total 800
It is known that 200 people read at least two of these
magazines. 460 people read either India today or Front ;,
I
of the total students, 50% like Cricket, 30% like Hockey line. 360 read exactly one among three. 160 read neither '
& 20% !ike both the games. 20% of the girls like both India today nor Business today. 260 read the India today ;
the games. or Business today but not Front line.
"'
® Set Theory I i57
g. How many people read at least one of the other two (a) 70 (b) 40
magazines along the Front line? (c) 80 (d) None of these
(a) 140 (b) 180
20. What is the minimum possible number of people,
(c) 200 (d) 220 who took at most one of the two drinks?
10. How many people read all three magazines? (a) 40 (b) 50
(c) 60 (d) 70
(a) 140 (b) 120
(c) 80 (d) Can't determined 21. What is the minimum possible number of people
who took none of these two drinks?
11. How many people read India today only?
(a) 20 (b) 30
(a) 100 (b) 120
(c) 40 (d) None of these
(c) 140 (d) i60
14. What is the maximum possible no. of people who Q.1 to Q.5
took none of these two? Number of girls who like both games
(a) 60 (b) 30
20
(c) 40 (d) None of these = 300x-=60
100
15. What is the maximum possible number of people Number of students linking none of two games
who took exactly one of these two drink? is40%
(a) 80 (b) 90 40
(c) i20 (d) 100
Hence - x 800 = 320 students like none of
100
the game.
i 6. What is the maximum possible number of people
Total number of students who like both game
who took both of the tea and coffee?
20
(a) 80 (b) 90 =- 800x- = i60
iOO
(c) 70 (d) 100
Total number of students who like Cricket
17. What is the maximum possible number of people
who took at least one of these two drinks? ::: 800 X SO = 400
iOO
(a) 120 (b) 100
Total number of students who like Hockey
(c) 90 (d) Noneofthese
30
= 800x = 240
18. What is the minimum possible number who took both iOO
the drinks? Now we are easily able to complete the table
(a) 40 (b) 30 Total No. of Like none two
Hockey Cricket Both
(c) 50 (d) None of these Students of games
Boys 100 230 100 300 170
19. What is the minimum possible no. of people who Girls 140 170 60 500 270
Total 240 400 160 800 440
took at least one of the two drink?
158 I 0 Reasoning & Aptitude fflRDE ERS~
Boys= b + 20 + 25 = 105
b = 105 - 45 == 60
Hence option ( d)
8. (b) x + b = 50
x + b + z + 20 = 105
None= 170
=> Z=35
X + y + Z = 45
⇒ x+y=iO
a + + y + 20
X = 100
⇒ a-70
a + x + b = 70 + 50 = 120
None= 270 Hence option (b)
Question 9 to 13
1. (c) No. of girls like cricket = i 70.
2. (d) No. of boys like both games= 100. India Today
Question. No. 6 to 8
14.(c)
- - - - - 120 people .....-----4>1
70 tea
70 tea
Answer 30. Option (b)
Maximum possible no. of people took none of
19.(c)
these two.
(120-80) = 40. Option (c)
- - - - - 120 people -----+1
15.(b) To maximize the no. of only one drink taken try
to minimize both and none.
- - - - - 120 p e o p l e - - - - - ?0tea
- - - 80 coffee ....---~
To minimize the no. of people who took at least
one, we need to maximize the number of people
70tea who took none. Option (c).
Maximum number of only one drink, taken 20.(b) To minimize the no. of people who took at most
= (80-30) + (70- 30) = 90. Option (b) one. I need to maximize who took both.
120 people
offee ---l>I
?0tea
a
30
70tea
70tea
© S ll.1 ,
In the exams, the success of a candidate in the questions Sol.(b) The only daughter of the woman's father is the
on blood relations depends upon his knowledge about woman herself, and hence the man in the
various blood relations. The easiest and non-confusing photograph is her son. Therefore the woman is
way to solve these problems would be to draw a family the mother of the man in the photograph.
tree diagram and increase the levels in the hierarchy as
Example 2.
shown below:
Pointing to a man in photograph a woman said "The
1st stage:
father of his brother is the only son of my grand
Grand Parents-Grand father, Grand mother, Grand
father". How is the woman related to the man in
uncle, Grand aunt.
photograph.
2nd Stage:
Parents & in laws-Father, Mother, Uncle, Aunt, Sol.(d) The only son of woman's grand father is the father
Father-in-law, Mother-in-law. of woman and the father of the man's brother is
rd
3 Stage: the father of the man. On combining these two
Siblings, spouse & in laws-Brother, Sister, Cousin, information together single information emerges
Wife, Husband, Brother in law, Sister-in-law.
that the man's father is the woman's father. Hence
111
4 stage:
woman is sister of the man in the photograph.
Children & in laws-Son, Daughter, Niece, Nephew,
Son-in-law, Daughter-in-law. Direction for Ex. 3 to 5.
th
5 stage: A + B means A is the daughter of B
Grand children-Grand son, Grand daughter. A - B means A is the husband of B
A x B means A is the brother of B
Mother's or father's brother Uncle
Mother's or father's sister Aunt Example 3.
Mother's or father's mother Grand mot11er If P + 0 - R, which of the following is true:
Mother's or father's father Grand father
Grand father's or grand mother's brother Grand uncle (a) R is the mother of P
Grand father's or grand mother's sister Grand aunt (b) R is the sister-in-law of P
Children's children (Grand son, Grand daughter) Grand children
Uncle or Aunt's son/daughter Cousin (c) R is the aunt of P
Son's wife Daughter in iaw (d) R is the mother-in-law of P
Dauther's husband Son in law
Sister's or brother's son l ✓ephew
Sol.(d) P + 0 - R means Pis the daughter at Q who is
Husband's or wife's brother or sister's husband Brother in law
Husband's or wife's sister or brother's wife Sister in law the husband of R. i.e. R is P's mother.
Chindren at same parents: siblings (could be broiner.
a!I sister's or some brother's & some sister's Example 4.
If P x O + R, which of following is true
Example 1. (a) P is the brother of R
Pointing to a man in a photograph a man said to a (b) P is the uncle of R
woman "his mother is the only daughter of your (c) P is the son of R
father". How is the woman related to the man in the (d) P is the father of R
photograph.
(a) Sister (b) Mother Sol.(c) P x O + R means Pis the brother of Q who is
(c) Wife (d) Daughter daughter of R. i.e. Pis son of R.
162 I ® Reasoning & Aptitude fflROE ERS'
Example 5. Example 9.
If P + 0 x R, which at following is true If T - S x B - M, which of the following is not true
(a) P is the Niece of R (a) B is mother of S
(b) Pis Daughter of R (b) M is husband of B
(c) Pis Cousin of R (c) Tis wife of S
(d) P is the Daughter-in-law of R (d) Sis daughter of B
Sol. ( a) P + 0 x R, means Pis the daughter of Q who is Sol.(d) T - S x 8 - M means Tis the wife of S, who i
the brother of R i.e. P is niece of R the son of B who is the wife of M i.e. T is th<
wife of son at M (father) i.e. Tis the daughter-in
Example 6. law of M. But as this is not qiven in the choic:e
A party consists of grand mother, father, mother, So the fact in ( d) is not true.
four sons & their wives and one son and two
daughters of each of the sons. How many females
Example 10.
If Z x T - S x U + P, what is U to Z.
are?
(a) Mother
(a) 14 (b) 16
(b) Grand mother
(c) i8 (d) 24
(c) Father
Sol.(a) 14 (d) Can't be determined
Grandmother is one female, mother is another,
Sol.(b) Z x T -S x U means Z is the son of T who is thE
wives of four sons are 4 female and two
wife of S who is the son of U. i.e. Z is the son o1
daughters of all four sons are 8 female. So all
S who is the son of U. i.e. U is the grandson o1
are
U or U is the grand mother of Z. From the choice
1 + 1 + 4 + 8 = 14 female
(b) is true.
Example 7.
Laxmi and Meena are Rohan's wives Shalini is Some Standard Coding Technique
___ --· Meena 's stepd.augbteLl:::hwJs.La.xrni ..r:@late-Ef-te----l~..Er.ope.r:.AooR-slclmJEl--alwa-~enorec:tby·c-aplta1~
Shalini? letters
(a) Sister (b) Mother-in-law E.g. John, Merry and Puul can be codes a:::i
(c) Mother (d) Stepmother Johh -J, Merry - M, Paul - P
[Note: Here sex of A & Bis not mentioned] 4. Who is the wife of T?
ex. ➔ Seema is wife of Rahul ➔ R .S. (a) P
ex. ➔ Priya is wife of Rajesh R - .E (b) U
[Note: It is better to write male on the left & female (c) 0
on the right hand side to avoid any confusion] (d) can't be determined
(e) Is the sibling of
ex ➔ A is sibling of 8
5. Which of the following is a pair of females?
i. A .. B or (A & Bare brothers)
(a) PT (b) OS
2. 6 .. B or (A is sister of B) (c) SU (d) PS
3. A .. B or (B is sister of A) 6. How is U related to S?
4. 6 .. B or (A & B are sisters) (a) Father
(f) Grand father, Grand son relation (b) Brother
ex. ➔ Ramesh & Param are the two sons of Mr.
(c) Uncle
Shiv Kumar, who is the son of Mr. Sundarlal
(d) can't be determined
Su
Direction (Qs. 7 to 12): Read the following
➔ Sk information carefully and answer the question
R .. P below it.
A family consists of six members P, 0, R, X, Y and Z. Q
(Note: Shiv Kumar and Sundarlal are coded as Sk
and Su to avoid any. confusion) is the son of R but R is not mother of 0. P and Rare a
(g) Where the sex of a person is not confirmed married couple. Y is the brother of R. X is the daughter
write it in a box of P. Z is the brother of P.
like O is the only son of S
7. Who is the brother-in-law of R?
~ (Here O is male but sex of (a) P (b) Z
(c) Y (d) X
0 Sis not mentioned)
8. Who is the father of O?
□ aa □ (a) R (b) P
(c) Z (d) None of these
13. How is P related to S? 22. Who are the females in the group?
(a) Cousin (b) Nephew (a) T and P (b) Q and P
(c) Uncle (d) Brother (c) Rand O (d) T and 0
i 5. How many nephew does U have? Direction (Os. 24 to 28): Read the following information
(a) Nill (b) One carefully and answer the question below it.
(c) Two (d) Three (i) A is the married to B and L is A's brother-in-law. A
16. nanja11 is U,e brother of Sachin and Manick is the has two daughters.
father of Ranjan. Jagat is the brother of Priya and (ii) I is the cousin brother of J and is the brother of f<..
Priya is the daughter of Sachin. Who is the uncle of (iii) E and Fare B's son-in-law.
Jagat? (iv) E has 2 daughters and 1 son; F has 1 son and
(a) Ranjan (b) Sachin 1daughter.
(c) Manick (d) None of these (v) G and H are 2 daughter of C.
(vi) K and A share a grand daughter and grand
Direction (Os. 17 to 18): Read the following infor-mation relationship.
carefully and answer the questions given below it. (vii) D is also member of this family.
(i) A is the father of C
(ii) E is the daughter of C.F is the spouse of A 24. How is C related to I?
(iii) B is the brother of C.D is the son of B. (a) Mother (b) Brother
(iv) G is the spouse of B. His the father of G. (c) Aunt (d) Cousin
Direction (Os. 19 to 23): Read the following inforrnation 27. I low is D relc1ted to E? .
carefully and answer the questions given be,ow it. (a) Brother-in-law (b) Sister-in-law
P,O, R, Sand Tare members of the same family. There (c) Daughter (d) Wife
are two fathers, two sons and, two wives, three males
28, How is E related to C?
and two females in the group. The Engineer was the
(a) Daughter (b) Son-in-law
wife of Teacher, who was the son of a Painter.Tis neither
(c) Wife (d) Husband
a male nor the wife of a professional. R is the youngest
person in the family and S is the eldest. 0 is a male
□□□□
19. How is P related to O?
(a) Husband (b) Wife
(c) Mother (d) Daughter
Solutions
20. Who is the father of R?
Hints (1-6)
(a) P (b) 0
Sol. ➔ Q is son of R
(c) S (d) T
➔ R is not the mother, then R is father of O
21. Whose wife is the Engineer? ➔ P and Rare married couple R - .E
(a) P (b) 0
➔ T is brother of R T .. R
~) R ~) S
mRDIE ERSY ® Blood Relation I 165
p Pain
➔ S is daughter of P S ➔ Tea
I is a female and wife of Painter
➔ U is brother of Q Q .. U Pain -I
Complete Solution Tea-Eogg
T.. R -2 R
Q .. u .. s. ➔ R is youngest
1. (d) It means R is Son of Tea
2. (d) ➔ S is eldest, it means S is the grandfather
3. (c) Pain-I
4. (d) (S)
5. (d) Tea-Eogg
6. (b) (Q)-P
Sol. (7 - 12) R
Y .. R-E .. Z now complete tree is like this
0 .. x S -- I
7. (b) Q E
8. (a) R
9. (b) 19. (b)
iO. (b) 20. (b)
ii. (c) 21. (b)
Solutions 5. (a)
Woman's mother ~==I Mother's brother
i. (a)
Woman (Man)
Bhasker's Father
Hence man is woman's Nephew.
Father = Bhasker
6. (a)
Harsh father
Akash father
Harsh is looking at his son's picture.
Boy in blue
Akash
7. Sister (d)
shirt = brother,___ __, Daughter
(Vipui's father)
ll!llll!IUlillll
Direction (Qs. i to 5): Read the following information Direction (Qs. 6 to i 0): Read the following information
carefully and answer the question below it. carefully and answer the question below.
i. If 'Rat is called Dog', 'Dog is called Mongoose', L1 means 'is greater than', % means 'is lesser than', D
'Mongoose' is called Lion;, 'Lion is called Snake' means 'is equal to',= means 'is not equal to', + means
and 'Snake is called Elephant', which animal is reared 'is a little more than', x means 'is a little less than'.
as pet? 6. If a Lib and b + c, then
(a) Rat (b) Dog (a) a% c (b) c % a
(c) Mongoose (d) Lion (c) c + a (d) can't say
i 6. How is 'red' written in that code? 23. The digits in the unit is
(a) hee (b) silk (a) 2 (b) i
(c) be (d) none of these (c) 3 (d) can't say
17. How is 'roses' written in that code?
24. The digits in the tens place is
(a) ii
(a) 6 (b) 8
(b) pee
(c) 9 (d) 2
(c) be
(d) can not be determined 25. The digit in the hundreds place is
(a) 3 (b) 6
18. How is 'vegetables are red flowers' written in this (c) 9 (d) can't say
code?
(a) pee silk mit hee 26. The sum of the three digits is
(b) silk pee hee be (a) 7 (b) 12
(c) ii silk mit hee (c) iO (d) 14
(d) can not be determined 27. The three digit nu,:nber is
Direction (Qs. 19 to 22): Read the following information (a) 138 (b) 183
carefully and answer the question below it. (c) 381 (d) can't say
Shamita is very found of collecting greeting cards. Her 28. The three digit is number is always divisible by:
collection of 21 i cards has a good mix of Birthday cards, (a) 3 (b) 18
New year cards, Deepawali cards, Christmas cards and (c) 29 (d) None of these
even a Marriage Anniversary card. The number of
Direction (Os. 29 to 30): Read the following information
Birthday cards is equal to the sum of all other except
carefully and answer the question below it
the Marriage Anniversary card. The number of New Year
P, 0, R, S, T, U and V are seven positive integers and
card is double of Deepawali cards which in turn is double
(P x O x R x S x T x U x V) is odd.
of Christmas cards.
19. The number of Birthday cards in the collection is 29. Maximum how many of these integers can be odd?
(a) 95 (b) 105 (a) 4 (b) 5
(c) 110 (d) 85 (c) 6 (d) 7
170 I @ Reasoning & Aptitude mADE ERS'!:11
30. Minimum how many of these integers can be even? 4. (c) In the first and second statements, the common
(a) 3 (b) 2 code word is gnr and the common word is
(c) 4 (d) 0 Olympic. So, gnr means Olympic. In the second
31. In the sequence of alphabets, which letter would be and third statements, the common code is hyto
eight to the right of the letter which is sixteenth from and the common word is games. So, hyto means
the left? games. Thus, in the second statement, emf
(a) G (b) y means summer.
(c) Z (d) X
5. (a) From 1st and 3rd statements. Zoo means Good.
32. CEGJLN _ _ XZB. The missing group of letters in
the series are ... Answer 6 to 10:
(c:t) OSU (b) NPH 6. (b) a Lib and b + c ⇒ a> b
(c) PRT (d) TUX and b is a little more than c
⇒ a> C
Direction {Qs. 33 to 37): Read the following information
carefully and answer the question below it. ⇒ C <a
i.e. c % a.
In each of the Letter Analogy various terms of a letter
series are given with one term missing as shown by('?). 7. (c) c = a and a == b ⇒ c :t a and a ::f:. b
Choose the missing term out of the given alternatives. ⇒ b :ta i.e. b = a.
33. HUA GTZ FSY ERX? 8. (a) ax band b □ c ⇒ a is a little less than band b
(a) DWO (b) DOW = c.
(c) WOO (d) WOO ⇒ a is a little less then c.
34. OF GJ KM NO RT? ⇒ c is a little more than a i.e. c + a.
(a) UW (b) YZ
9. (a) c % b ⇒ c < b
(c) XZ (d) UX
and b x a ⇒ b is a little less than a.
35. DCXW FEVU HGTS? ⇒ c <a ⇒ a> c i.e. a~ c.
_ _ _ _ _ ~a)-kl{J20'--------i-(b )--ABZ'.'.--- ---------
10. (d) ac+bc ⇒ ac>bc ⇒ a>b ⇒ b<ai.e.b%a.
(c) JIRO (d) LMRS
36. AB DEF HIJK? STUVWX Answer 11 to 15 :
(a) MNOPO (b) LMNOP 11. (c) Using the proper notations in (3), we get the
(c) LMNO (d) ORSTU statements as 8 - 4 + 2 < 6 + 3 or 6 < g, which
37. CG LR? is true.
(a) Y (b) S 12. (b) Using the proper notations in (2), we get the
(c) U (d) Z statements as 4 + 3 x 8 - i = 6 -c- 2 + 24 or
27 = 27.
Solutions
13. (b) Using the proper notations in (2), we get the
Answer 1 to 5: statements as 9 + 5 + 4 = 18 + 9 + i 6 or 18 = i 8.
1. (c) 'Dog' is reared as pet. But 'Dog' is called
14. (d) Using the proper notations in (4), we get the
'Mongoose'. So, a 'Mongoose' is reared as pet.
statements as 3 i + i - 2 < 4 + 6 x 7 or 30 < 46.
2. (b) The 'Thumb' is a finger having a different namo.
15. (a) Using the proper notations in (1), we get the
But 'Thumb' is called 'A~kle'. So, 'Ankle' is the
statements as 7 + 7 - 7 -c- 7 < 14 or i 3 < i 4.
finger that has a different name.
3. (c) In the i st and 2nd statements, the common Answer 16 to 18:
code word is kew and the common word is she.
16. (b) In the second and third statements, the common
So, kew means she. In the i st and 3rd
code word is 'hee' and the common word is
statements, the common code is deko and the
;flowers'. So, 'hee' stands for 'flowers'. Thus, in
common word is apples. So, deko means
the second statement, 'silk' stands for ;red'.
apples.
® Coding and Decoding 171
17. (d) Since from the given information, we can only 222 (x + y + z) "" 2664 or .t + y + z = i 2
find the code for ·are' in the first statement, it Also x + z = y/2 thus y = 8
cannot be determined which of the remaining Thus, x + t = 4
two codes for 'rose!'!', As the digits are non l®rb and dl§tinm thus x
and z have to be i 1and 3 but not heC@iiisarily in
18, (a) Clearly, th@ required code will consi®t of the .
the same order. Thus we canndt Bay whether
sam® cod~~ at:1 in the third ~t®.t@m~ntt1 with the
the number Is 183 or 38 i,
code for 'red' iJf silk added to it.
23. (d)
Answer 19 to 22: 24. (b)
If; number of Christmas cards = x
25. (d)
Then, number of Deepawali cards= 2x
26. (b)
Number of New Year cards ;;;,: 4x
and number of Birthday cards 27. (d)
= x+2x+4x 28. (a)
= 7x Answer 29 to 32:
i.e. 7x + (x + 2x + 4x) + 1 "'211
29. (d) All integer should be odd to get odd result
i4x "' 210
30. (d) None Of the integer should be even.
X = 15
Thus, number of Birthday cards 31. (d)
= 16 X 7 32. (a)
= 106 Answer 33 to 37:
Nurnber of New Year cards 33. (b) All the letters of each term are moved one step
= 15 X 4 backward to obtain the corresponding letters bf
= 60 the next terrn.
Number of Oeepawali cards
34. (d) There is a gap of one letter between bbth the
f: 15 x 2
letters of first term; a gmp of two letters between
± 30
both the letters of second term and again a gap
Number of Christmas cards
of one and two letters between the letters of
= 15 x 1
third and fourth terms respectively. Besides, the
= i5
last letter of each term and the first letter of next
Number of Marriage Anniversary card
term are in alphabetical.
i: 1
Total = 211 35, (c) Pirf:lt two letters of eaGh term are in reverse order.
19. (b) Similarly third and fourth letters are also in
reverse order. Besides; the second letter of each
20. (b)
term is the letter next to the first letter of the
21. (b) proceeding terfll.
22.(a) i05:15==7:4. 36, (a) The number cf letters in the term goes on
Answer 23 to 28: increasing by 1 at each step. ta.ch term consists
Let the digits be x, y, z such that the number is of letters in alphabetical. The last letter of each
100x + 10y + z term and the fir~H letter of the next term are
Thus, the other five 3 digit number which we alterhate.
may Clbtls'i.ihed USihg these 3 digits Will be 37. (a) There is a gap of three letters between the first
100x + 10z + y and the second term, four letters between the
100y + 10x + z sect:md and the third term; and five letters
100y + 102'. + x between the third and the fourth term. So, there
100z + iOx + y should be a gap of six letters between the fourth
10oz + 10y + x term and the missing term.
Now when we add all the six three digit numbers IIIUUU!II
possible to be formed by these three digits:
Cubes
A type of cuboid in which all the sides i.e. length, I Practice Exercise: I
breadth & height are equal. All faces of cubes are
Directions for Questions (1 - 9):
of same area.
If a cube is cut into n3 identical cubelets using minimum
no. of cuts, after painting all faces of cube with white
color, then answer the following questions.
i. What is minimum number cuts required?
2. What is maximum number of possible painted faces
in once of such of cube!ets?
3. How many cubelets will have exactly 3 faces
In a cube there are 8-corners/vertices painted?
There are 6-faces [all equal in area]
4. How many cubelets will have exactly 2 faces
There are 12-edges [ all equal in length]
painted?
No. of edges = No. of vertices + No. of face - 2
5. How many cube lets will have exactly 1 face painted?
Primarily question in cubes are based on painting
·~--·of faces of cubes & then cutting the painted cube 6. f=tow many cubeletswnmave no faces painted.
into identical cubelets. Because questions are 7. How many cubelets will have at most 2 faces
based on the way cube is painted & how cuts are painted?
being made, hence we don't recommend any thumb 8. How many cube lets will have at least i face painted?
rule. Still however because some basic rules are
9. If initially V liters of paints was required to paint nil
always applicable which we'll be using as building
faces of cube than how rnuch extra paint will be
blocks for answering questions.
required such that all the unpained faces of all
8 If n equidistanced cuts are made (all parallel to same cubelets will be painted.
surface), cube will be divided into (n + i) identical
cuboidal pieces with each such cut there will 2a2 Direction (Os. 10 to 12) : A cube is divided into 216
new surface area will be generated which will be identical cubelets. Each cut is made parallel to some
unpainted. surface of cube. But before cutting the cube is colored
with green color on one set of opposite faces, red on the
@ If I want to cut my bigger cube into identical n 3
other set of opposite faces and blue on the third set.
cubelets, using minimum number cuts, I need total
3(n - 1) cuts, such that (n - i) cuts parallel to each 10. How many cubelets are there which are painted with
of these faces which are joining to corner. exactly one color?
8 If number of cuts are not multiple of three then cube (a) 96 (b) 108
can never be cut into identical cubes but still it can (c) i24 (d) 48
be cut into maximum number of identical cuboidal 11. How many cubelets are there which are painted with
pieces. To maximize such number of pieces we need at least two coluor?
to split the number of cuts into three parts which are (a) 96 (b) 56
closest. That we'll see in upcoming examples. (c) i08 (d) 124
ffl/RDE ERS~ ® Cubes and Dices I 173
12. How many cubelets are there which are painted with 21. What is the number of cubelets which are painted
at most one color? with exactly 3 colors?
(a) 124 (b) 108 (a) 4 (b) 6
(c) 160 (d) 96 (c) 8 (d) 2
Direction (Os. i 3 to 17): A cube is divided into 343 · 22. What is the number of cubelets possible which are
identical cubelets. Each cut is made parallel to some
painted with none of the color?
surface of the cube. But before doing that the cube is
(a) 16 (b) 24
colored with green color on one set of adjacent faces,
(c) 36 (d) LIB
red on the second and blue on the third set.
13. How many cubelets are there which are colored with
□□□□
exactly are color?
(a) 180 (b) 150
(c) 165 (d) 148 Solution
14. How many minimum cuts you have made?
1. Total number cuts required in 3(n - 1 ).
(a) 15 (b) 18
(c) 21 (d) 9 (n - 1) equidistant cuts parallel to each of 3 face
which are joining to corner.
15. How many cubelets are colored with exactly two
2. Maximum faces painted will be three in any such
colors?
cubelets, which will be in case of cubelets coming
(a) 48 (b) 36
(c) 51 (d) 50 out of corners of big cube after cutting.
3, 4, 5, 6.
16. How many cubelets are thin which are colored with
exactly three colors? Cubes painted from 3 faces = 8
(a) 6 (b) 4 Cubes painted from 2 faces = 12(n - 2)
(c) 2 (d) 8
Cubes painted from 1 face= 6(n - 2)2
17. How many cubelets are painted with multiple colors?
Cubes painted from none face = (n - 2) 2
(a) 51 (b) 53
(c) 115 (d) 120 Where 'n' is number of part into which each side is
divided.
r Direction (Os. 18 to 22): A cuboid is divided into 192
7. To find out the number of cubelets with at most
identical cubelets. This is done by 1:1aking minimum no.
of cuts possible. All cuts are parallel to some of the
2 faces painted, I need to remove all those cubes
face. But before doing so. The cube is painted with green which have exactly 3 faces painted
color on one set of opposite faces. Blue on other set of = Total no. of cubes - No. of cubes with 3 faces
opposite faces and red on remaining their pair of opposite painted
faces.
=(n 3 --8)
i 8. What is the maximum number of cubelets possible 8. To find out the number of cubelets with at least
which one colored with green colored only?
one face painted, I need to remove all the cubelets
(a) 48 (b) 64
which have no face painted
(c) 96 (d) 72
= n 3 - (n -· 2) 3
19. What is the minimum no. of cuts I have made?
(a) 15 (b) 18
9. As with every cut unpainted area of 2a2 is created
6a2 (n-1)
cubelets is = x V = (n - 1)V liters
682
[(n - 2) + 2]3 = 3 3 3 2
C0(n - 2) + C/n -2) x 2 + Cin 3 Red
2 3
- 2) X 2 + C3 X 23 As we can see in above figure. 3 faces are visible in
10. Exactly one color painted 3-diff. colours, out of hidden faces, bottom is red,
= 3C/n-2)2 x 2 one is green & another is blue.
=3 X (n - 2) 2 X 2
15. Number of cubelet with no face painted will
=3 X 42 X 2 = 96 = (n - 2)3 = (7 - 2)3 = 125
Option (a) No. of cubelets with 2 color
11. Basically we are looking for no. of cubes which = Total no. of cubelet - [cubelets with one color
colored on two faces or three face + cubelets with no color+ cubelets with three color]
= 3C2(n - 2) x 22 + 3C3 x 23 = 343 [ 125 + 2 + 165] = 51
= 3 X (n - 2) X 4 + 8 Option (c)
14. n3 = 343 = 73 ⇒ n = 7
(6 x8)
Minimum number of cuts= 3(n-1)
= 3(7 - 1) = 3 x 6 =18. Option (b)
® Cubes and Dices I 175
22. Number of unpainted cubelets will be 1. What is the minimum no. of cuts required?
(4 - 2) X (6 - 2) X (8 - 2) ::: 2 X 4 X 6 = 48 (a) 21 (b) 18
Dices are cubical structure with number of points from 1 3. How many smaller pieces have at least 2 face
to 6 are marked on faces. Dices are of two kinds. painted?
1. Symmetric: Where sum of numbers on opposite pair (a) 64 (b) 68
of faces is same. (c) 72 (d) 76
Naturally the faces opposite to each other will be 4. How many smaller pieces have at most one face
i -, 6, 2 -, 5, 3 ....-,, 4 painted?
2. Asymmetric: When sum of numbers on opposite (a) 248 (b) 264
pair of faces is different. (c) 268 (d) None of these
7. How many of the smaller cubes have only pink & (a) 4 (b) 5
black colors on them? (c) 1 (d) 2
(a) 16 (b) 18
16. The six faces of a cube as marked 1, 2, 3, 4, 5 & 6..
(c) 19 (d) 24
Given below are two different view of same cube.
8. How many smaller cubes or cubelets have exactly
two colors on them?
(a) 32 (b) 36
(c) 37 (d) 42
(I) (II)
9. How many the cubelets have exactly two painted
Which face is opposite the face mark as 6?
surface in two different colors?
(a) 3 (b) 4
(a) 36 (b) 32
(c) 2 (d) f\lone of these
(c) 48 (d) 38
10. How many of the cubelets have exactly one color 17. How many cuboids of dimensions 4 x 5 x 6 are
on them? required to form a cube of least size if cuboids have
(a) 107 (b) 109 to be placed adjacent, above or below each other?
(c) 96
(a) 1600 (b) 1800
(d) 113
(c) 1200 (d) Noneofthese
11. How many of the cubelets have exactly one painted
surface of exactly one color? 18. A cube has been cut into cuboids of size
(a) 84 (b) 108 2 x 3 x 4. What is least possible integer length of
(c) 96 (d) 102 the edge of cube and how many such cuboids are
obtained from this cube?
12. How many cubelets do not have pink color on them? (a) 96 (b) 72
(a) 120 (b) 100 (c) 60 (d) 84
(c) 150 (d) 125
Direction (Qs. 19 to 22) : Read the following information
~---~-3.L.How.mBI4L.cuheJe1s.hwablacknr-violet-coJor.0n-~carefDl1Y &-answer the q uest,on below ,t.
them but not pink color on them?
(a) 60 (h) 59 A dice is prepared in following manner?
(c) 51 (d) 64 (i) 1 should lie between 2 & 3
(ii) 2 should lie opposite to 3
14. The six faces of a cube are marked as 1, 2, 3, 4, 5 (iii) 4 should lie between 5 & 6
& G. Given below are two differer1l view of same (iv) 5 & 6 should lie opposite to each other
cube. (v) 4 should lie face down
~
(a) 5, 4, 1, 2 (b) 1, 2, 5, 6
(c) 5, 6, 4, 1 (d) 2, 6, 4, 5
~
(i) □□□□
mRDE EASY ® Cubes and Dices I 177
7. (c) The cubelets with only pink & black are found
at corners & along the edges PB
==2 PPB cubelets + 1 BBP cubelet + 4 P edges
x (6 - 2) = 19 cubelets.
Means our cube cut into 8 parts along Z-directions 8. (c) The cubelets with exactly two colors are found
say nz = 8, similarly ny = 7, nx = 6. at corners & along the edges having different
For total number of identical pieces we can say
colors on either side = 5 corner pieces + 8 edges
nx x ny x nz = 336
x (6 - 2) = 37 cubelets.
{(nx-2)+ 2} x {(ny-2) + 2} x {(nz-2) + 2}
Now look at table below: 9. (b) In the previous question there is no. restriction
! Answers for
on the number of painted surface. But in this
~Jo of pieces Formula
6 x7x8 cube question there is a restriction. The paint should
i 3-face painted be only on two faces only. Hence we'll not
I 23 8
i (Comer pieces)
consider cubelets from corner. Hence only 32
2-faces painted 4[(8-2)+(7-2)+
piece
4[(nx -2) + (ny -2)+(nz -2)]
(6-2)]=60
cubelets.
Only one face 2[(nx -2)(ny -2)+ (ny -2) 2((8-2)(7 -2) +(7 -2)
1 O. (d) The cubelets with exactly one color are
painted pieces (n 2 -2)+(n,-2)(nx-2)] (6-2)+(8-2)x(6 2)]
(a) One exactly at corner (PPP) = 1
Pieces no face
(nx -2)(ny -2)(11 2 -2) (8 2)(7-2)(6-2)=120 (b) At the three edges each of PP category =
painted
3(6 2) = 12
1. (b) Minimum no. of cuts (c) At the one edge of BB category= 1(6 2) = 4
(8 ·- 1) + (7 - 1) + (6 - 1) = i8 (d) At the middle of each of the six faces
2. (b) All pieces from corners of cube = 8 = 6 X (6 - 2) = 96
So total cub lets with exactly one colour are
3. (b) 2 faces painted + 3 faces painted 1 + 12 + 4 + 96 = 113
= 8 + 60 = 68
11. (c) In this question only the cupelets from middle
4. (c) No. faces painted + one face painted
of faces of cube will be considered
= 120 + i 48 = 268
= 6 x (6 - 2) 2 = 96 cubelets.
5. (c) (8 - 2)(7 - 2)(6 2) = 120
12. (d) The number of cubelets with no pink color
Answer 6 to 13: Cube is cut in 6 x 6 x 6 = 216 -- (36 from one pink surface + 30 from
= 216 cubelets second + 25 from third)
Pattern of painting is an follows. = 125 cubelets.
Pink
13. (c) Here cubelets without any face colored has to
Black 4--
--+ Black be removed from previous question number. The
Pink Pink (P) (B)
number of cubelets with black or violet but not
pink
= 125 -- (6 - 2) 3 = 61
178 I I!! Reasoning & Aptitude fflADE &:AS~
14. (c) As 3 is adjacent to (1 & 6) as W$II as i 8. (b) The least possible dlm~nsion of cubie
(4 & 4) hence 3 is opposite to the remaining LCM of 2, 3, 4 "" 12
face 5, No. of cuboids that can be obtained
15. (d) 2 & Ei both are adjacent to both 3 & 4,
Hence 2 & 6 have to be oppo6lte,
= 12x12x12 . _
,,.,,,,,,., ,,,,.,,.,, ce: 72
-- 2x3x4 · ··
16. (b) From (I) & (II) we know that 1 is adjacent to 3, 4,
Answer 19 to 22:
5, 6. Hence i is opposite to 2. We can find the
other two pairs of opposite faces by rotating
one of the view to reach another,
(3 or 2)
17. (b) Least possible dimension of cube is LCM of 19. (b) The face opposite tc 1 will be 4.
(4, 5, 6) =i 60
20. (a) The upper face is 1,
The number of cuboids required
21. (c) 6 is opposite to 5, hEmc€:l 1, ?, 3, 4 must be
= 60~60x60 = 1800 adjacent to 5.
· 4x5x6
22. (c) 3 is opposite to 2 hence 1, 4, 5, 6 are adjeicent
to 3.
t!IUIUIJIIII ·
1. Dalbir is facing south. He turns 136° in the (a) 20 metres (b) 30 metres
anticlockwise direction and then 180° in the (c) 50 metres (d) 60 metres
clockwise direction. Which direction is he facing
6. Kuldeep starts from his house towards West. After
now?
walking a distance of 30 metres, he again turned
(a) North-east (b) North-west
towards right and walked 20 metres. He then turned
(c) South-east (d) South-west
left and moving a distance of 1Om turned to his left
2. Muong a college student is facing north-west. He and walked 40 metres, turns to the left and walks
turns 90° in the clockwise direction and then 135° in 5 metres. Finally he turns to his left. In which direction
the anticlockwise direction. Which direction is he is he walking now?
facing now? (a) North (b) South
(a) East (b) West (c) East (d) South-West
(c) North (d) South
7. A rat rwns 20' towards East and turns a right, runs
3. Rakesh starts walking straight towards east. After 1O' and turns to right, runs 9' and again turns to left,
walking 75 metres, he turns to the left and walks 25 runs 5' and then turns to left, runs 12' and finally
metres straight. Again he turns to the left, walks a turns to left and runs 6'. Now, which direction is the
distance of 40 metres straight, again he turns to the rat facing?
left and walks a distance of 25 metres. How far is (a) East (b) West
he from the starting point? (c) North (d) South
(a) 25 metres (b) 50 metres 8. Shawna leaves from her home. She first walks
(c) 140 metres (d) none of these 30 metres in North-west direction and then 30 m in
4. Neelesh leaves for his office from his house. He walks South-West direction. Next, she walks 30 metres in
towards East. After moving a distance of 20 m, he South-east direction. Finally, she turns towards her
turns towards South and walks 1Om. Then he walks house. In which direction is she moving?
35 mtowards the West and further 5 m towards the (a) North-east (b) North-west
North. He then turns towards East and walks 15 m. (c) South-east (d) South-west
What is the straight distance in metres between his 9. I am facing South, I turn right and walk 20 m. Then
initial and final positions? I turn right again and walk 1. 0 m. Then I turn left and
(a) 0 walk 10 m and then turning right walk 20 m. Then
(b) 5 I turn right again and walk 60 m, In which direction
(c) 10 am I from the starting point?
(d) oannot be determined (a) North (b) North~west
(c) East (d) North-east
5. Vinod walks 20 metres towards North. He then turns
left and walks 40 metres. He again turns left and 1O. A man walks 1km towards East and then he turns to
walks 20 metres. Further, he moves 20 metres after South and walks 5 km. Again he turns to East and
twrning to the right. How far is he from his ortginal walks 2 km, after this he turns to North and walks
position? 9 km. Now, how far is he from his starting point?
180 I O Reasoning & Aptitude fflRDE ERS!::i
(a) 3 km (b) 4 km It then turns left and runs for another 25 kms and takes
(c) 5 km (d) 7 km the direction back to reach the main road. In the
meantime, due to a minor breakdown, the other bus
11. From his house Sanjay went iS kms to the North.
has run only 3G kms along the rnairi road. What would
Then he turned West and covered 10 kms. Then, he
be the distance between the two buses at this point?
turned South and covered 5 kms. Finally, turning to
(a) 65 kms (b) 75 kms
East, he covered 10 kms. In which direction is he
(c) 80 kms (d) 85 kms
from his house?
(a) East (b) West 17. X and Y start moving towards each other from two
(c) North (d) South places 200 m apart. After walking 60 m, Y turns left
artu goes 20 m, then he turns right and goes 40 m.
12. Going 50 m to the South of her house, Radhika turns
He then turns right again and comes back to the
left and goes another 20 m. Then, turning to the
road on which he had started walking. If X and Y
North, she goes 30 m and then starts walking to her
walk with the same speed, what is the distance
house. In which direction is she walking now?
between them now?
(a) North-west (b) North
(a) 20 m (b) 30 m
(c) South-east (d) East
(c) 40 m (d) 50 m
13. Michael walks 20 m North. Then he turns right and
18. Five boys are standing in a row facing East. Deepak
walks 30 m. Then he turns right and walks 35 m.
is to the left of Sarneer, Tushar and Shailendra.
Then he turns left and walks 15 m. Then he again
Sameer. Tushar and Shailendra are to the left of
turns left and walks 15 m. In which direction and
Sushil. Shailendra is between Sammer and Tushar.
how many metres away is he from his original
If Tushar is fourth from the left, how far is Sameer
position? from the right?
(a) 15 metres west (b) 30 metres east (a) First (b) Second
(c) 30 metres west (d) 45 metres east (c) Third (d) Fourth
14. A child is looking for his father. He went 90 metres 19. After walking 6 km, I turned right and covered a
is the East before turning to his right. HP. went distance or 2 km, then turned left and covered a
20 metres before turning to his right again to look distance of 1 O km. In the end, I was moving towards
for his father at his uncle's place 30 metres from the north. From which direction did I start my journey?
this point. His father wRs not there. From here he (a) 1~url~1 (b) South
went 100 metres to the North before meeting his (c) East (d) West
father in a street. How far did the son meet his father
from the starting point?
20. A postman was returning to the post office which
was in front of him to the north. When the post office
(a) 80 metres (b) 100 metres
was 100 metres away from him, he turned to the left
(c) 140 metres (d) 260 metres
and moved 50 metres to deliver the last letter at
15. The door of Aditya's house faces the East. From the Shantivilla. He then moved in the same direction for
back side of his house, he walks straight 50 metres, 40 metres, turned to his right and moved 100 metres.
then turns to the right and walks 50 metres again. How many metres was he away from the post office.
Finally he turns towards left and stops after walking (a) O (b) 90
25 metres. Now, Aditya is in which direction from (c) 150 (d) 100
the starting point?
21. A boy rode his bicyle northwards, then turned left
(a) South-east (b) North-east
and rode one km and again turned left and rode
(c) South-west (d) North-west
2 km. He found himself exactly one km west of his
16. Two buses start from the opposite points of a main starting point. How far did he ride northwards initially?
road, 150 kms apart. The first bus runs for 25 kms (a) 1 km (b) 2 km
straight and takes a right turn and then runs for 15 kms. (c) 3 km (d) 5 km
fflRDE ERS!::11 ® Direction Sense Test I 181
Direction (Qs. 22 to 26): Read the following information 28. A starts crossing the field diagonally. After walking
carefully and answer the questions given below it: half the distance, he turns right, walks some distance
(i) There are six flat on a floor in two rows facing and turns left. Which direction is A facing now?
North and South are allotted to P, Q, R, S, T (a) North-east (b) North-west
and U. (c) North (d) South-east
(ii) 0 gets a North facing flat and is not next to S. 29. From the original position given in the above figure,
(iii) Sand U get diagonally opposite flats A and B move one arm length clockwise and then
(iv) R, next to U, gets a South facing flat and T gets cross over to the corner diagonally opposite; C and
a North facing flat D move one arm length anti-clockwise and cross
over the corner diagonally opposite. The original
22. Which of the following combinations get South facing
configuration ADBC has now changed to
flats?
(a) CBDA (b) BDAC
(a) OTS (b) UTP
(c) DACB (d) ABCD
(c) URP (d) Data inadequate
23. Whose flat is between O and S? 30. From the original position, Band D move one and a
(a) T (b) U half length of sides clockwise and anticlockwise
(c) R (d) T respectively. Which one of the following statements
is true?
24. If the flats of T and Pare interchanged, whose flat (a) B and D are both at the midpoint between A
will be next to that of U? and C
(a) P (b) 0 (b) Dis at the midpoint between A and C, and 8 at
(c) R (d) T the corner originally occupied by A.
25. The flats of which of the other pairs than SU is (c) 8 is at the midpoint between A and C, and D at
diagonally opposite to each other? the corner originally occupied by A.
(aj OP (b) QR (d) B and D are both at the midpoint between A
(c) PT (d) TS and D.
26. To arrive at the answers to the above questions, 31. From the positions in original figure, C and A move
which of the following statements can be dispensed diagonally to opposite corners and then one side
with? each clockwise and anticlockwise respectively. 8
(a) None (b) (i)only and D move two sides each clockwise and
(c) (ii) only (d) (iii) only anticlockwise respectively. Where is A now?
(a) At the north-west corner
27. AnJu started walking positioning her back towards
the sun. After sometime, she turned left, then turned (b) At the north-east corner
right and then towards the left again. In which (c) At the south-east corner
direction is she going now? (d) At the south-west corner
(a) North or South (b) East or West 32. After the movements given in 0.31 above, who is at
(c) North m West (d) South or West the north-west corner?
Direction (Os. 28 to 32): The following questions are (a) A (b) 8
based on the diagram given below showing four persons (c) C (d) None of these
stationed at the four corners of a square piece of plat as 33. A square field ABCD of side 90 mis so located that
shown. its diagonal AC is from north to south and the corner
C A B is to the west of D. Muong and Thames start
E
N+S
walking along the sides from B and C respectively
in the clockwise and anticlockwise direction with
speeds of 8 m/s and 1Om/s. Where shall they cross
each other the second time?
(a) On AD at a distance of 30 m from A
w
B D (b) On BC at a distance of 10 m from B
i 82 I 'ill Reasoning & Aptitude fflRDE ERS~
(c) On AD at a distance of 30 m from D 3. (d) The movement of Rakesh are as shown in Fig.
(d) On BC at a distance of 10 m from C
N
Solutions
1. (d) As shown in Fig. 1, Dalbir initially faces in the
direction OA, On moving 135° anticlockwise, he
faces in the direction OB. On further moving
180° clockwise, he faces in the direction OC,
which is South-west.
D 40m C
25m 25m
SW
s A E B
- - - - 75 m - - - -
Clearly, EB = DC = 40 m.
:. Rakesh's distance from the starting point
A= (AB - EB) = (75 - 40) rn = 35 m.
N
A
15m
..-------F 10m
D 35m
s
Clearly, DC =-AB + EF.
p~ /
Q~ :. F is in line with A
135° _,,,......
Also, AF = (BC - DE) ::: 5 m.
5, (d) The movements of Vinod are as shown in Fig. 7. (c) The movements of the rat from A to G are as
shown in Fig.
N
N
R 40 m Q
20 m 20 m
A ----':--~-~l~o Gj
sf ~ r6
x---. . . ······························ P
20 m S
E 12 F
Clearly, it is finally walking in the direction FG
Clearly; Vi nod's distance from his initial position
i.e, North
P ⇒ PX = (PS + SX)
=(OR+ SX) 8. (a) The movements of Bhawna are as shown in Fig.
== (40 + 20) m = 60 m. (A to B, B to C, C to 0, D to A)
Clearly, she is finally moving in the direction DA
6. (a) The movement of Kuldeep are as shown in Fig.
i.e, North-east
from A to G.
N
NW NE
~, ;fl!
N ''
' , ,,
',, ,11
' ,
W ~---;:'i.-':--__.._
, '
E
,,," ',,,
,, ''
;:/ ''-4.
SW SE
s
B
D 10m C
D
20m
9. (d) The movements of the person are from A to F,
as shown Fig. Clearly, the final position is F
40 m
G
~-----A
B
30m which is to the North-east for the starting point A
N
NE
,f
,
,, ,
E F ,
Sm ,,
w----+--/___ E /
E 60m F 10 km
C ~
B
20m
10m 5km
.____.--.c
D
iO m
.__ ___,y,_--~E~E
D iOkm ~
B 20m A
N
NW
~,
',
''
'' '
s W + - - - - ''' + - - - - - E
30m
Clearly, OF = BC = 5 km.
EF = (DE - OF)= (9 - 5) km= 4 km.
B 20m C
BF= CD= 2 km
Clearly, she is finally moving in the direction DA
AF= AB+ BF= AB+ CD= (i + 2) km= 3 km
i.e. North-west.
:. Man's distance from starting point A
13. (d) The movement of Michael from A to F are as
= AE = ✓ AF 2 + EF 2 = ✓3 2 + 4 2
shown in Fig.
= J?_s = 5 km
t. . . . . •. . . +·· ►
B 30m C
/
/
PA, AB, BC, CD
/
Now, AD == BC = 25 km
/
/
/
w----✓ ---E
Bus Y travels 35 km upto E
Distance between two buses
== PO - (PD + OE)
s == [ 150 - ( 50 + 35)] = 65 km
, E
~ ~
E
0
0
~
,/// ;o m
8
P@
25 km
Ii>
15 km
t. ._.......,_t ~
A
- fl,
D
<II
35 km
@Q
B 25 km C
A 60 m ..__ _. 20 m
I " ~
:. Distance between X and Y
= ,;60" + 80" = 100 m = AD = (100 - 60) m = 40 m.
'
''
vv----''+----+E
''
f' Ill
t ►
f 20 k~
C 40m B
s South [g] ~ 0
20. (b) Clearly, the route of the postman is as shown. @]IT][}]
So, at the final point the distance of postman
So, the arrangement is:
from post office
= PD = AC = AB + BC South facing flats [Q] ffi] [!]
= 50 + 40 = 90 m
D ••••• •••• •• •• •• •••••••••• Post Office North facing flats [§] QJ IT]
100m 100 m 22. (c) The South facing flats are U, R, P
B
C i......~-r--l---.t1--'A 23. (a) T's flat is between O and S
40m 50m
~ - - - - - - - - - - - - 8 " " " 1,antrvm-------------24.-(e)--fhe-fhat-n-extio-tfs--ftahs--of-Ft,--whidnematns--
21. (b) Clearly, the boy rode from A to B, to C and finally u n changed if the flats of T and P are
upto D. Since D lies to the west of A, so required interct i&nyeu
distance
25. (a) The diagonally opposite pairs are SU and OP
== AB= CD= 2 km
1 km 26. (ct) Clemly, all the statements are necessary to
C--~-B
answer the given questions
Answer 22 to 26
0 gets a North~facing flat and is not next to S means
I✓
South
(i) (ii)
1
-1
w-----=--►E A
,' ,
/
/
/
/
It,,:/
SW
s D
29. (a) Clearly, (i), (ii), (iii) and (iv) show the movements
of A, B, C, and D respectively while the new
arrangement so obtained is shown in (v). So,
the congiruration changes to C8DA
It is given that
Muong's speed == 8 m/s
Thames's speed .:c i O m/s
It is clearly visible from the above figure that to
meet for the first time they have to cover a
30. (a) The movements of 8 and Dare clearly shown in distance of 270 m i.e three arms length of the
the adjoining diagram field.
So, statements (a) is true. BA + AD + CD = 90 + 90 + 90 :::;: 270 m
Since they are running in clockwise and
' ... B....J? -- .,
anticlockwise direction respectively, they have
IOI
El . b
to cover a distance of 360 m (I.e. 4 arms length
of the field) for their second meeting from the
first meeting point.
Thus, total distance to be covered for their
31. (d) The movements of A, C, B, and Dare shown ln
second meeting ti: 270 + 360 = 630 m
figures (i), (ii), (iii) and (iv) respectively. The final
configuration is shown in (v). Comparing (v) with . . 630 630
Time required = -..• -
.. -·· = _
... ·-
the given diagram, A is in the south~west corner. 8 + 10 i8
E = 35 seconds
NE SE
'lo::, /
'/(
Distance covererd by Muong in clockwise
'' /
', /
/
direction in 35 seconds == 35 x 8 = 280 m
' ', ,/
' / Distance covered by Thames in anticlockwise
N ---.-;,'1,-',---S
, '' direction in 35 seconds
', //
/
[SJ
(i)
:.:J □ LJ
♦--· l_. ,,/
··
(ii)
B
(iii) (iv)
D
"DB
C
(v)
A
l!!lll!iUUIIII
(Final Position)
..•1•l
Data Analysis Strategy-i: Doing all easy questions from all
the given sets (that ask for fast reading, wide
Data analysis is a process of collection, presentation
and quick eye span and efficient data ;
and interpretation of the information contained in the comprehension from all sets).
data to aid to reach some kind of conclusions. Usually Strategy-2: Doing all the questions from
data collection and presentation is further coupled with selective and relatively easier DI sets where
mathematical and analytical tools to react on further data compherension is far more easier.
conclusion. Visual representation is the best 5. Basic command over elementary mathematical
representation of data because of ease of calculation tool. Which comprises of following:
comprehension. Visual representation data can be • Percentage and equivalent fractional forms
.. Averages
primarily represented into following forms.
.. Ratios
1. Line-graphs/cartesian graphs
• Approximation
2. Tables
3. Bar diagrams
4 _ caselets Data Interpretation
6. Some other non-conventional miscellaneous Executive member, managers and personnel at different
- - - - - - - - " - f § t t f ' · " "----------------~ffilBLlnlbruiatirn..LS.oLgaoizatlons_are..x.eq.uke-t0-anal:yse
Skills: That individual need to, to handle DI (Data different sets of datas. Balance sheet, Records of
Annual General Meeting, Quarterly Report etc. requires
Interpretation) efficienlly fr urn exmnpoir1t of view.
interpretation of various data types. On the basis of the
1. Data Comprension Skills: Ability to select given sets of datas future projections can be made and
relevant information from any given set of data corrective action can be taken at regular intervals.
in context of question be asked, based upon Mainly there are following sets to represent datas.
that data set. .. Graph
2. Ability to differentiate between easy & • Table
difficult questions: It has many aspect for • Bar Diagrams
example, how many question in data set • Pie Charts
requires actual calculation and how many can • Miscellaneous Figures etc.
be answered by observation which are based
an patterns. How many questions which are Line Graph
having options can't not be determined or data Line graph is the simplest way to represent data. Single
insufficient and out of such actually for how set or multiple sets of datas can be shown in a graph.
many question required information is not Normally following things are required to analyse.
available. e Increase in profit in absolute terms
e Increase in profit in percentage term
3. Ability to apply approximation techniques: e Growth rate of the given duration
That is based upon the scenario that how far e Average annual growth rate
the options are value wise. • Average profit
4. Last but not the least: In entire DI section which .. Capacity utilisation \1
strategy can he!p me to maximize my attempts ..- ctvc of1 I'""'"'
11 1 C"""
In 1 auv Tr'"'d'"'
1 1a, v n,..,f;,..,;;
uc::IIvrL ctr~~,,.J ·-~de S· "P'""
tu 11 ct u1 1uv ,1
;_:,_J
Advantage: Advantage with line graph is that it 4. Average Annual Growth Rate
gives an ideal about the quantity expressed in this Increase in profit for the duration
graph between the given time period as well. =--- --------
Base years profit
Disadvantage: Usually exact values are not
X------
mo
available, when we try to represent exact value on ·
Number of years
the line graph, incase of multiples line representation
In the above example
graph become complex.
500-150 iOO
Lets have an example of graph ---x-
150 3
.500 500
== 350 X 100 = 66 _66 %
ui
a: 400 150 3
._:
u 300 A f Sum of profits of the duration
.£
-
e(l_
200
5. verage pro ,t = Total number of years
100 150
t ⇒ 150+250+480+500 = 1380 = 345.
4 4
2001 2002 2003 2004 ~ years
Capacity Utilisation
Fig. (I) Balance Sheet of ABC corporation.
:~ ~
320+--....----.--.;-.--.--....-----
Jan. Feb. Mar. April. May. June. July. \.
manufacturing) divided by the total consumption of
Metal (for car manufacturing) over the period will
give a ratio closest to
\ (a) 4: 3 (b) 5 : 4
(c) 6 : 5 (d) 7: 4
1 . Which month showed the highest absolute difference
in the Consumer Price Index (CPI) over the previous 8. Which item and for which year shows the highest
month?
percentage change in consumption over the
(a) March (b) Aprii
previous year?
(c) May (d) July
(a) Metal 2003 (b) Plastic 2003
2. Which month showed the highest percentage (c) Metal 2002 (d) Plastic 2005
difference in the CPI over the previous month?
(a) March (b) April 9. Forthetwodataseriesshown,howmanyyearshave
(c) May (d) July shown decrease in consumption over previous year
,-----~foLb.o.tbJhe-it@l+l.$-r--.- - - - - - - - - - - ~ ·
&-Forhnwma:nvmontrrwastne CPf greater tnans-STI?
(a) One (b) Two
(a) One (b) Two
(c) Three (d) Four (c) Three (d) Four
4. In how many months was there a decrease in the 10. Which year showed the highest percentage
CPI over the previous month? decrease in the total consumption of the two?
(a) One (b) Two (a) 2001 (b) 2002
(d) Four (c) 2004 (d) 2005
5. The difference in the number of months in which Direction (Qs. 11 to 15): Study the following graph to
there was an increase in the CPI and the number of answer the given questions:
months in which there was a decrease was
Percent profit earned by two companies over the
(a) One (b) Two
given years:
(c) Three (d) Four
70 -+-Company 8
Direction (Qs.6 to Q.10) : Study the following graph to -+-Company A
answer the given questions: 60
_ 50
30 'E
25 o. 40
..... Metal c
20 ~ 30
15 oi
-0-Platic 11.. 20
10
5 10 ··············•······································
0+-=-.----i---.--....----....
2000 2001 2002 2003 2004 2005
0--..-----..----,--------.-----,..- I
I
1997 1998 1999 2000 2001 2002
® Line Graph I 19i
i 1. If the expenditure of Company B in year 2000 was 17. In which of the following years was the difference
Rs. 200 crore, what was its income? between the imports made by Company 8 and C
(a) Rs. 240 crore the maximum?
(b) Rs. 220 crore (a) 1995 (b) 1994
(c) Rs. 160 crore (c) 1991 (d) 1992
(d) Cannot be determined
i 8. in which of the following years was the imports made
12. If the income of Company A in year 2002 was Rs. 600 by Company A exactly half of the total imports
crore, what was its expenditure? made by Company B and C together in that year?
(a) Rs. 360 crore (a) 1992 only (b) '1993 only
(b) Rs. 480 crore (c) 1992 and 1993 (d) 1995 only
(c) Rs. 375 crore
(d) Can't be determined i 9. What was the percentage increase in imports of
Company B from 1992 to 1993?
13. If the income of Company B in 1998 was Rs. 200 (a) 10 (b) 25
crorers, what was its profit in 1999? (c) 40 (d) 20
(a) Rs. 21.5 crore
(b) Rs. 153 crore 20. In which of the following years was the total imports
(c) Rs. 46.15 crore made of all the three companies together the
(d) Can't be determined maximum?
(a) 1996only (b) 1997 only
14. If the income of the two companies in 1998 were
(c) 1995only (d) 1995&1997only
equal, what was the ratio of their expenditures?
(a) 1 : 2 Direction (Os. 21 to 24): The line graph gives the
(b) 26: 27 number of students in a Engg. college's M.Tech program
(c) 100: 67 integrated (IMP) for the years 2008 & 2009. The program
(d) Can't be determined in takes the student in only 1st year & students can leave
15. What is the percent increase in percent profit for college only if they complete 6-years program. Every
company 8 from year 2000 to 2001? year the students who pass the annual exam of a
(a) 75 (b) 175 particular year are promoted to the next class while
(c) 42.86 (d) Can't be determined students who fail, have to study in the same class next
year also.
Direction (Qs. 16 to 20): Study the following graph
carefully and answer the questions given below it.
100 100
(/)
0
ci
100 ··•················•··•••·•··········•··•··•·····•·•····· z 72
so i-------~"""5~~$~r
60 .• . ... IMP IMP IMP IMP IMP IMP
st nd rd th th th
I year 2 year 3 year 4 year 5 year 6 year
40
X-2008
20 ···············l+A+B-+C 1..................... @ 2009
1991 1992 1993 1994 1995 1996 1997 Imp: It is known that 76 students passed out of
final (6 th year) class of IMP in the year 2008.
16. In which of the following years, the imports made
by Company A was exactly equal to its own average 21. How many fresh students joined the program (IMP)
imports? in the year 2009?
(a) 1992 (b) 1993 (a) 94 (b) 92
(c) 1994 (d) None of these (c) 90 (d) 88
192 I • Reasoning & Aptitude fflAOE ERS~
22. In how many classes did exactly tour students fail 11. (a) Income of Company Bin 2000
in year 2008? 120
(a) 5 (b) 3 = 200 x == Rs. 240 er.
100
(c) 4 (d) 2
12. (c) Expenditure of Company A in 2002
23. How many students in total failed in the entire 100
program of six years together in year 2008? = 600 x = Rs. 375 er.
160
(a) 38 (b) 36
13. (d) We can find outthe amount of profit in i998, we
(c) 32 (d) 34
do not know the income and expenditure of A
24. In which year pass percentage was the heighest in and B therefore option (d) is the correct choice.
year2008'?
14. (b) Ratio of their expenditures
(a) Vth year (b) v1 th year
(c) IVth year (d) lll rd year
100 BO
:::: 135 X 100 = 26. : 27.
25. How many total student failed together in entire
program in 2009? 35-20
15. (d) Reqd. % increase= - - - x 100 == 75%.
(a) 32 20
(b) 34 16. (d) Average imports made by company A
(c) 38 30+50+60+40+ 70+60+ 75
(d) Can't be determined 7
26. How many total no. of students passed in (IMP) in 385
= 7 =55
all 6 years combined in year 2008?
(a) 492 In none of the given years the impmts is exactly
(b) 484 equal to 55 (crore). Hence. the answer is (d)
(c) 462 17. (d) By visual inspection it is clear that i992 is the
--------1eJ}--6an'-t-be-determtne desired year (as the distance between two
points is the maximum in 1992).
□□□□ 18. (a) By observation, in year 1992
( as 50 == 40 ; 60)
Solutions
Answer 1 to 5: So 1992 is the desired year. We do not need
any calculation see the year where the point A
1. (b)
lies exactly in the middle of points of B and C
2. (b) Visually clear that it is April
50 40
3. (b) April and July - Two. l 9. (b) Reqd percentage increase =
40
4. (b) The CPI drecreased in March and May = 25%
5. (a) The CPI increased in three months 20. (c) The total imports (in crore) made by all the three•
(April , Juno and July) while it decreased in two companies together : horn the height of the;
months (March and May). points we observe that the total heights of three)
6. (d) Visully seen as 4. 2000, 2001, 2004 & 2005. points is the maximum either in 1995 or 1997. lfj
you observe carefully our clear answer is 1995,;I
7. (b) 145 : 115 = 29 : 23 is closest to 5 : 4
but to be sure we find actual values for the two)
:)
8. (c) Metals in 2002 is more than double over it's ~
years. .•1
,;1
value in 2001. ,1
In 1995 = 70 + 80 + 85 = 235
9. (a) 2001 is the only year which satisfy the condition. In i997 = 75 + 70 + 85 = 230 l
1
i 0. (a) 33.33% between 200 to 2001. Clearly, 1995 is the desired year. ;J
;1
t
fflRDE ERS~ ® Line Graph I 193
Solution 21 to 26
If 76 students passedout of 6th year in 2008 out of 90 students, it means 14 students who are failed in 2008, they will
stay back in 6th year in 2009 also. It means in 2009, out of 98 students, 14 students are those who failed in 2008, in
6th year, and rest 98 - 14 = 84 students are those, who passed in 5th year in 2008 & being promoted to 6th year in
2009.
On the basis of same reasoning we can draw following table.
2008 2009
No. of students
No. of students No. of students No. of students No. of students
promted from
in class passed failed in class
last year class
99- 6 = 88
1st year 84 78 6 94
fresh students
2nd year 72 68 4 82 82 4 = 78
3rd year 100 96 4 72 72 - 4 = 68
4th year 94 90 4 100 100 - 4 = 96
5th year 86 84 2 92 92 - 2 = 90
6th year 90 76 14 98 98 - 14 = 84
22. (b) In each of classes 2nd , 3rd , 4 th year of 2008 four students failed.
24. (a) From table directly pass percentage is highest in the 3 rd year in 2008.
25. (d) Can't be determined, because no data about total number of students passed in 2009 is available.
26. (a) From table, number of students passed in all six years combined in 2008 = 492,
WI
~
Practice Exercise: I
Direction (Qs. 6 to 9): The table given below shows
production of five types of cars by a company in the
years 1989 to 1994. Study the table and answer
Direction (Qs. 1 to 5): Study the following Table carefully questions.
and answer the questions given below
Production of main crops in India (in million tonnes) Production of cars by a comp_fillY_____.___ _
Crops -~~-9L92':9L93..,Q4-"8+9S---95-96~- 9e:97 --~ ----Ye·;~~--19;9-~ 99;--~;;~-~ 992 1993 1994 Total
------··--· ~Pulses·· - 20.5 22.4 24.6 23.5 27.8 28.2 Type ,i,
OIiseeds 32,4 34.6 40.8 42.4 46,8 52.4 p 8 20 16 17 6 88
21
Rice 80.5 86.4 88.2 92.6 94.2 90.8
0 16 10 14 12 12 14 78
Sugarcane 140.8 150.2 152.2 160.3 156.4 172.5
R 21 17 16 15 13 8 90
Wheat 130.2 138.4 146.8 141.6 152,2 158.4
s 4 6 10 16 20 31 87
Coarse grain 45.6 52.8 60.4 62.2 58.2 62,8
T 25 18 19 30 14 ?_7 133
Total 450 484.8 513.2 522.8 535 6 565 1
Total 74 71 75 90 80 86 476
1. Production of sugarcane in 1993-94 was
6. In which year the total production of cars of types P
approximately what percentage of production of rice
and Q together was equal to the total production of
in 1992-93?
cars of types R and S together?
(a) 50 (b) 75
(c) 150 (d) 175 (a) 1990 (b) 1991
(c) 1994 (d) None of these
2. Production of which type of crop was continuously
increasing in the given years? 7. During the period i 989-94, in which type of cars
(a) Rice (b) Pulse was a continuous increase in production'?
(c) Sugarcane (d) Oilseeds (a) P (b) 0
(c) R (d) S
3. What was the average production of pulse in the
given years? 8. The production of which type of cars was 25% of
(a) 26.8 million tonnes the total production of all types of cars during 1993?
(b) 20.5 million tonnes (a) S (b) R
(c) 24.5 million tonnes (c) Q (d) P
(d) 22.5 million tonnes
I
inADE EASY @ Table I 195
9. The per cent increase in total production of all types A Country's Foreign Trade
of cars in 1992 to total in 1991 was? (Rupees in Crores)
(a) 15 (b) 20 Year Exports Imports Trade Deficit
(c) 25 (d) 30 1990-91 6711 12549 5838
1991-92 7806 13608 5802
Direction (Qs. 10 to i 4 ): The following five questions
1992-93 8803 14293 5490
are to be answered on the basis of the following table:
1993-94 9771 15831 6060
Weight Distribution in the Average Adult
1994-95 11855 17173 5318
Organs Weight 1995-96 10420 18371 7951
(in grams) 1996-97 12550 20063 7513
Muscles 30,000
15. Which of the following showed an increase every
Skeleton 10,000 year?
Blood 5,000 (a) Exports (b) Imports
2,000 (c) Trade deficit (d) All of these
Gastrointestinal Tract
Lungs 1,000 16. The ratio of imports to exports was maximum in the
year
Liver 1,700
(a) 1990-91 (b) 1996-97
Brain 1,500
(c) 1995-96 (d) 1992-93
1O. The total body weight of the average adult is
17. The percentage increase in exports over previous
(a) 70,000 grams
year was maximum in the year
(b) More than 51 kg
(a) 1990-91 (b) 1996-97
(c) 50,000 grams (c) 1994-95 (d) 1993-94
(d) Less than 50 kg
18. The total trade deficit for the last five years?
11. If the weight of the skeleton is represented as S, (a) Rs 28,508 crore (b) Rs. 32,332 crore
then the weight of the liver can be represented as (c) Rs. 44,322 crore (d) Rs. 33,232 crore
(a) 1.7S (b) 0.178
(c) 17S (d) 71S 19. The difference between imports and exports was
maximum in the year
12. The ratio expressed in decimals of the weight of the (a) 1995-96 (b) 1996-97
blood to the weight of the gastrointestinal tract is (c) 1994-95 (d) 1993-94
(a) 0.4 (b) 4.0
(c) 2.5 (d) 0.25 Direction (Qs. 20 to 25): Information given below shows
% of students, who applied for selection in a company.
13. The ratio expressed in decimal for weight of the brain For final selection one has to pass through five major
to the weight of the mucles is stages written test, aptitude test, group task, group
(a) 0.50 (b) 0.15 discussion, interview & a medical checkup in the same
(c) 0.20 (d) 0.005 order. At every stage only those candidates who fulfill
the required standards are qualified for next stage.
14. The ratio decimal of the weight of the brain to the
Table below given the region wise details.
weight of the lungs is
(a) 1.5 (b) 0.15 Task Written Aptitude Group Group Inter Medical
Region test Task Discussion view
(c) 15.0 (d) 5. i West 70% 90% 85% 50% 90% 90%
North 68% 88% 90% 60% 85% 90%
Direction (Qs. 15 to 19): The figures for a country's South 80% 85% 80% 70% 70% 80%
North-east 75% 95% 78% 40% 65% 85%
Foreign Trade for the years i 990-9i to i 996-97 are given Central 70% 80% 68% 50% 60% 70%
test & 80% of those have taken aptitude are qualified 2. (d)
for group test and so on. 3. (c) Average production of pulse
20.5+ 22.4+ 24.6+23.5+ 27.8+ 28.2
20. If the candidate appearing for written test from =
6
Northern & southern region are in mtio of 4: 5 what
is the approximate ratio of people who will appeared 147.0
= - - = 2 4 .5 m1·11·1ontonne.
6
for final interview from these region respectively,
32
(a) 13: 19 (b) 19: 13 4. (a) Required percentage= .4 x 100
450
(c) 16:19 (d) 19:16 = 7.2%
21. If equal number of candidates are called for group 5. (b) Total production of oilseeds in the given years
task from each region, then find the region frorn ""42.4 + 46.0 + 52.4 = 141.6
where maximum candidate. Participated in selection Which is equal to the production of wheat in
process. 1994-95.
(a) North (b) $outh 6. (d) In year be 1993
(c) West (d) Central P + Q = 33 = R + S
7. (d)
22. If equal no. of candidates are called for group task from 8. (al 25% of 80 = 20 = production of car S in 1993
each region, then find the region from where minimum 9. (b) Required per cent increase
candidate will be able to clear their medical test 90-75
(a) Northeast (b) West = X 100 = 20%
75
(c) Central (d) None of these 10. (b) 51200 gm= 51 kg and 200 gms
23. If candidates from South & North who passed the
11 . (b) S = 10000
:. Weight of Liver = 1700
group discussion are in ratio 9 : 8 and finally from
South 1000 candidates clear the medical checkup = _S_ X 1700 = 0.17 S
then approx. How many candidates from North have 10000
gle9red their medical check11r;2?_ _ --s--·-·····
12. (c)
(a) 1300 (b) 1200 2 = 2.5
(c) 1400 (d) 1600
5
24. If same number of candidates appeared for written
13. (d) i 00 = - 1 = 0.05
30000 20
test from all regions then from which region final
selection % is best 1500 3
i4(a)
. . , - -=-=
1000 2 ii:
.. ;
~.
(a) South (b) West
(c) North (d) Central 15. (b)
25. If finally same number of candidates have been 16. (a) The ratio of imports to exports in the year
selected from each region, then from which region 12549
1990-9i = = i ,87
success rate was best in % terms 6711
(a) South (b) West 20063
1996-97 = =1.60
(c) North (d) Central 12550
78311
DOD □ 1995-96 = =i .76
10420
14293
1992-93 = =1.62
8803
Solutions
Answer 1 to 5: 17. (c) Percentage increase in exports in
152 2 i 2520 -10420
1. (d) Required per cent = · x 100 ,,, 175%. 199R-97
. , · - · . -- - - - - - X ➔I UU
nn -- 2"'
U. •501
I 10
86.4 10420
IDRDE ERS'!:::1 ® Table I 197
i8. (b) 5490 + 6060 + 5318 + 7951 + 7513 23. (b) Say number of candidates from south who
= Rs. 32332 crore. cleared group discussion is 9x, the number of
candidate who cleared medical test.
19. (a) The difference between the imports in
= 9x X 0.7 X 0.8 = 1000
1995-96 = Rs. 7951 crore
1996-97 = Rs. 7513 crore 1000
X:::
1997-98 = Rs. 5318 crore 9 X 0.56
1993-97 = Rs. 6060 crore No. of candidates from North who cleared
20. (a) No. of students from northern region= 4x medical test
No. of students appeared for find interview 1000
= 4x x 0.68 x 0.88 x 0.9 x 0.6 = 1.2925x = 8x---x0.85x0.9
9 x0.56
Similarly no. of students for final interview
= 1214 approximately option (b).
from southern region
= 5x x 0.8 x 0.85 x 0.80 x 0.70 = 1.904x 24. (c) From table it is clear final multiplication factor is
Hence required ratio= 13: i9. Option (a) highest corresponding to North.
Hence option (c).
21. (d) Suppose number of candidates from west be
equal to x. Sox x 0.7 x 0.9 = 0.63x candidate 25. (c) North, hence option (c).
will appear for group test. Hence lower the
ll!IIIIIUllll!lil
Bar Diagrams ® Maximum increase in profit was observed in
financial year 2001-02.
There are mainly six types of Bar Diagrams.
® Minimum increase in profit was observed in
11 Simple Bar Diagram
11 Multiple Bar Diagram financial year 2003-04.
11 Compound Bar Diagram ' 120+200+250+i80
2. Average pro f,t =
11 Percent Bar Diagram 4
11 Horizontal Bar Diagram 750
= ~--- = 187.5 crore
11 Floating Bar Diagram, etc. 4
Advantage 3. Annual average growth rate from 2001-2004
Can represent many different categories. We can directly 180-120 100
= ----x-
compare multiple categories. 120 3
01 02 03 04 --+ Years
~ Software Section
Fig. (1) Balance Sheet of MTS incorporation D Hardware Section
Sol.: From the Bar Diagram given in Fig. ( 1) We can ffilillj Consultancy
deterrnine following things
1. Percent increment in profits
Fig. (2) Balance Sheet of Megha-soft for two
2001 -02 ⇒ SO x100 = 66.66% financial years 2005 and 2006.
120
From the multiple bar diagram shown in Fig. (2) we can
2002-03 ⇒ SO x100 = 25% calculate following things
200
_7()
1. Sectional Growth
2003-04 ⇒ .......'.....:'x 100 = -28% (a) Software section i80-150 == 30 Cr.
250
(b) Hardware section 400 -- 350 ""30 Cr.
(- Sign indicates decrement)
fflRDE EASY ® Bar Diagram I 199
Consultancy ⇒ 26
~~ioo x 100 == 30%
percent of export of Automobile product of
Japan?
• Minimum growth is observed by Hardware Sol. Export of Agriculture product of UK
section = $ 50 billion
• Maximum growth is observed by Export of Automobile product of Japan
consultancy section. = $ 200 billion
3. Growth rate of megha soft for the duration 50
= x100 = 25%
2005-06. 200
Ex. 2. Export of Automobile product of Japan is how
Profit in 2006 - Profit in 2005 x
100 many times of automobile product.
Profit in 2005
Sol. Automobile Export of Japan
⇒ (180+ 400+260)-(150+350+200) x
100 = $ 200 billion
(150 + 350 + 200) Automobile Export of UK = $60 billion
840 - 700 X 1QQ
260 10 .
⇒ 700 = = 3.33 times
60 3
_ 140x100 = 20 %
- 700 Ex.3. Total export given in Fig. of Japan is how much
percentage higher than that of UK?
Compound Bar Diagram
. Compound Bar Diagrams are similar to Multiple Bar Sol.
== 640 - 480 X
1OO
480
Diagrams. The only difference between these two Bar
Diagrams is that in Compound Bar Diagram a single
160
= Xi 00 = 33.33%
480
Bar Diagram is subdivided into different parts, while
multiple bars are used in Multiple Bar Diagrams. Percent Bar Diagram
Compound Bar Diagrams are also known as sub-divided Percent Bar Diagram is similar to Compound Bar
Bar Diagrams. Diagram. The only difference is that the total height of
each bars are equal and represent hundred percent in
percent bar diagram, whereas heights of different bars
in Billion Dollars
700 in compound bar diagram may or may not be equal.
640
600 480 1200
100
500 480 25% 25%
430
400 ... ,. 75
, ...
350 (f)
...
300 -- --- §
t;
... .... 35% 25%
50
200 .£
ui
20%
a:
100 25
30%
50
2001 2006
?00 I ® Reasoning & Aptitude fflRDE EASY
• Automobile
F7 Financial
LiJ t
~~~~j Electronics 2003 ll-,U.J.J./.Ji.J.1..1..J..J..J.U.U..u..i...........................
(/)
- Agriculture ro 2002
~ 1-1,.U:.J.J..l..u.L.IJ
3
oo - 72 = 3. i 6 times approx. 1989 1990 1991 1992 1993 1994 1995 1996
72
Horizontal Bar Diagrams
1. The average production of 1990 and 1991 was
Horizontal Bars are similar to simple bar diagrams. The
exactly equal to the average production of which of
only difference is that in case of horizontal bar,
the following pairs of years?
representation is horizontal, whereas vertical bars are (a) 1991 and 1992 (b) i992and 1994
used for simple bar diagram. Horizontal bar diagram is (c) 1993and i994 (d) None of these
used in cases when we requires to represent data sets
2. What was the difference in the production of :
for comparatively large number of years. All the
foodgrains between 1991 and 1994?
calculation and observation of both the Bar Diagrams
(a) 10000 tons (b) 15000 tons
are similar.
(c) 500 tons (d) 5000 tons
fflADE EASY ® Bar Diagram I 201
3. In which of the following years was the percentage 9. Approximately what percentage of the total rose
increase in production from the previous year the production is shared by the other States?
maximum among the given years?
(a) 12.5 (b) 17.5
(a) 1991 (b) 1993
(c) 19.5 (d) 22.5
(c) 1995 (d) i990
10. If total percentage contribution of the States having
4. In how many of the given years was the production
of foodgrains more than average production of the production of roses below twenty thousand is
given years? considered, which of the following statement is true?
(a) 2 (b) 3 (a) It is little above 40%
(c) 4 (d) i (b) It is approximatey 36.8%
(c) It is below 35%
5. What was the percentage drop in the production of
(d) It is little below 30%
foodgrains from 1991 to 1992?
(a) 15 (b) 20 Direction (Qs. 11 to 15): These questions are based
(c) 25 (d) 30
on the following bar graph. Read the graph and answer
Direction (Qs. 6 to 10): Study the following graph the questions
carefully and answer the questions given below:
Finances of XYZ Railway
Rose Production
25000
25,000
§Gross traffic ReceiptstiJTotal Expenditure
10000
20,000
8000
15,000
6000
10,000
4000
5,000 2000
6. Which of the following State(s) contribute(s) less than 1993-94 1994-95 1995-96 1996-97 1997-98
1O % in the total rose production?
(a) Only Rajasthan
(b) Rajasthan, Karnataka 11 . What is the percentage increase in the gross traffic
(c) Rajasthan, Karnataka, Haryana receipts in 1995-96 as compared to 1993-94?
(d) Rajasthan, Karnataka, Haryana and Gujarat (a) 33.9% (b) 29.3%
7. By what percentage rose production of other States (c) 20.7% (d) 17%
is more than that of the Maharashtra?
12. If profit = gross traffic receipts-total expenditure,
(a) 25 (b) 30
then in 1996-97 what percentage of gross traffic
(c) 20 (d) 15
receipts is the profit made?
8. What is the approximate average production of roses (a) 5.9% (b) 6.4%
(in thousands) across all the states? (c) 7.2% (d) 8%
(a) 21 (b) 20
i 3. In which year was the profit as a percentage of gross
(c) 19 (d) 18 traffic receipts the highest?
202 I 111 Reasoning & Aptitude
; 4. In order to make a profit of i0%, what should have 19. In which year, there was minimum percentage of
been the gross traffic receipts (in Rs. crore) in export with respect of production?
1994-95, total expenditure remaining the same? (a) 1991 (b) 1992
(a) 5667 (b) 5876 (c) 1993 (d) 1994
(c) 6444 (d) 7667
20. In which year we had maximum quantity of tea for
15. By what amount (in Rs. crore) has the expenditure domestic consumption
increased over the period 1993-94 to 1997-98? (a) 1994 (b) 1991
(a) 4100 (b) 3900 (c) 1993 (d) 1996
(c) 3850 (d) 3700
It is 36.8% approximately.
13. (d)
In 1992,
·mo x 100 = 33- %
1 (a) 1.33 (b) 0.75
540 3 (c) 0.56 (d) 1.77
288 2. In which of the states there is a steady increase in
In 1993 - x 100 = 40%
720 the prodution of cotton during the given period?
400 (a) A&B (b) A&C
In 1995, - x 100 = 66.66%
600 (c) B only (d) D & E
450
In 1996, - x 100 = 68.18% 3. How many tonnes of cotton was produced by state
660
E during the given period?
17. (b) 720-288 = 432 million= Total consumption of (a) 2900 (b) 29000
tea in 1993 (c) 290000 (d) 29000000
432
Population of India= = 1080 million. 4. How many states showing below average production
0.4
in 1992-93 showed above average poduction in
18. (d) Average tea exported during 1991--1996 = 1993-94?
1754 million kg. Average tea produced during (a) 4 (b) 2
1991-1996 = 3700million kg (c) 3 (d) 1
. 1754
Required ratio= = 0.47. 5. Which of the following statement is in false?
3700
(a) States A and E showed the same production in
19. (a) 1993-94
20. (c) (b) There was no improvement in the production of
In 1991, 480 96 = 384 million tonnes cotton in state B during 1994-95.
In 1993, 720 - 288 = 432 millin tonnes (c) State A has produced maximum cotton during
In 1994, 700-340 = 360 million tonnes the given period
In 1996, 660 -- 450 = 210 million tonnes (d) Production of States C and D together is equal
to that of state B during i 993-94
□□□□
204 I 0 Reasoning & Aptitude
Direction (Qs. 6 to 1O): Study the diagram carefully ahd Direction (Qs. 11 to 15): Study the following bar charts
am;wer the question glveh below it (Figs) before answering the questions;
lrnptJrts
A
~ bemand ~ Production tl
c:-t:..=·.............
D
E
I~#;#~;!.,;
J--1;$:~~~
K~:;:#;:~~~~-~..-,.-.-...,..,.,.,,
L 12699
l::xports
A B C D E
Companies
E
F
6. What is the ratio of companies having more demahd
than production to those having more production
than demand?
(a) 2: 3 (b) 4 : 1
(c) 2 : 2 (d) 3 : 2
7. What is the difference between average demand Fig. Foreign Trade by countrifls_.to..___ _--+--·
----....,.,.nd-average-productiOn-otth-e-itvFcompaFYtes-taKen·------ year 1993~94
gotether?
(a) 1400 (b) 400 11 . How n 1any countries exhibited a tra.de surplus?
(a) 5 (b) 4
(c) 280 (d) 138
~) 3 ~) 6
8. The production for comapny D is approximately how
12. The higest trade deficit was shown by which country?
many times that of production of company A?
(a) C (b) G
(a) 1.8 (b) 1.5
(c) G (d) L
(c) 2.5 (d) U i
13. ihe ratio of Exports to Imports was highest for which
9. The demand for company 8 is approximately what country?
percent of the demand for company C? (a) A (b) I
(a) 4 (b) 24 (C) J (d) K
(c) 20 (d) 60
14. The total trade/surplus for all the oouhtries put
1 o. If company A desires to meet the demand by together was?
purchasing i.V. sets from a single company, which (a) i 1286 surplus (b) 1 i 286 deficit
one of the following companies can meet the need (c) 10286 deficit (d) None of these
adequately?
15. The ratio of the maximum exports to the minimum
(a) 8 (b) C
imports was closest to
(c) D (d) None of these
(a) 64 (b) 69
I
(c) 74 (d) None of these
@II Bar Diagram I 205
Miscellaneous is = 1840.
(a) Rs. "18,500 crore (b) Rs 18,000 crore :. Difference in having average demand &
(c) Rs. 21,000 crore (d) Rs. 15,000 crore
production= (2120-1840) = 280.
i 9. Which country accounts for higher earning out of
8. (a) Let production of D
Services and Miscelianeous together?
= k x production of A
(a) India
(b) Pakistan 2700
Then, 2700 = k x - - = i .8.
(c) Both spend equal amounts 1500
(d) Cannot be determined 9. (b) Let x% of demand for C = Demand for B.
20. If the total GDP is the same for both the countries X
Then, x 2500 = 600
then what percentage is Pakistan's income through 100
agriculture more than India's income through services?
600-
orx= ( - X100)
- =. 240110
(a) 100% (b) 200% 2500
(c) 133.33% (d) None of these
10. (c) Since D produces highest number of sets and
A desire to meet the demand by purchasing
surplus sets from a single company,
□□□□
So, D can meet the demand of A.
206 I ® Reasoning & Aptitude fflRDE ERS!::!!
11 . (b) Out of a total of i 2 countries, 8 showed a deficit 17. (c) 20% of 10000 = 2000
while 4 showed a surplus.
18. (c) (40+20+ 10)%of30,000==Rs.21,0000crore.
12. (d) Visually it is clear that L has highest trade deficit.
13. (b) I has a ratio of 4002/27 44 = 1.45, which is. the 19. (d) Although the percentage on Service and
highest. [ Miscellaneous put together is equal for both the
countries, but we cannot comment on this since
14. (b) Sum of exp~rts-sum bt imports= deficit ( 11286) we have no data about the respective GDPs.
15. (b) 6045/87
20. (a) Since the GDP is same, the answer will be got 1·,·
or 1% =
Cs 8
)
~~ ~I -+ Industries (Secondary Sector)
or vice versa
Pie-Chart: 11
Pie-Chart: I
18%
✓
~➔ Urban Areas
g~ ~I ➔ Rural Area
27%
Demographic composition of Indian
55%
population living in Urban & Rural
areas according to census 2001.
Pie-Chart: 111
35%
Sectoral Conbribution to GOP in Indian Economy
(Financial Year 2008-09)
20% -
- - -- - - - H+++++++++>o.
- - - - =:::t:t:::::t:t:::::t!tl:t:U
1iJ;11Tujjj:tf:ttl>..
,..1:t,tt':t:.'t.'t.::t::tt:t'.
65%
~➔ illiterate Population
Pie-Chart:IV Pie-Chart
8%
There are two type of Pie Charts
1. Pie-Charts represented in Angular form
2. Pie~Charts represented in percent form
Conversion
.
,,_,,_.,_,,_ '_,,..,, ............. ~25%
100% - 360°
/ ·:::::::::::::::::::::::::_
64% ...
Similarly
360° :::; 100%
111§~§] ~ Thermal Energy
10 :::; i 00 :::; ( i O)
0
m Practice Exercise: I
Over
44 years 25-34 years
45% 24%
Science
65°
1. The marks scored by the student in Hindi and 35-44 years 20%
Mathematics exceed the marks scored in English
and Social Science by
(a) 60 (b) 75 7. In 2002, what was the approximate personal income,
(c) 40 (d) 30 in billions of rupees, of the age-group 35-44 years?
(a) 500 (b) 600
2. The subject in which the student scored approx
(c) 1125 (d) Noneofthese
22.2% marks is
(a) Hindi (b) Science 8. About how many degrees are there in the central
(c) Social Science (d) English angle devoted to the personal income of the age
group 35-44
3. The subject in which the student scored 105 marks
(a) 20 (b) 40
is
(c) 72 (d) None of these
(a) Mathematics (b) Hindi
(c) Science (d) Maths 9. What is the ratio of the personal income of the age
4. The marks obtained in the three subjects: Englsih, group 25-34 to that of the age-group 35-44?
i0.28%
(Electricity)
34.72%
(Food)
i9.44%
(Rent)
20.28%
(Clothing)
Chart-i
12. The ratio between the money spent on Rent and
Food is
(a) 1 : 2 (b) 7 : 5
(c) 14: 25 (d) None of these
during the year 1999'. Study the graph and answer these Solutions
questions. Answer 1 to 5:
Percent of money spent by a family on various 1. (a) Marks scored in Hindi and Maths
items during 1999 160
=
360
X 540 = 240
Marks scored in English and Social Science
Others 120
= X 540 = 180.
360
2. (b) 100% = 360°
Transport
~:;;.._..,.----; 5% 360
Food 22.2% = X 22.2 = 79.92° = 80°
23% iOO
Education
12% Hence difference is
240 - 180 = 60.
3. (b) 540 = 360°
360
105 = X 105 = 70°.
540
22. If the total amount spend during the year 1999 was 4. (d) 360° = 100%
Rs. 46000, the amount spent on food was 100 5
(a) Rs. 2000 (b) Rs, 10580 200° = -
360
X 200 = 55 -9 %.
(c) Rs. 23000 (d) Rs. 2300
5. (d)
23. If the total amount spend was Rs 46000, how much
6. (d)
money was spent on clothing and housing together?
(a) Rs. 11500 (b) Rs.1150 7. (a) 20% of 2499.4 billions
(c) Rs. 10000 (d) Rs. 15000
2499.4
= = 499.88 c:::. 500 billion.
24. The ratio of the total amount of money spent on 5
housing to that spent on education was 8. (c)
(a) 5 : 2 (b) 2: 5
9. (d) Required ratio 24 : 20 =6 : 5
(c) 4 : 5 (d) 5 : 4
10. (b) Total personal income in 2002 = 2499.4 billion
25. Graph shows that the maximum amount was spent
Total personal income in 2000 = x, say
on
(a) Food (b) Housing
,'. X + 20°/o Of X = 2499.4
(c) Clothing (d) Others 5
⇒X = X 2499.4 = 2082.83 = 2100.
6
26. If the total expenditure of the family for the year
1999 was Rs. 46000, the family saved during the 11. (d) Total income project in the year 2007 = 150,
year when the income in the year 2002 was 100.
(a) Rs. 1500 (b) Rs. 15000 :. Annual compound rate of growth is 8.5%
(c) Rs. 6900 (d) Rs. 3067 approx
12. (c) Ratio between the money spent on rent and food
ODDO 19.44 1944 14
= 25.
34.72 3472
fflRDI! EASY ® Pie-Chart I 213
26. (c) 15% of 46000 = Rs. 6900 6. If the total investment flows from Fll's were to be
doubled in the next year and the investment flows
214 I III Reasoning & Aptitude fflRDE EASY
Direction (Qs. 8 to 11 ): (A) and (B) exhibit the out flow Direction (Qs. 12 to 16): Pie Chart given below shows
of the tourist traffic from India. The two charts show the the expenditure incurred in bringing out a book by a
tourist distribution by country (A) and the age wise (B) publisher
traffic of the tourist respectively. Study the charts
carefully and answer the question which flow.
Printing
35%
(A)
12. What is the central angle of the sector for the cost
(B) of the paper?
(a) 22.5° (b) 16°
(c) 54.8° (d) 57.6°
(a) Rs. 25.20 (b) Rs. 37.50 21 . The ratio of the area of the circle above COF to the
(c) Rs. 31.50 (d) Rs.30 area of the circle below it is, nearly :
(a) 1 (b) 0.966
16. Royalty on the book is less than the advertisement
(c) 0.94 (d) 0.92
charges by:
(a) 3% , (b) 16
2
%
□□□□
3
(c) 20% (d) None of these
Solutions
Direction (Qs. 17 to 21 ): The gross investment of Life
Insurance Corporntion of India (in crores of rupees) in
different sectors are shown in the pie chart given below Answer 1 to 5 :
(a) 7.7% (b) 7.8% 8. (b) 40 + 10 = 50% (from the first chart)
(c) 8.6% (d) 9.2%
9. (b) 40 : 15 = 8 : 3
18. The magnitude of LAOC is nearly :
(a) i03° (b) i32° 10. (d) 5% corresponds to Switzerland's 25 lakh. Hence
(c) 126° (d) 115° 15% will be 75 lakh
19. The investment in socially oriented sectors (plan and 11. (c) US account for 40%, i.e., 8 times 5%. Since,
non plan) is ..... than the investment in Government Switzerland's 5% is 25 lakh, US will be 200 lakh
securities (Central and State) by ... 12. (d) Central angle for the cost of the paper
(a) More, 4 crore (b) More, 1 crore
(c) More, i 11 crore (d) Less, 106 crore = c~~ X 360)° = 57.6°.
20. The investment in private sectors is nearly... percent
higher than the investment in State Government 13. (b) Let the royalty be Rs. x. Then
Securities? 35 : I 5 ; ; 17500 '. X
(a) 66 (b) 54 (15xi7500) __
(c) 46 (d) 40 X == ~ ) == HS. 7500
35
216 I ® Reasoning & Aptitude fflRDE: E:RS~
i8x9000)
:. Required percentage= (
1539
1
rn x 100)%
The, 4 : 18 : : 9000 : x or x = (
4 :::: 7.i%
= Rs. 40500.
15. (c) Let the total charges be Rs. x. 18 _(b) LAOC"" [(458+i07) crores]·
1539 crores
Then, 4 : 100 : : 5544 : x or x
== (1 8
x: 444
) = Rs. 138600. = (-
565
1539
><360) = 135°.
3 = 66%
:. Required percentage = ( xi 00) % = Rs.
18
21. (c) Required Ratio
31.50
(183+454+110) 747
=
i 7. (a) Total investment = (458 + i 07 + 183 + 454 + 107 + 458 + 227 - 792
110 + 227) crores
83
⇒ 88 = 0.943
IIIIIHIIIII
Puzzles The smallest possible ten digit number is
A puzzle is a problem given that tests the ingenuity of 1,00,00,00,000
the solver. In a basic puzzle, It solver should intended We will go on modifying this number, since last
to piece together objects (puzzles pieces) in a logical digit represent number of zeros it should be 9.
way in order to come up with the desired shape, picture 1st step : 1,00,00,00,009.
or solution. Since we have only 8 zeros last digit should be
8.
Solutions to puzzles may require recognizing patterns
I 1nd step : 1,00,00,00,008
and creating a particular order. People with a high
Since number of "8" present in the above case
ireasoning aptitude may be better at solving these
is one the next modification wiii be.
puzzles.
IIJ rd Step: 1,00,00,00, 108.
Types of Puzzles Here number of zeros are only seven so
The large number of puzzles that have been created 1\flh step: 1,00,00,00, 107.
can be divided into various categories for example Here, since the number of "T' present is one
logical puzzle using chess board. so next modification will be
Other categories include vth step: 1,00,00,01 ,007.
0 Puzzle or Numbers Here, since number of" 1" present is two so the
11 Mathematical next modification will be
• Mathematical problem such as the missing square v1 th step: 2,00,00,01 ,007
puzzle Number of "2" present is be one so next
• Picture puzzle modification will be
e Connect the dots VI 1th step : 2, 10,00,01 ,007
.. Logical puzzle iike Sudoko. Here, since number of zeros present is six so
.. Spot the difference etc. next modification will be
v111 th step: 2, 10,00,01 ,006.
Lets have a glimpse of different types of puzzles
Here, since number of "6" present is 1 so next
Numbers modification will be
1x1h step: 2, 10,0o, 10,006.
Ex.1 In a ten digit number first digit represents
This number 2, 10,00, 10,006 satisfy all the
number of one present in the number. Second
condition so this is the desired number.
digit represents number of two present in the
number. Similarly, third digit represents number
of three present in the number and so on till Triangles
ninth digit, which represents number of nine
Ex.2 Find out the total number of triangles in the given
present in the number. Last digit represents
figure.
number of zero present in the number. Find that
Soi.:
number.
Let the ten digit number be CXM)<J
Ia1 Ia2 Ia3 l a4 Ias l~J a7 Ia~ Iag I~ I Fig. (1)
218 @ Reasoning & Aptitude fflRDE ERS~
Sol. First of all we will count single triangle. Sol. Number of "Single triangles'! present in the Fig.
Number of single triangle == 12 (6) are 16.
A B E G
D
~ C F H
Fig. (2)
Now, we will count triangle made by joining two
triangles for example in the below given fig. (3) D
""'---_....""'----~c
H
we have 4 triangles made by joining two single Fig. (6)
triangles. A F
A B
@
D C Fig. (7)
Fig. (3)
.6.ADB = .6.1 + L\2 Now, we will find number of double triangles in
L\ADC = .6.1 + L\2 Fig. (7) which is equal to 4 i.e.
L\BDC = .6.3 + L\4 LlAFE = L\ 1 + .6.2
L\ABC = .6.2 + .6.3 LlAFO = L\2 + L\3
So in the given fig. (2) we will have 12 such "double .6.FOE = L\3 + L\4
triangles. LlAEO = L\ 1 + L\4
Now, we will count number of triangles made by joining So total "double triangle" in fig. (6) are 16.
four tirnalges.
A B E G
~;2§]
D C F H
Fig. (4)
.6.ACE = .6.2 + .6.3 + L\5 + .6.6
D C
.6.DBF = .6.4 + ll3 + L\5 + .6.8
Fig. (8)
L\CEH = L\8 + L\7 + L\9 + L\12
Number of "Four triangle" will be 4.
L\BFG = .6.6 + L\7 + L\9 + .6.10
LlAOD = L\ 1 + L\4 + .6.10 + .6.9
So we have
LlAOB = L\2 + .6.3 + .6.5 + .6.6
12 "Single triangle"
.6.BOC = ,6,, 7 + ,0,,8 + Ll 14 + .6.15
12 "Double triangle" and
ADOC = ,6,, 12 + ,6,, 11 + Ll 13 + .6. 16
4 "Four triangle"
F
⇒ 28 triangles Ans.
Fig. (9)
Number of "Four triangle" will be 4
LlEFH = .6.3 + ,6,,4 + ,6,, 10 + L\ 11
Fig. (5)
® Miscellaneous Puzzles I 219
LlEFG = .6.4 + 113 + .6.5 + .6.8 (1) We will start solving it from back side. After
.6.FGH = .6.5 + 118 + .6.14 + .6.13 devoting 8 flowers in third temple Kavitha had
.6.EHG = Li iO + 1111 + Li 13 + Li 14 no flowers. It means before washing in third
In fig. (9) Number of "Eight Triangles" will be 4 tank she had 4 flowers.
(2) It also shows that she had 12 flower before
A B
devoting 8 flower in second temple.
(3) From this information we come to know that she
had 6 flowers before washing it in second tank.
(4) It means she had (8 + 6) == 14 flowers before
devoting to first temple.
(5) Now we can easily get the number of flower
D C she initially had before washing in first tank.
Fig. (10)
(6) From the above Fig. we can easily find solution
.6.ABC = Ll2 + 113 + 115 + .6.6 + in a simple and lucid manner.
Ll7 + 118 + .6.14 + .6.15
.6.BCD = t,,,7+ .6.8 + .6.14 + tli5 + Cat and Mouse
.6.16+.6.13+L'l1i +u12
i::iADC = t,,,1 + .6.4 + Li9 + .6.10 + Ex.5 There are four holes numbered 1, 2, 3 and 4.
The unique properties of these holes is that
Li11 + 1112 + .6.13 + .6.16
number of mouse become double, triple and
.6.ADB = t,,, 1 + L\2 + .6.3 + tl4 +
four times after entering into hole number 2, 3
.6.5 + .6.6 + .6.9 + Li 10
and 4 respectively. While it remains same if
So, total number of triangles will be
they enter in first hole. One cat is running in
16 + 16 + 4 + 4 +4 = 44 Ans.
search of few mouse. To save themselves from
cat, mouse entered in first hole and came out.
Temple and Tank - - - - - - _ZA__oL.ihem__b.acome.-Gl-fBt---ef-·ea·t:~They------·
respectively entered into second, third and
Ex.4 There are three temples near Tanjaur. In front of
fourth hole anrl come with double, triple and
;:ill temples there are one tanl\ each. l<avitha, a
four times in number. After their exit from each
lady deity went to a temple with certain number
hole i.e. second third and fourth, 24 mouse
of flower. She washed flowers in tanks in front
become diet of cat each time. At the end there
of first temple and it become doubled. She
is no mouse left. Find the initial number of
devoted 8 flowers in that temple and moved to mouse before entering into first hole.
second temple. She again washed remaining
flowers and again became doubled. She Sol.: !El+ 17 fil]
devoted 8 flowers in the second temple. She 41 34 30
again washed remaining flowers in third tank
and it become double in front of the last temple
and devoted 8 flowers. Then she was left with
no flowers. How many flowers Kavitha initially
6 0 0 6
41 17 10 6
Temple➔t'S ~ ~
become diet of cat and 6 remained.
• before entering into third hole there were
10 mouse.
Tank-+
14=8+6
l®I
tI
7
12=8+4
l®I
4I
6
l~ t4
®
e
34 i.e. 24 + 10 mouse came out of second hole,
24 become diet of cat and 1O remained.
17 mouse entered into second hole.
fflRDE EAS~
no intersecting points
2. Two lines
3. Three lines
2.
3.
4.
Ten intersecting points
⇒ O+ 1 +2+3+4= 10
Similar for n intersecting lines, maximum
intersecting points will be
Similarly we can observe in given hexagonal
0 + 1 + 2 + 3 + ... (n - 1) = l: (n - 1)
star that tvvelve coins are present in six !ines
such that each lines contains four coins.
222 ,I ® Reasoning & Aptitude fflRDE ERS~
l
exept the colour filled in strip one. Successive
y Q HKE A
strips will also be filled in the same manner i.e.
5 ways. Number of possible flags will be
j F U W XO V z D
(ii) The candidate must have completed 21 years 14. Mr. Roy is a commerce graduate, has passed his
and should not be more than 35 years as on CA examination and was born on 18-04-1974.
31-07-1996.
(iii) If the candidate does not satisfy the criterion in 15. Mr. Subramaniam whose date of birth is
(i) above but has completed his CA examination 04-05-1965 is a post graduate with Commerce and
he will be referred to Director - Finance, who has obtained 60% marks in the interview
can allow the candidate to appear in the interview Direction (Qs. 16 to 20): Read the following information
if otherwise eligible.
carefully and answer the question below it.
(iv) If the candidate fulfils all tne criteria mentioned
Three small children Sonu, Monu and Tonu went on a
in (i) and (ii) above, he/she will be called for
group discussion. picnic with their dog Jhony. They carried with them few
(v) The candidate must get 50% marks to qualify chocolates, which none of them incidentally counted on
in the group discussion. their way. They rested under a tree and slept for a while.
(vi) If the candidate qualifies in the group discussion After some time Sonu woke up, gave one chocolate from
he/she will be called for interview. the total to Jany and distributed the remaining into three
(vii) The candidate must get 30% marks in the equal parts, ate his share and slept. After some time,
interview (out of 50) to get finally selected. Monu woke up, gave one chocolate to Jony and
Based on the above criteria, decide which of distributed that remaining into three equal parts, ate his
the following course of action should be taken share and slept. After some time Tonu woke up and
in the case of candidate described in each of repeated the same. A little later all of them woke up
th e following que st ions. together, gave one chocolate from the total to Jhony
Mark answer as: and divided the remaining chocolates among them and
(1) If the candidate can be selected; each one ate his share. By chance we know that the
(2) If the candidate is to be referred to Director - total number of chocolates were less than 150 in the
_ _i_F~in~an~c~e~;_ _ _ _ _ _ _ _ _ _ _ _ _ _ --b~mringanditrey-ditln'fl'.Yreak an·STcnocolate.
(3) If the candidate can be called for group
discussion; 16. How many chocol;:ites were in the beginning?
(4) If the candidate can be called for interview; (a) 66 (b) 84
(5) lfthe candidate cannot be selected (c) 1i8 (d) 79
3. Mr. Das i~ M.Com and his date of birth is 17. What is the difference in the number of chocolates
30-06-1961 eaten by Monu and Tonu?
(a) 6 (b) 11
~- Mrs. Krishnamurthy is a qualified CA and was
(c) i4 (d) 18
25 years old on 31-07-1996
I 0. Mr. Kant, a post-graduate with Commerce obtained 18. What is the difference in the number of chocolates
70% marks in group discussion and secured eaten by Sonu and Tonu?
20 marks in Interview. He was 35 years as on (a) 25 (b) 15
3i-07-i996 (c) 20 (d) 18
1. Mrs. Desai is a post-graduate in Economics and i 9. Monu and Tonu ate the chocolates in the ratio ...
her date of birth is 3-04-1965 (a) 13 : 11 (b) 9: 7
(c) 4: 3 (d) 5: 4
2. Mr. Patel, a post-graduate was permitted by Director
Finance. He was born on 30-05-1963. He obtained 20. How many more chocolates did monu eat, than
12 marks in interview Jhony?
3. Mr. Mathur is Ph.Din Cornrnerce and was 34 years (a) 20 (b) 27
old on i-08- i 995. He obtained i 4 marks in interview. (c) 32 (d) 36
224 I ® Reasoning & Aptitude fflRDE ERS~
Direction (Qs. 21 to 25) : Read the following information Direction (Qs. 29 to 33): Read the following information
carefully and answer the question below it. carefully and answer the question below it.
Atul, Bhupinder, Charles and Deepak are four brothers Rahul was at a crossroads in his life. He went to an
playing a game where the loser doubles the money of astrologer to learn what the future held for him. The
each of the other players by giving them from his share astrologer explained that Rahul had many choices to
at that point of time. They played four games and each make and he could only expalin what each choice lead
brother lost one game in alphabetical order. At the end to. On leaving the astrologer, Rahul could could take
of the fourth game each brother had Rs. 64.
bus routes 213 or 231 to go to his next stop. The first
21 . Who started with the lowest amount? led to a choice between two jobs and the second to a
(a) Atul (b) Bhupinder choice between two areas of business.
(c) Charles (d) Deepak In all four cases, Rahul would go abroad. Rahul would
22. What was the amount left with Charles at the end of marry a woman whose name begins with 'P' if he either
the second round? took a job in production or had a business in readymade
(a) 72 (b) 144 garments. Rahul would go to the US if he either had a
(c) 132 (d) 136 job in marketing or a business in spare parts manufacture.
23. How many rupees did Bhupinder start with? 29. If Rahul has a wife called Pradnya and a production
(a) 64 (b) 136 job, which bus did he take?
(c) 68 (d) 72 (a) 213
24. At the end of the fourth round, who had the max. (b) 231
profit? (max. wining vis-a-vis opening amount) (c) (1)or(2)
(a) Atul (b) Bhupinder (d) cannot be determined
(c) Charles (d) Deepak
30. If Rahul took bus number 231 and married Rohini,
25. Who had the minimum deviation from the opening what work does he do?
amount? (a) marketing job
(a) Atul (b) Bhupinder (b) production job
(c) Charles (d) Deepak (c) garments business
(d) spare parts business
Direction (Qs. 26 to 28): Read the following information
carefully and answer the question below it. 31. If Rahul took bus number 213 and is married to
Life was not easy before Aryabhatta. Zero was not Rehana, which country does he live in?
invented and also people could not multiply numbers. (a) India (b) UK
Thus mathematics had only 9 digits (1 to 9) and after (c) US (d) Canada
that came 11 and so on. Find the answer to following
operations in mathematics used then. 32. If Rahul has a spare parts business, which of the
following could be his wife's name?
26. What is 7 + 1 i + 3 =?
(a) Pradnya (b) Prachi
(a) 20 (b) 21
(c) Savita (d) Any of these
(c) 22 (d) 23
33. If Rahul works in Australia, which of the following is
27. What is 21 + 29 + i =?
(a) 50 (b) 51 possible?
DD □□
Solution~ , a, {5) CMaltlon saHsfi@s all the th1e t©rKHtlon excepf
dollditGfl (Vil)
er 1 to 3:
tilt th@ number of sweet$ b~ ,t 14, (~) A6cmd1ng to coMitiof1 (Hi) he should be ref@r@d
Number of @W@(C)ts l@ft with him ~tt~r Temple, to Dir@Gtm ~ Finance,
Ht (1) He will b@ §ei@ct@tJ
Answ1r 15 to ~O:
Let th§ number of chocolates tn the beg1Mtfl§J
x , b@ X,
begg@rn "" =2
2 ~o when Sonu woke up,
Numbm of owccto left with him ~ft©r givlnfi to the
X
he left 12 (x ~ i)·..tor the mhers,
poor children ;;;
4= 1 When Manu ~1oke u~ he left
Number of swe@ts l@ft with him gjffer giving firs
brother ,,,
x
~
-
2=i
2[23'(x=i,~i
3
. ]= · ·§ · 4.t=W
4
Thh\J gives x :i.':: 12
1. (c)
3. (b)
AnSW€1f 4 to 8 :
parts,
4. (c) Lis missing in the pyram,d
~x = ~8-.e. 27 Sx-=65 . .
5. (d) PR are not occurring together in atphabeficar So - - -- · · or --- ~-:::···· was tfrvrded imo
27 'lc7
order WX iind UV t:tr~ not v@rtlcal Mlghb©urs in
---------g---giv~pytamltf::---S+oc:utrr--t~rtMtn . . _. fi=OO
alphabetical ord~r and aJ~© ~r@J v~rtioaf mrne e-q~a, pans. This means · · "i"i-· ts
neighbour~ in ei filv@n pyr@lrnki.
aivlsioie ~Y 3 or Sx"" 65 Is a muijlpfe tt1 tn, tef
6. {b) Y and O; N i\lnd P; E and K are i1miz©nfa1I Bx.= 6~ = ti ni wiwre (ii Is an irlfeget
neiabourn in th~ ~lv@n pyr3mid1 bYi flt1me of ~(, ~in -f 65 sh®id be dMsibl§ t}y @. It ha§ i§
these pairs does r1ot ©c:©!H Wgeff'MU k1 be ~fil ew@n, numbgr,, s~ 8fo1 should be' ooa
alphabetical order. Both in Hie pyramid arra In, litJFt/6'@(. fryifig Witfi, 1, j, 5 ... ,.... We gm 7 arig
alphab@tica! ortJer V and x are teparated by t t as: trre tw@ inm~r, possrb>le number bui n= ii-
exactly one fetter.
wm qi.we ~§ an INitrar. stari/inig lfii!.Hnber x = iOO,
7. (c) F and U are separatet:l !Jy i4 l@H@rs ir11 ~n wM@rv vs lfl@t at@ept~bte. §©•, n: = 1' /is t~e
aJphat:ieticaf order, while O ~nd Yare oopar.med act@pt@d waioo. ~ lni t~ beglmin~ tlh:ete weuJ
by 7 fMters; C and S sep~ratea by 1& letters . 7g o~oim@s wiif:il trie' gw~n oorMjfiio'fr§. We t{in
and H arid Oare ~@paratoo iJy t: letters. Helf'l:ce dfsirrtJute 111,e·m ~ccoiroiijg;ly. T'11e tab#} oolow
C and s are the tanhest away from eac:hi other show ~~m©er §f G~~at~@ eat@n' f!}y eaoh.
among given pairs.
Answer 8 to 15:
8. (5} Condrtiorr M ,s: noi sat i,s.f iiea
9. (2) AU the condrtrons. of. ef.rgib"i:ty ~re· saMfi'etf
i o. (1; AH condmon is are sa1i§fiied
1 i , (5) condttlorr M is not sa!isH~d
1 (5) Condrrion sa1rsfres an tr.re tive concMiorT except
c-0ndrton (vii)
226 ® Reasoning & Aptitude fflRDE ERS~
C D
IIAB represents A and B only i.e. both A and B. le represents Conly (not A, B or D)
I 0 represents D only (not A, B or C)
Case n : Whem there are only three articles
HAa represents A and B only (not C or D)
IIAc represents A and C oniy (not B or D)
Hao represents Band D only (not A or C)
IIco represents C and D only (not A or B)
IIIAac represents A, Band Conly (not D)
IIIaco represents B, C and D only (not A)
IIIAco represents A, C and D only (not B)
IIIAao represents A, Band D only (not C)
IVAaco represents A, BC and D all.
i.e. region which is common to all A, B, C and D.
Here IA represent A only (not B or C)
Note: A, B, C and D can be represented in form of a
Ia represents 8 only (not A or C)
triangle, circle, rectangle or square. The logic behind
Ic represents Conly (not A or B)
venn diagram will remain unchanged irrespective of the
IIAa represents A and B only (not C)
type of figure.
II 8 c represents Band Conly (not A)
□□□□
228 I ® Reasoning & Aptitude ffiADE EASY
Direction (Os. 2 to 5): The figure given below consists Direction (Qs. 7 to 11) : Below is given a figure with
of three interesting circles which represent sets of four intersecting circles, each representing a group of
students who play Tennis, Badminton and Volleyball. persons having the quality written against it. Study the
Each region in the figure is represented by a small letter. figure carefully and answer the qu8stions that follow.
Tennis Badminton
Hard working
F H
Volleyball
M
On the basis of the above figure, answer the
questions given below. 7. The region which represents the people who are
2. Which letter represents the set of persons who play intelligent, honest and truthful but not hard working
all the three games? is denoted by
(a) b (b) C (a) E (b) F
(c) M (d) I
(c) f (d) g
1o. People who are not hard working, intelligent and 16. In the following diagram, square represents women,
truthful are represented by triangle represents sub-inspectors of police and
(a) G (b) H circle represents graduates. Which numbered area
~) K ~) L represents women graduate sub-inspector of police?
11 . People who are not honest and truthful but are hard
working and intelligent both, are represented by
(a) E (b) B
(c) M (d) I
(a) 3 (b) 4
(c) 5 (d) 6
8 13
13. Number of graduates in social organisations
(a) 1 (b) 5
(c) 6 9 14
(d) 5 and 6
18. Who among the following is an educated male who 26. Non-rural, employed, hard working and intelligent
is not an urban resident? people are indicated by region
(a) 4 (b) 5 (a) 8 (b) 9
(c) 9 (d) 11 (c) 10 (d) ii
19. Who among the following is neither a civil servant 27. Non-rural, employed people who are neither
nor educated but is urban and not a male? intelligent nor hard working are represented by region
(a) 2 (b) 3 (a) 12 (b) 11
(c) 6 (d) 10 (c) 10 (d) 7
20. Who among the following is a female, urban resident 28. Intelligent, employed and hard working non rural
and also a civil servant? people are indicated by region
(a) 11 (b) 6
(a) 6 (b) 7
(c) 9 (d) 4
(c) 10 (d) 13
29. Hard working non-rural people who are neither
21. Who among the following is an educated male who
employed nor intelligent are shown in region
hails from urban area?
(a) 8 (b) 7
(a) 4 (b) 2
(c) 6 (d) 12
(c) 11 (d) 5
30. Employed, hard working and intelligent rural people
22. Who among the following is uneducated and also
are indicated by region
an urban male?
(a) 1 (b) 2
(a) 2 (b) 3 (c) 3 (d) 4
(c) 11 (d) 12
31 . Rural hard working people who are neither employed
23. Who among the following is only a civil servant but nor-intelligent are indicated by region
neither a male nor urban oriented and uneducated? (a) 6 (b) 5
(a) 7 (b) 8 (c) 4 (d) 3
(c) 9 (d) 14
32. Rural employed people who are neither intelligent
24. Who among the following is a male, urban oriented nor hard working are indicated by region
and also a civil servant but not educated? (a) 2 (b) 4
(a) 13 (b) 12 (c) 6 (d) 9
(c) 6 (d) 10
33. Rural people who are hard working and employed
25. Who among the following is a male civil servant, but not intelligent are indicated by region
who is neither educated nor belongs to urban area? (a) 1 (b) 2
(a) 7 (b) 13 (c) 3 (d) 4
(c) 4 (d) 1
34. Unemployed rural hard working and intelligent people
Direction (Qs. 26 to 35): In the following figure, the circle are indicated by region
stands for employed, the square stands for hard working, (a) 1 (b) 2
the triangle stands for rural and the rectangle stands for (c) 3 (d) 4
intelligent. Study the figure carefully and answer the 35. Rural employed people who are neither intelligent
questions that follow. nor hard working are indicated by region
(a) 10 (b) 9
(c) 6 (d) 4
Solutions
1. (c) The required region is the one which is common
to the triangle and the circle but is not a part of
the square i.e. IV.
® Logical Venn Diagrams I 231
(b) The required region is the one common to all the the triangle and the rectangle but lies outside
three circles i.e. c. the circle i.e. 11.
(d) The required region is the one which is common 19.(b) The person satisfying the given conditions is
· to circlos P nnd R but is not a part of circle 0 represented by the region which lies inside the
i.e. b. circle but outside the square, the rectangle and
(a) The required region is the one which lies inside the triangle i.e. 3.
circle P but is not common to circle O or circle R
20.(c) The person satisfying the given conditions is
or both i.e. a.
represented by the region which lies outside the
(b) The required region is the one which is common rectangle and is common to the circle and the
to circles P and O but lios outside circle R i.e. d. square i.e. iO.
i6. (b) The required region is the region which is common 21. (a) The person satisfying the given conditions is
to the triangle and square but lies outside the represented by the region which is common to
circle i.e., 8. the triangle and the rectangle and also lies inside
7. (c) The required region is the one which is common the circle i.e. 4.
to the circles 2, 3 and 4 but is not a part of circle
22.(d) The person satisfying the given conditions is
1 i.e. fVl. represented by the region which lies outside the
8. (d) The required region is the one which is common triangle and is common to the circle and the
to all the four circles i.e. E. rectangle i.e. 12.
9. (c) The required region is the one which is common Remember: The condition which is not
to the circles 1, 2 and 4, but lies outside circle mentioned shouldn't be considered or assumed.
3 i.e., F. For instance, here, 6 also denotes the required
1O.(d) The required region is the one which does not lie region. But since it lies inside the square and
inside circles 1, 2 and 4 i.e. L. there is no mention of 'civil servant', so it cannot
----be-t~e-answe .
----Tf:f61The given conditions are satisfied by the persons
denoted by the region which is common to circles 23.(a) The person satisfying the given is denoted by
i and 2 but is not a part of either circle 3 or circle the region which lies inside the square but outside
4 i.e. B. the circle, rectangle and triangle i.e. 7.
12.(b) Stronq armymen will be represented by the region 24.(c) The person s8tisfying the given conditions is
which is common to the square and the triangle denoted by the region which is common to the
but lies outside the circle i.e. 4. rectangle, circle and the square but lies outside
the triangle i.e. 6.
13.(d) The required region is the one common to the
circle and triangle i.e regions 5 and 6 25.(b) The person satisfying the given conditions is
14.(d) The required reQion is the one common to the represented by the region common to the
triangle and circle but lies outside the square rectangle and the square but lying outside the
15. (a) The required region is the one common to the 26.(b) The required set of people is represented by the
triangle and square i.e. regions 5 and 7. region which lies outside the triangle and is
common to the circle, square and rectangle i.e. 9.
16.(b) The required region is the one common to the
square, triangle and circle i.e. 3. 27 .(d) The required set of people is represented by the
region which lies outside the triangle, inside the
17. (a) The required region is the one which is common
circle but outside the rectangle and the square
to circles X and Z but lies outside circles Y.
i.e. 7.
i.e. T.
28. (c) The required set of people is represented by the
18 .( d) The person satisfying the given conditions is region which is common to the rectangle, circle
represented by the region which is common to and square but lies outside the triangle i.e. 9.
232 I ® Reasoning & Aptitude fflRDE EASY
29.(d) The required set of people of denoted by the 33 .(b) The a'quired set of people is represented by the
region which lies inside the square but outside region which is common to the triangle, square
the triangle, circle and rectangle i.e. i2. and circle but is not a part of the rectangle i.e. 2.
30.(a) The required set of people is denoted by the 34.(d) The required set of people is represented by the
region common to the circle, square, rectangle region which lies outside the circle and is
and triangle i.e. 1. common to the triangle, square and rectangle
31.(d) The required set of people is represented by the i.e4.
region which is common to the triangle and the 35.(c) The required set of people is denoted by the
square but lies outside the circle and rectangle region which is common to the triangle and circle
i.e. 3. but is not a part of either the rectangle or the
32.(c) The required set of people is denoted by the square i..e 6.
region which is common to the triangle and the
circle, but is not a part of either the rectangle or llllllllllllllli
the square i.e. 6.
four equal parts, the three of them ate one part each
I Practice Exercise: I and kept the remaining laddoos in the box. Later when
D came, he again divided the laddoos in four equal parts
Direction for Questions: (1 to 3) and all four ate their respectively share. In total D ate
(i) Six friends A, B, C, D, E and Fare sitting along 3 laddoos.
the sides of a hexagonal table for playing a 4. How many laddoos, in total did C eat?
game, though not necessarily in the same order, (a) 12 (b) 15
(ii) F, who is sitting exactly opposite of A. is to the (c) 39 (d) None of these
immediate right B. 5. How many iaddoos, in total did Beat?
(iil) D ls between A and B and is exactly opposite (a) 24 (b) 15
to C. (c) 39 (d) None of these
1. A is sitting between which of the following pairs of 6. How many laddoos, in total did A eat?
persons? (a) 56 (b) 68
(a) D and E (b) 8 and E (c) 71 (d) None of these
,.,, E . d C
.~-·J~=an'-'=-=-- ·__(_d}_t!onaoUbesf.:L----- -7,--Row many laddoos were given to A by his Uncle?
2. Who is sitting opposite B? (a) 128 (b) 125
(a) E (b) F (c) 113 {d) None of these
(c) A (d) C 8. How many laddoos, did A eat the first time?
3. Three of the following are alH<e in a certain way on (a) 32 (b) 24
the basis of sitting positions and so from a group. (c) i5 {d) None of these
Which is the one that does not belong to the group? Direction for Questions: (9 to 13)
(a) B, C (b) A, D (I) A, B, C, D, E, F G and Hare eight friends. Three
(c) B, F (d) E, A of then play cricket and table tennis each and
Direction for Questions: (4 to 8) tvvo of them play football. Each one of them has
A. B, C and D are four friends living together in a flat a different height
and they have an agreement that whatever edible comes (ii) The tallest does not play football and the shortest
they will share equally among themselves. One day A' s does not play circket
uncle came to him and gc;1ve a box of laddoos. Since no (iii) F is taller than A and D but shorter than H and
one was around, A divided the laddoos in four equal B. E who does not play cricket, is taller than B
parts and ate his share after which he put the rest in the and is second to the tallest G is shorter than D
box, As he was closing the box, B walked in and took but taller than A
th~ box, He again divided remaining laddoos in four (iv) H, who ls fourth from the top, play table tennis
equal parts, A and 8 ate one part each and kept the with D,
remaining iaddoos in the box, Suddenly C appeared (v) G does not play either cricket or football. 8 doest
and snatched the box. He again divided the laddoos in not piay football.
234 / 0 Reasoning & Aptitude fflRDE ERS~
23. Which colour shirt is sponsored by Raymond's? 30. Which course is taught in the month of January?
(a) Yellow (a) C (b) D
(b) Blue (c) E (d) Data inadequate
(c) Pink
Direction (Os. 31 to 34)
(d) Cannot be determined
Rajeev planted some plants In his lawn but in a certain
24. Which pair is correctly matched? fixed pattern:
(a) Red-Raymond's-A (i) In most of the rows there are neither Rose nor
(b) Red-Trump 8 Gates-8 Marigold
(c) Green-Raymond's-C (ii) There are two more row of Orchids than Tulips
(d) None of these and two more rows of Rose than Orchids.
(iii) There are four more rows of Rose than Tulips.
25. Which of the following is true? (iv) There aren't as many rows of Lilly as Fireball.
(a) Udupi sponsors Green Shirt (v) There is one less Marigold row than Rose
(b) Dis working in Trump & Gates (vi) There is just one row of Tulips
(c) E wears Red Shirt (vii) The maximum number of rows he planted is six.
(d) Red shirt is sponsored by Trump & Gates
31 . How many rows of rose the planted?
26. What is the sequence of companies representing (a) Two
A, B, C, D, E & F? (b) Five
(a) Quark, Pentasoft, Trump & Gates, Raymond's (c) Four
Udupi, Sunmet (d) cannot be determined
(b) Quark, Trump & Gates, Pentasoft, Raymond's
Udupi, Sunmet 32. Which of the above information is redundant and
(c) Quark. Pentasoft, Trump & Gates, Sunmet, can be dispensed with?
Udupi, Raymond's (a) (i) (b) (iii)
---1c-d)--WGA@-0f-m@s@-- - - - - - - - - - - - ~ c L ( i } and {iii) both (ct.LAIi are neCLessa-ryc_____ ---------------
27. If Raymond's and Sun met decide to interchange the 33. What is the sum of the rows of Orchids and Marigold
colours of sponsored shirts, then which two persons he planted?
had to interchange their shirt? (a) Three
(a) D & F (b) A & C (b) Nine
(c) o & E (d) 8 & 0 (c) Seven
(d) Cannot be determined
Direction (Qs. 28 to 30)
3'4. How many rows of fireball did he plant
=ive courses - A, B, C, D and E each of one month
(a) Two (b) Six
:luration are to be taught from January to May one after
(c) Two or Six (d) Data inadequate
:he other though not necessarily in the same order by
ectures P, 0, R, Sand T. P teaches course 'B' but not in Direction (Os. 35 to 37)
:he month of April or May. Q teaches course 'A' in the (i) Five friends, Amar, Kapil, Sarvesh, Rohan and
rionth of March. R teaches in the month of January but Nagesh wear trousers of different colours= red,
:foes not teach course 'C' or 'D', yellow, blue, white and green (not necessarily
28. Which course is taught by S? in the same order)
(a) C (b) E (ii) Each one of them has different likings, viz,
(c) Either C or D (d) D reading, playing travelling, singing and writting.
(iii) Kapil, who has liking for singing does not wear
29. Which lecture's course immediately follows after yellow trousers. Sarvesh wears red trouser and
course B? does not like reading or writing. Nagesh likes to
(a) 0 (b) p play and does not wear blue or Yellow trousers.
(c) S (d) T Amar has liking for writing and Ror,an does not
wear yellow or green trousers.
II
236 I Ell Reasoning & Aptitude fflRDE ERS~
I
------,.2
B's share 24 24 48
6 3 C's share 12 12 12 12
D's share 3 3 3 3 0
5 4
From the second statement it is clear that F
71 39 15 3
l
and A are sitting opposite to each other and F
is to the immediate right of B.
Now we can answer all the question very easily.
4. (b)
I
The sitting arrangement can be like this 5. (c)
B-,-----.. 6. (c)
7. (a)
F A
8. (a)
Solution 9 to 13
From the third statement it is clear that D is We will read each statement one by one and start coding
sitting between A and Band is opposite to C. accordingly.
The sitting arrangement can be like this ® Three of them play cricket and table tennis each
B.,______ o
and two of them play football.
I
3 Cr 3 TT 2 Fb
F A Each of them has a different height
,__,__ Tallest
C
There is only one vacant position for E, so the
overall sitting arrangement will be like this
s____.....o
C E
fflRDE EASY "' Analytical Reasoning I 237
e The tallest does not play football and the shortest 9. (c)
does not play cricket. 10. (c)
11. (b)
0~ 0~ 12. (a)
• Fis taller than A and D but shorter than H and B. 13. (b)
The arrangement can be like this
Solution ( i 4 to 17)
A F D
Left
D A F
E ➔ C>z From the last sentence, we get that Fis also on the
and is taller than B and also second to the tallest left side, So E will be on the right side. D will be on
the left side
So E's position is ® E does not have a corner office so E must be in the
G is shorter than D but taller than A middle.
E and F do not face each other and F's office is
further down the corridor, so F occupies the last
office on the left side.
Office of C and D face each other, so C will have
"' H is fourth from the top so its position is first office on the right side and D will have the first
H➔0 office on the left side.
· -----Finat-officetrrrangemern will15elfke this
H plays TT with D so
H ➔ TT
=~
Left~
D➔ TT
41 G does not play either cricket or football so G will
play table tennis
G ➔ TT 14. (c)
Now overall arrangements will be like this 15. (d)
16. (a)
Since there is a corridor between office of
C ➔ ~➔ Cr F & B so A is the only neighbour of F.
17. (b)
2 E ➔ tz ➔ Fb
Solution (18 to 22)
3 B Cr
----------➔
There are three vehicles
~
Co ,pa es
A B C D E F ~
Comoanies Blue Green Pink Yellow Purple Red
Pentasoft Pentasoft
Quark Quark
Raymonds Raymonds
Sunmet Sunmet
T&G T&G
Udupi Udupi
We will start reading each conditions and mark a ' ✓' sign or 'x' sign accordingly.
From the condition (i) we get following tables.
~s
Compane,
A B C D E F
~
Companies Blue Green Pink Yellow Purple Red
Pentasoft Pentasoft J(
✓ X J( X X
Quark Quark X J(
~
C
A B C D E F ~
Comoan,es Blue Green Pink Yellow Purple Red
Quark Quark JC JC
Raymonds )(
Raymonds .IC JC
Sunmet Sunmet ✓ JC JC JC J( j(
T&G )(
T&G JC .)C
~s
Comoanies
A B C D E F ~
Comoanies Blue Green Pink Yellow Purple Red
Pentasoft )( j( Pentasoft X ✓ X X X X
Udupi )( Udupi J( j( JC
.,
~
Companies
A B C D E F
C~ Blue Green Pink Yellow Purple Red
~~s
Compan es
Pentasoft
Quark
A
)(
✓
B
J<
C
)<
D
J<
E
)(
F
)(
·~
Companies
Pentasoft
Quark
Blue Green Pink Yellow Purple Red
X
.it
✓
.IC ✓
JC .IC
JC
JC
JC
JC
.I(
T&G )( X )( )(
T&G JC JC j(
Udupi )( X )( )( ✓ )( Udupi JC J( J(
240 I ® Reasoning & Aptitude ffl!RDE ERS'!:i
.~
C mpa e
A B C D E F
~
C
IP Blue Green Pink Yellow Purple Red
X
Sunmet l< ·X X X Sunmet ✓ X X X X X
Udupi X X X X ✓ X Udupi X X X
Now we will mark' ✓' on the only left space and mark 'x' on the remaining spaces. Then the following table emerges.
~s
C ,p, A B C D E F
~
Co 1p Blue Green Pink Yellow Purple Red
Pentasoft X X ✓ X X X Pentasoft X ✓ X X X X
Quark ✓ X X X X X Quark X X ✓ J( X X
Raymonds X X X ✓ X X Raymonds X X X ✓ X X
Sunmet X X X X X
✓ Sunmet ✓ X X X X X
T&G X ✓ X X X X T&G X X X X X ✓
Udupi X X X X ✓ X Udupi X X X X ✓ X
With the help of the above tables we can solve all the questions easily.
Note: After drawing table further modification can be done according to given conditions. There is no need to draw
further tables. Tables drawn further is just to explain the steps in elaborate manner.
23. (a)
24. (b)
25. (d)
26. (b)
27. (a)
Solution (28 to 30)
1.1\/e can be solved these questions with the help of tabular chart.
~s
Cou ses
Jan Feb Mar Apr May ~s
s
p Q R s T
A A
B 8
C C
D D
E E
We will start reading each condition and mark' ✓' sign or 'x' sign accordingly.
)'
Months
Courses
Jan Feb Mar Apr May ~s
Courses
p Q R s T
A A J<
C, C l<
D D X
E E l<
~~
-- Lecturers
s
Jan Feb Mar Apr May
Co~-
p Q R s T
C l< C )( X
D l< D )( .x
E J< E .x Jc
~s
Courses
Jan Feb Mar Apr May ~s
Courses
p Q R s T
A l< J< ✓ le )( A )(
✓ X X Jc
B J< ✓ X )( X B ✓ JC )( l< Jc
Now, we can easily answer given questions witl1 Uie help of above table.
Note: After drawing first table further modifications can be done according to conditions given. There is no need to
draw further table. Further table drawn is just to explain the steps in elaborate manner.
Final table can be referred as.
28. (c)
29. (a)
30. (c)
Also
R= M + 1
When T :::: 1, 0 = 3, R = 5, M = 4
Since from (iv) There are not as many rows of lily as fireball
Lily < Fireball
So, Lily "" 2, Fireball t::: 6
Now we can answer all the question easily.
31. (b)
32. (c)
33, (C)
34. (b)
Solution (35 to 37)
These questions can be solved easily by drawing tabular chart.
i
~
s Red Yellow Blue White Green
~
s Read Play Travel Sing Write
Amar Amar
Kapil Kapil
Sarvesh Sarvesh
Rohan Rohan
Nagesh Nagesh
We will start reading each condition and mark' ✓' sign or '.1e' sign accordingly.
ID Kapil, who has liking for singing does not wear yellow trouser.
~
F
Red Yellow Blue White Green
~
e Read Play Travel Sing Write
Amar Amar !(
@ Sarvesh wears read trouser and does not like reading or writing.
~
F
Red Yellow Blue White Green
~
s Read Play Travel Sing Write
@ Nagesh like to play and does not wear blue or yellow trousers.
~
s
Red Yellow Blue White Green
~ . Read Play Travel Sing Write
Amar )(
Amar ,I< )(
Sarvesh ✓
)( )( )( )( Sarvesh )( )( )( l<
Rohan )(
Rohan )( )(
@ Amar has liking for weiting and Rohan does not wear yellow or green trousers.
Amar )(
✓ Amar )( )( )( )( ✓
Kapil )( J( Kapil )( )( )( ✓ )(
Rohan )( )( )( Rohan ✓ )( )( )( )(
Nagesh )( )( )( Nagesh )(
✓ )( )( )(
Now, we can easily answer given questions with the help of above table.
Note: After drawing first table further modifications can be done according to conditions given there is no need to
draw further table. Further table drawn is just to explain the steps in elaborate manner.
35. (d)
36. (b)
37. (c)
11!111111111111111
Section
·~
10
i. 25 persons are in a room. 15 of them play hockey, In what order were they born ( oldest first)?
i 7 of them play football and 1Oof then 1 µlay both (a) HSIG (b) SGHI
hockey and football. Then the number of persons (c) IGSH (d) IHSG
playing neither hockey nor football is [2010, 2 Marks]
(a) 2 (b) 17
6. If Log(P) = (1/2)Log(O) = (1/3)Log(R), then which
(c) 13 (d) 3
of the following options is TRUE?
[2010, 1 Mark]
(a) P2 = 0 3 R2 (b) 0 2 = PR
2. If 137 + 276 = 435 how much is 731 + 672? (c) 0 2 = R3 P (d) R = P2 0 2
(a) 534 (b) 1403 [CE, ME, CS 2011, 1 Mark (Set-1 )]
,.J\ ➔ Cr).f
( u; 1001
7. A container originally contains 1 O litres of pure
(2010, 2 Marks]
spirit. From this container i litre of spirit is replaced
3. 5 skilled workers can build a wall in 20 days; with 1 litre of water. Subsequently, i litre of the
8 semiskilled workers can build a wall in 25 days; mixture is again replaced with i litre of water and
10 unskilled workers can build a wall in 30 days. If this processes is repeated one more time. How
a team has 2 skilled, 6 semiskilled and 5 unskilled much spirit is now left in the container?
weFk-er:s,hew-l0n§-wilHHake1u·tmrtcttnewalT7- · · · -Taf7-:-587ffres. (b) 7.84 litres
(a) 20 days (b) 18 days (c) 7 litres (d) 7.29 litres
(c) 16 days (d) 15 days [CE, ME, CS 2011, 2 Marks (Set-1 )]
[2010, 2 Marks)
8. The variable cost (V) of manufacturing a product
4. Given digits 2, 2, 3, 3, 3, 4, 4, 4, 4 how many varies according to the equation V = 4q, where q
distinct 4 digit numbers greater than 3000 can be is the quantity produced. The fixed cost (F) of
formed? production of same product reduces with q
(a) 50 (b) 51 according to the equation F = 100/q. How many
(c) 52 (d) 54 units should be produced to minimize the total cost
[2010, 2 Marks] (V + F)?
(a) 5 (b) 4
5. Hari (H), Gita (G), lrfan (I) and Saira (S) are siblings (c) 7 (d) 6
(i.e. brothers and sisters). All were born on
[CE, ME, CS 2011, 2 Marks (Set-1)]
1st January. The age difference between any two
successive siblings (that is born one after another) 9. P, 0, R and S are four types of dangerous microbes
is less than 3 years. Given the following facts: recently found in a human habitat. The area of each
circle with its diameter printed in brackets
1. Hari's age + Gita's age > lrfan's age + Saira's
represents the growth of a single microbe surviving
age.
human immunity system within 24 hours of entering
2. The age difference between Gita and Saira is
the body. The danger to human beings varies
1 year. However, Gita is not the oldest and Saira
proportionately with the toxicity, potency and
is not tr1e youngest.
growth attributed to a microbe shov✓n in the figure
3. There are no twins. below:
246 I ® Reasoning & Aptitude fflRDE EIRS~
120
C ~
r r-{(50mm)
.Q .~
Q:: 90
\. LJ" E Q)
:::, 0..
(/)
C
(/)
~ 60
Q~m) o2
0 Q)
(DR(30mm)
S(20mm) -E
Q) 0
:::,_ 30
u,_;g
0.4 06 0.8
0
0 15 30 45 60 75 90
(Probability that microbe will overcome human immunity system)
Speed
(kilometers per hour)
A pharmaceutical company is contemplating the The distance covered during four laps of the
development of a vaccine against the most journey are listed in the table below:
dangerous microbe. Which microbe should the
company target in its first attempt? Distance Average speed
(aj p (~Q Lap (kilometers) (kilometers per hour)
(c) R (d) S p 15 15
[CE, ME, CS 2011, 2 Marks (Set-1 )]
Q 75 45
10. A transporter receives the same number of orders R 40 75
each day. Currently, he has some pending orders s iO 10
(backlog) to be shipped. If he uses 7 trucks, then
at the end of the 4th day he can clear all the orders. From the given data, we can conclude that the
Alternatively, if he uses only 3 trucks, then all the fuel consumed per kilometre was least during the
orders are cleared at the end of the 10th day. What lap
(a) p (b) Q
is the minimum number of trucks required so that
(c) R (d) S
there will be no pending order at the end of the
5th day? [EE, EC 2011, 2 Marks (Set-2)]
(a) 4 (b) 5 13. Three friends, R, S and T shared toffee from a
(c) 6 (d) 7 bowl. R took 1/3 rd of the toffees, but returned four
[CE, ME, CS 2011, 2 Marks (Set-1 )] to the bowl. S took 1/4th of what was left but
returned three toffees to the bowl. T took half of
11. There are two candidates P and Q in an election.
the remainder but returned two back into the bowl.
During the campaign 40% of the voters promised
If the bowl had 17 toffees left, how many toffees
to vote for P, and rest for 0. However, on the day
were originally there in the bowl?
of election 15% of the voters went back on their
(a) 38 (b) 31
promise to vote for P and instead voted for O .
(c) 48 (d) 41
25% of the voters went back on their promise to
[EE, EC 2011, 2 Marks (Set-2)]
vote for Q and instead voted for P. Suppose, P
lost by 2 votes, then what was the total number of 14. Given that f(y) == Iy I/y, and q is any non-zero real
voters? number, the value of If( q) - f(--q) Iis
(a) 100 (b) 110 (a) 0 (b) -1
(c) 90 (d) 95 (c) 1 (d) 2
[EE, EC 2011, 1 Mark (Set-2)] [EE, EC 2011, 2 Marks (Set-2)]
12. The fuel consumed by a motorcycle during a 15. The sum of n terms of the series 4 + 44 + 444 + ...
journey while travelling at various speeds is is
indicated in the graph below (a) (4/81) [10n + 1 - 9n -1]
(b) (4/81)[10n- 1 -9n-1J
® Previous Years GATE Solved Questions I 247
(c) (4/81)[1on+ 1 -9n-i0] 20. Given the sequence of terms, AD CG FK JP, the
(d) (4/81)[10n-9n-10] next time is
[EE, EC 2011, 2 Marks (Set-2)] (a) OV (b) OW
(c) PV (d) PW
16. The cost function for a product in a firm is given
[CE, ME, CS 2012, 2 Marks (Set-1 )]
by 5q 2 , where q is the amount of production. The
firm can sell the product at a market price of~ 50 21. If (1.00, ) 1259 = 3.52 (1.001 )2062 = 7.85, then
per unit. The number of units to be produced by (1.001)3321
the firm such that the profit is maximized is (a) 2.23 (b) 4.33
(a) 5 (b) 10 (c) 11.37 (d) 27.64
(c) 15 (rl) ?5 [EE, EC 2012, 1 Mark (Set-2)]
[CE, ME, CS 2012, 1 Mark (Set-1}] 22. Raju has 14 currency notes in his pocket consisting
17. A political party orders an arch for the entrance to of only, 20 notes and t 10 notes. The total money
the ground in which the annual conventions is being value of the notes is ~ 230. The number of~ 1o
held. The profile of the arch follows the equation notes that Raju has is
(a) 5 (b) 6
y = 2x - 0.1 x2 where y is the height of the arch in
(c) 9 (d) 10
meters. The maximum possible height of the arch
[EE, EC 2012, 2 Marks (Set-2)]
is
(a) 8 meters (b) 1O meters 23. and B friends. They decide to meet between
,D.,
(c) i 2 meters (d) i4 meters 1 PM and 2 PM on a given day. There is a condition
[CE, ME, CS 2012, 2 Marks (Set-1 )] that whoever arrives first will not wait for the other
for more than i 5 minutes. The probability that they
18. An automobile plant contracted to buy shock
will meet on that day is
absorbers from two supplies X and Y. X supplies
(a) 1/4 (b) i/i6
60% and Y supplies 40% of the shock absorbers.
(c) 7/16 (d) 9/16
All shock absorbers are subjected to a quality
[Ef:,-EC-2CffZ;~-MarKS(Set=~m--
test. The ones that pass the quality test are
considered reliable. Of X's shock absorbers, 96% 24. The data given in the following tahle summarizes
are reliable. OF Y's shock absorbers, 72% are the monthly budget of an average household.
reliable. Category Amount
The probability that a randomly chosen shock Food 4000
e.bsorber, which is found to be reliable, is made
Clothing 1200
by Y is
Rent 2000
(a) 0.288 (b) 0.334
(c) 0.667 (d) 0.720
Savings 1500
[CE, ME, CS 2012, 2 Marks (Set-1 )] Others 1800
The approximate percentage of the monthly budget
19. Which of the following assertions are CORRECT?
NOT spent on savings is
P: Adding 7 to each entry in a list adds 7 to the (a) 10% (b) 14%
mean of the list (c) 81% (d) 86%
0: Adding 7 to each entry in a list adds 7 to the [EE, EC 20i 2, 2 Marks (Set~2)]
standard deviation of the list
R: Doubling each entry n a list doubles the mean 25. There are eight bags of rice looking alike, seven
of the list of which have equal and one is slightly heavier.
S: Doubling each entry in a list leaves the The weighing balance is of unlimited capacity.
standard deviation of the list unchanged Using this balance the minimum number of
weighting required to identify the heavier bag is
(a) P, 0 (b) 0, R
(a) 2 (b) 3
(c) P, R (d) R, S
(c) 4 (d) 8
[CE, ME, CS 2012, 2 Marks (Set~1)]
[EE, EC 2012, 2 Marks (Set-2)]
248 I ® Reasoning & Aptitude mRDE ERS~
26. A number much greater than 75 it is smaller than 31. What will be the maximum sum of 44,. 42, 40,
117 is ..... ?
(a) 91 (b) 93 (a) 502 (b) 504
(c) 89 (d) 96 (c) 506 (d) 500
[CE 2013, 1 Mark (Set-1)] [ME, Pl & CS/IT 2013, 1 Mark (Set-2)]
(a) 2, i (b) ;, 3
decrease by 1/24 of the current period, then the
new cost of erection in Rs., is
(c) 3' 3
2 1
(d) i, 3
(a) 16,500
(c) 11 , 000
(b) 15, i80
(d) 10, 120
[ME, Pl & CS/IT 2013, 2 Marks (Set-2)]
[CE 2013, 2 Marks (Set-1)]
36. In the summer of 2012, in New Delhi, the mean
30. x and y are two positive real numbers, such that
temperature of Monday to Wednesday was 41 °c
equation
and of Tuesday to Thursday was 43°c. If the
2X + y $ 6 ; X + 2y $ 8
temperature on Thursday was 15% higher than
For which values of (x, y), the function
that of Monday, then the temperature in °c on
f(x, y) = 3x + 6y will give maximum value Thursday was
(a) 4/3, 10/3 (b) 8/3, 20/3 (a) 40 (b) 43
(c) 8/3, 10/3 (d) 4/3, 20/3 (c) 46 (d) 49
[CE 2013, 2 Marks (Set-1)) [EE, EC, IN 2013, 1 Mark (Set-3)]
0 Previous Years GATE Solved Questions 249
37. A car travels 8 km in the first quarter of an hour, 43. Find the odd one from the following group:
6 km in the second quarter and 16 km in the third WEKO IOWA FNTX NVBD
quarter. The average speed of the car in km per (a) WEKO (b) IQWA
hour over the entire journey is (c) FNTX (d) NVBD
(a) 30 (b) 36 [EC, ME 2014, 2 Marks (Set-1 )]
(c) 40 (d) 24
44. For submitting tax returns, all resident males with
[EE, EC, IN 2013, 2 Marks (Set-3)]
annual income below Rs iO lakh should fill up Form
38. Find the sum to n terms of the series 10 + 84 P and all resident females with income below Rs 8
+ 734 + .... lakh should fill up Form Q. All people with incomes
above Rs i O lakh should fill up Form R, except
non residents with income above Rs 15 lakhs, who
should fill up Form S. All others should fill Form T.
An example of a person who should fill Form T is
9(9n -1) (d) 9(9n -1) + n2
(c) +n (a) a resident male with annual income Rs 9 lakh
8 8
(b) a resident female with annual income Rs 9 lakh
[EE, EC, IN 2013, 2 Marks (Set-3)) (c) a non-resident male with annual income Rs 16
lakh
39. The set of values of p for which the roots of the
(d) a non-resident female with annual income Rs
equation 3x 2 + 2x + p(p - i) = O are of opposite
161akh
sign is
[EC, ME 2014, 2 Marks (Set-1)]
(a) (-oo, 0) (b) (0, i)
(c) (1, oo) (d) (0, oo) 45. A train that is 280 metres long, travelling at a uniform
[EE, EC, IN 2013, 2 Marks (Set-3)] speed, crosses a platform in 60 seconds and passes
a man standing on the platform in 20 seconds. What
40. What is the change that a leap year, selected at
is the length of the platform in metres?
random, will contain 53 Saturdays?
,:._:___c__ _ _ _____:..__ _ _ _ _ __ c ' _ ,_ _ _ _ _ _ _ _ _ _ _ _ _1 Elc;.vll=ZOT4;-2-rvrarRs-(SeFiJr__ _
2 3
(a) (b) 46. The exports and imports (in r,roms of Rs.) of a
7 7
country from 2000 to 2007 are given in the
1 5 following bar chart. If the trade deficit is defined
(c) - (d) as excess of imports over exports, in which year
7 7
is the trade deficit i /5th of the exports?
[EE, EC, IN 2013, 2 Marks (Set-3)]
+---,·-i··
70 -t-----~-
Batsman Average Standard Deviation
50 · ·-
K 31.2 5.21 60
40
30
L 46.0 6.35 20
M 54.4 6.22 10
0
N 17.9 5.90 2000 2001 2002 2003 2004 2005 2006 2007
(a) K (b) L
(a) 2005 (b) 2004
(c) M (d) N
(c) 2007 (d) 2006
[EC, ME 2014, 1 Mark (Set-1 )]
(EC, ME 2014, 2 Marks (Set-1 )]
42. What is the next number in the series?
4 7. You are given three coins: one has heads on both
12 35 81 173 357 ---·
faces, the second has tails on both faces, and
[EC, ME 2014, 1 Mark (Set-i )]
the third has a head on one face and a tail on the
250 ® Reasoning & Aptitude MADE ERS~
Raw Material
20%
59. A firm producing air purifiers sold 200 units in 2012. 63. Find the next term in the sequence:
The following pie chart presents the share of raw 13M, 170, 19S, _.
. ,.
material, labour, energy, plant & machinery, and (a) 21W (b) 21V
transportation costs in the total manufacturing cost (c) 23W (d) 23V
of the firm in 2012. The expenditure on labour in [EC, ME 2014, 2 Marks (Set-4)]
2012 is Rs. 4,50,000. In 2013, the raw material 64. If 'KCLFTSB' stands for 'best of luck' and
expenses increased by 30% and all other 'SHSWDG' stands for 'good wishes', which of the
expenses increased by 20%. If the company following indicates 'ace the exam'?
registered a profit of Rs. 10 lakhs in 2012, at what (a) MCHTX (b) MXHTC
price (in Rs.) was each air purifier sold? (c) XMHCT (d) XMHTC
[EC, ME 2014, 2 Marks (Set-4)]
252 ® Reasoning & Aptitude fflRDE ERS~
Own vehicle
J
mRDE ERS!::§ e Previous Years GATE Solved Questions I 253
76. The ratio of male to female students in a college comparison, the GDP Is compared in US Dollars
for five years is plotted in the following line graph. (USD) after conversion based on the market
If the number of female students doubled in 2009, exchange rate. During the period 20i2-20i3 the
by what percent did the number of male students exchange rate for the USO increased from Rs. 50/
increase in 2009? USO to Rs.60/ USO. India's GDP in USO during
the period 2012-2013.
<0 3.5 ~-------.--.-.....,..-.'--~~-'--,,-,i (a) Increased by 5%
1
$
31-----------~----,,,.,_ (b) Decreased by 13%
o El 2.5 1--__..,::,...__~...........,_;_.,....,....,,..;.,,_;;,......;.,.,__;;..;
(c) Decreased by 20%
i~
ctl"O 2f------..,...,.,..~-~--
(d) Decreased by 11 %
_!§ ij 1.5 i------.,,..----'--,--"<;;--,-~-----::,......,.
0
[EE, CS 2014, 2 Marks (Set-3)]
ia:: 1
0.51----------"------'--
81. The ratio of male to female students in a college
o~ - - - - - - - - - -
2008 2009 2010 2011 2012 for five years is plotted in the following line graph.
If the number of female students in 2011 and 2012
is equal, what Is the ratio of male students in 2012
[EE, CS 2014, 2 Marks (Set-2)]
to male students in 2011?
77. At what time between 6 a.m. and 7 a.m. will the
minute hand and hour hand of a clock make an ID 3.5 , - , . . . ~ - - ~ - - ~ - - - ~
angle closest to 60°? I
'iii
3~---,-~~~~--~---
(a) 6:22 a.m. (b) 6:27 a.m. 0 El 2.5 1--_,:::___...,....;:,..__ _ _ _ __
- C:
(c) 6:38 a.m. (d) 6:45 a.m. ~~ 21-------~------
[EE, CS 2014, 2 Marks (Set-2)]
! i 1.5 i-,,.;....,___,.,--"-:-,,.,..;.._........,.,~__,..,.,~~---------,~
0
0
i
78. Which number does not belong in the series a: 0.5 i-------------
below? .o-,_;...;,--""'-=~~~~--~
2009 2010 . 2011 2012
~--__._,5r_i(l,_J]r26,-J."l,-_50,_6,;1,--_ _ _ _ _ _ _ _ _ _+--_,____,-....:~~~'-:---~~'-:-----------t- - - - -
(a) i7 (b) 37
(c) 64 (d) 26 (a) i : "I (b) 2 : 1
~
I
I /
.. ''
''
and so on). Anuj lives on an even-numbered floor. I I
'' '',
Bhola does not live on an odd numbered floor.
200
, ,
I
I
I/
I/
I
I
\'', ',, __
,,
'
. ·.....
/
100
Chandan does not live on any of the floors below , ,,,
(c) Statements i and 2 are both required to 97. A function f(x) is linear and has a value of 29 at
determine the eldest child. x == -2 and 39 at x = 3. Find its value at x = 5.
(d) Statements i and 2 are not sufficient to (a) 59 (b) 45
determine the eldest child. (c) 4-1 (d) 35
[CE, IN 2014, 2 Marks (Set-2)] [CE (Set-1), CS (Set-3) 2015, 1 Mark]
93. The total exports an9 revenues from the exports 98. The exports and imports (in crores of Rs.) of a
of a country are given in the two pie charts below. country from the year 2000 to 2007 are given in
The pie chart for exports shows the quantity of the following bar chart. In which year is the
each item as a percentage of the total quantity of combined percentage increase in imports and
exports. The pie chart for the revenues shows the exports the highest?
percentage of the total revenue generated through
export of each item. The total quantity of exports 120
110
D Exports !@llmports
f---
70 ,.
i per kilogram to the revenue generated through 60 <-- f--- ,__I• f---
..
export of Item 4 per kilogram?
50 --~·-- - I--
'····-· ~- - f- f-
20
10 -
,___
I---
,__
,__
>--1
I---, .
~
f-.
--1'
1---,..
I•
. _r~ ~
f-
<
0
2000 2001 2002 2003 2004 2005 2006 2007
[CE (Set-1 ), CS (Set-3) 2015, 1 Mark) that if S gets either Power or Telecom, then she
256 ® Reasoning & Aptitude fflRDE EASY
must get the other one. T insists on a portfolio 104. How many four digit numbers can be formed with
if P gets one. the 10 digits 0, 1, 2, ..... 9 if no number can start
Which is the valid distribution of portfolios? with O and if repetitions are not allowed?
(a) P-Home, O-Power, A-Defence, S-Telecom, [CE 2015, 2 Marks (Set-2))
T-Finance
105. The given question is followed by two statements;
(b) A-Home, S-Power, P-Defence, O-Telecom,
select the most appropriate option that solves the
T-Finance
question
(c) P-Home, O-Power, FDefence, S-Telecom,
Capacity of a solution tank A is 70% of the
U-Finance
capacity of tank B. How many gallons of solution
(d) O-Home, U-Power, T-Defence, A-Telecom,
are in tank A and tank B?
P-Finance
Statements:
[CE (Set-1 ), CS (Set-3) 2015, 2 Ma;rks]
I. Tank A is 80% full and tank B is 40% full.
i01. Four cards are randomly selected from a pack 11. Tank A if full contains 14,000 gallons of
of 52 cards. If the first two cards are kings, what
solution.
is the probability that the third card is a king?
(a) Statement I alone is sufficient.
4 2 (b) Statement II alone is sufficient.
(a) 52 (b) 50
(c) Either statement I or 11 alone is sufficient.
(d) Both the statements I and II together are
(c) Type II (d) Type I (II) The total weight of these poles is 160 kg more
than the total weight of two poles.
[CE 2015, 2 Marks (Set-2))
49 Previous Years GATE Solved Questions I 257
(a) Statement I alone is not sufficient 113. Base on the given statements, select the most
(b) Statement II alone is not sufficient appropriate option to solve the given question. If
(c) Either I or II alone is sufficient two floors in a certain building are 9 feet apart,
(d) Both statement I and II together are not sufficient how many steps are there in a set of stairs that
[CS (Set-1), EE (Set-2) 2015, 1 Mark] extends from the first floor to the second floor of
the building?
109. If the list of letters, P, R, S, T, U is an arithmetic
Statements:
sequence, which of the following are also in
(I) Each step is 3/4 foot high.
arithmetic sequence?
(II) Each step is i foot wide.
(I) 2P, 2R, 2S, 2T, 2U
(a) Statement-I alone is sufficient, but statement-
(11) P-3, R-3, S-3, T-3, U-3
II alone is not sufficient.
(II) P2, R2, S2-, T2, IJ2
(b) Statement-II alone is sufficient, but statement
(a) I only (b) I an II
I alone is not sufficient.
(c) II and Ill (d) I and Ill
(c) Both statements together are sufficient, but
[CS (Set-i ), EE (Set-2) 2015, 1 Mark]
neither statement alone is sufficient.
110. If p, q, r, s are distinct integers such that (d) Statement I and II together are not sufficient.
~p, q, r, s) = max(p, q, r, s) [CS (Set-2), EE (Set-1) 2015, 1 Mark]
g(p, q, r, s) = min(p, q, r, s)
h(p, q, r, s) = remainder of (p x q)/(r x s) if
i 14. Given Set A= /2. 3, 4, 5} Set B = {1 i, 12, i3, 14,
(p x q) > (r x s) or remalnder of (r x s)/(p x q) it i 5}, two number: 1re randomly selected, one from
each set. What is the probability that the sum of
(r x s) > (p x q)
the two numbers are equals 16?
Also a function fgh(p, q, r, s) = ~p, q, r, s) x g(p,
(a) 0.20 (b) 0.25
q, r, s) x h(p, q, r, s)
(c) 0.30 (d) 0.33
Also the same operations are valid with two variable
function of the form ~p, q) [CS (Set-2), EE (Set-1) 2015, 1 Mark]
_ _ _ _W_ha~t_is_th_e_v_a_lu_e_o-::---f_,,fg~('--h_,_(2-'-,_5'--,7-c-'-,_3"--),_4_:_.,_6-'--,8::-'):--?-:.--=-__115___ _Tbapie..charlbetowbas..:tl:+e-br::eak-Yp-0f-th&FH::1mtlfPe""-r- - - -
[CS (Set-1 }, EE (Set-2) 2015, 2 Marks] of students from different departments in an
111. Four branches of a company are located c=it M, N, engineering college for the year 2012. Tho
0, and P. Mis north of Nat a distance of 4 km; P proportion of male to female students in each
is south of Oat a distance of 2 km; N is southeast department is 5: 4. There are 40 males in Electrical
of O by 1 km. What is the distance between M Engineering. What is the difference between the
and Pin km? numbers of female students in the Civil
(a) 5.34 (b) 6.74 department and the female students in the
(c) 28.5 45.49
(d) Mechanical department?
[CS (Set-1 ), EE (Set-2) 2015, 2 Marks)
(q+r) qr
(a) 116. The probabilities that a student passes in
qr (b) (q+r)
Mathematics, Physics and Chemistry are m, p,
(q+r)2 and c respectively. Of these subjects, the student
(c) ✓(c/+r2) (d) qr
has 75% chance of passing in at least one, a 50%
[CS (Set-1), EE(Set-2}2015, 2 Marks} chance of passing in at least two and a 40%
258 ® Reasoning & Aptitude ffllRDE ERS!:i
chance of passing in exactly two. Foliowing 121. Fill in the missing value
relations are drawn in m, p, c:
(I) p+m+c=27/20
(II) p+m+c=13/20
(Ill) (p) x (m) x (c) = 1/10
(a) Only relation I is true
(b) Only relation II is true
(c) Relations II and Ill are true
(d) Relations I and Ill are true
[CS (Set-2), EE (Set-i) 2015, 2 Marks]
[EC 2015, 2 Marks (Set-1)]
117. The number of students in a class who have
answered correctly, wrongly, or not attempted each 122. An electric bus has onboard instruments that report
question in an exam, are listed in the table below. the total electricity consumed since the start of the
The marks for each question are also listed. There trip as well as the total distance covered. During
is no negative or partial marking. a single day of operation, the bus travels on
stretches M, N, 0 and P, in that order. The
Answered Answered Not
QNo. Marks correctly
Wrongly Attempted
cumulative distance travelled and the
corresponding electricity consumption are shown
1 2 21 17 6
in the Table below:
2 3 15 27 2
3 1 11 29 4 Stretch Cumulative distance Electricity used
4 2 23 18 3 (km) (kWh)
5 5 31 12 1 M 20 12
-- N 45 25
What is the average of the marks obtained by the
class in the examination? 0 75 45
(a) 2.290 (b) 2.970 p 100 57
(c) 6.795 (d) 8.795
[CS (Set-2), EE (Set-i) 2015, 2 Marks] The stretch where the electricity consumption per
km is minimum is
118. If logx (5/7) =- 1/3, then the value of x is
(a) M (b) N
(a) 343/125 (b) 125/343
(c) 0 (d) P
(c) -25/49 (d) -49/25
[EC (Set-2), ME (Set-3) 2015, 1 Mark]
[EC 2015, 1 Mark (Set-1 )]
123. Ram and Ramesh appeared in an interview for
119. Operates □, ◊ and ➔ are defined by:
two vacancies in the same department. The
a-b a+b . probability of Ram's selection is 1/6 and that of
a □ b=--; a◊b=--; a ➔ b= ab. Find
a+b a-b Ramesh is 1/8. What is the probability that only
one of them will be selected?
the value of ( 66 □ 6) ➔ (66 0 6) (a) 47/48 (b) 1/4
(a) -2 (b) -1 (c) 13/48 (d) 35/48
(c) i (d) 2 (EC (Set-2), ME (Set-3) 2015, 1 Mark]
[EC 2015, 1 Mark(Set-1)]
124. Given below are two statements followed by two
120. A cube side 3 units is formed using a set of conclusions. Assuming these statements to be
smaller cubes of side 1 unit. Find the proportion true, decide which one logically follows.
of the number of faces of the smaller:cubes
Statements:
visible to those which are NOT visible.
(a) i : 4 (b) i : 3 I. All film star are playback singers.
(c) i : 2 (d) 2 : 3 II. All film directors are film stars.
[EC 2015, 2 Marks (Set-1)]
mRDE ERS~ @ Previous Years GATE Solved Questions I 259
125. If a2 + b2 + c = ·1, then ab + be + ac lies in part is used to make a conical surface, then the
the tiger and the deer cover 8 metre and 5 metre will have to held to complete the league round of
matches?
per leap respectively, what distance in metres will
(a) 20 (b) iO
the tiger have to run before it catches the deer?
(c) 8 (d) 5
[EC (Set-2), ME (Set-3) 2015, 2 Marks]
[ME (Set-1 ), IN & Pl 2015, 1 Mark]
127. If x > y > 1, which of the following must be true?
(i) /nx>lny (ii) e'>eY 133. TanyaisolderthanEric.
(iii) yx > xY (iv) cosx > cosy Cliff is older than Tanya.
Eric is older than Cliff.
_ _ _____,(=a) (iJancL(ii.L--_ _(b.UlLandJjii.,___ _ _ _ _ _ _ _~ ~ ~ - - - - - - - - - - - - - - - -
(c) (iii) and (iv) (d) (ii) and (iv) If the first two statements are true, then the third
[EC (Set-3), MF (Set-2) 2015, 1 Mark] statement is
(a) True (b) False
128. Find the missing sequence in the letter series (c) Uncertain (d) Data insufficient
below: [ME (Set-1), IN & Pl 2015, 1 Mark]
A, CD, GHI?, UVWXY
134. In the given angle O is a right angle,
(a) UviN (b) MNO
PS : OS= 3 : 1, RT: QT= 5 : 2 and PU: UR= 1 : 1.
(c) MNOP (d) NOPQ
If area of triangle QTSis 20 cm 2 , then the area of
[EC (Set-3), ME (Set-2) 2015, 1 Mark] triangle PQR in cm 2 is __ .
129. Ms. X will be Bagdogra from 01/05/2014 to 20105/
R
2014 and from 22/05/2014 to 31/05/2014. On the
morning of 21/05/2014, she will reach Kochi via
Mumbai.
Which one of the statements below is logically
valid and can be inferred from the above
sentences? p ~-------""--__J Q
(a) Ms. X will be in Kochi for one day, only in May. s
(b) Ms. X will be in Kochi for only one day in May. [ME (Set-1 ), IN & Pl 2015, 2 Marks]
(c) Ms. X will be only in Kochi for one day in May.
(d) Only Ms. X will be in Kochi for one day in May. 135. Given below are two statements followed by two
conclusions. Assuming these statements to be
[EC {Set-3), ME (Set-2) 2015, 2 Marks]
true, decide whicl-1 one iogicaiiy follows:
260 rt Reasoning & Aptitude fflRDE ERS!::ll
one of the following statements is TRUE? chess. Pavithra wins more often than Leela does.
(a) X and Y are not independent Which one of the following statement must be
(b) Yand Z are dependent TRUE based on the above?
(c) Yand Zare independent (a) When Shiva plays chess with Leela and
(d) Xand Zare independent Pavithra, he often loses.
[ME (Set-1 ), IN & Pl 2015, 2 Marks] (b) Leela is the oldest of the three
(c) Shiva is a better chess player than Pavithra
137. Right triangle POR is to be constructed in the (d) Pavithra is the youngest of the three
xy-plane so that the right angle is at Pand line PR
[EC, ME 2016, 2 Marks (Set-1 )]
is parallel to the x-axis. The x and y coordinates
of P, 0, and Rare to be integers that satisfy the
inequalities: 142. If q
-a - 1
- r and r -b = s1 and S-c = q,1 the value of
-4 sx s 5 and 6 sys 16. How many different
triangle could be constructed with these abc is. _ __
properties? (a) (rps)- 1 (b) 0
(a) iiO (b) i 100 (c) 1 (d) r + q + s
(c) 9900 (d) 10000 [EC, ME 2016, 2 Marks (Set-1)]
[ME (Set-1 ), IN & Pl 2015, 2 Marks] 143. P, 0, Rand Sare working on a project. O can
finish the task in 25 days, working alone for
138. In a huge pile of apples and oranges, both ripe
and unripe mixed together, 15% are unripe fruits.
i 2 hours a day. R can finish the task in 50 days,
working alone for i 2 hours per day. O worked
Of the unripe fruits, 45% are apples. Of the ripe
ones, 66% are oranges. If the pile contains a total i 2 hours a day but took sick leave in the beginning
of 5692000 fruits, how many of them are apples? for two days, Rworked 18 hours a day on all days.
(a) 2029198 (b) 2467482 What is the ratio of work done by O and R after
(c) 2789080 (d) 3577422 7 days from the start of the project?
[EC, ME 2016, 1 Mark (Set-1 )] (a)iO:ii (b)ii:10
(c) 20 : 21 (d) 21 : 20
139. Michael lives i O km away from where I live. Ahmed
[EC, ME 2016, 2 Marks (Set-1 )]
lives 5 km away and Susan lives 7 km away from
where I live. Arun is farther away than Ahmed but 144. Given (9 inches) 112 = (0.25 yards) 112 , which one of
closer than Susan from where I live. From the the following statement is TRUE?
information provided here, what is one possible (a) 3 inches = 0.5 yards
distance (in km) at which I !ive from Arun's place? (b) 9 inches= 1.5 yards
® Previous Years GATE Solved Questions I 261
(c) 9 inches= 0.25 yards 149. A wire of length 340 mm is to be cut into two parts.
(d) 81 inches= 0.0625 yards One of the parts is to be made into a square and
[EC (Set-2), ME (Set-3) 2016, 1 Mark] the other into a rectangle where sides are in the
ratio of 1 : 2. What is the length of the side of the
145. S, M, E and Fare working in shifts in a team to.
square (in mm) such that the combined area of
finish a project. Mworks with twice the efficiency
the square and the rectangle is a MINIMUM?
of others but for half as many days as Eworked.
(a) 30 (b) 40
S and M have 6 hour shifts in a day, whereas E
(c) 120 (d) 180
and F have 12 hours shifts. What is the ratio of
[EC (Set-2), ME (Set-3) 2016, 2 Marks]
contribution of Mto contribution of Ein the project?
(n) 1 : 1 (b) 1 : 2 "l 50. The number that least tits this set:
(c) 1 : 4 (d) 2: 1 (324, 441, 97 and 64) is _ __
[EC (Set-2), ME (Set-3) 2016, 1 Mark] (a) 324 (b) 441
(c) 97 (d) 64
146. The Venn diagram shows the preference of the
student population for leisure activities.
[EC (Set-3) & IN 2016, 1 Mark]
i 48. Mand N start from the same location. M travels (a) O (b) 3
i O km East and then 10 km North-East. N travels (c) 4 (d) 5
5 km South and then 4 km South-East. What is [EC (Set-3) & IN 2016, 2 Marks]
the shortest distance (in km) between Mand Nat
153. A flat is shared by four first year undergraduate
the end of their level.
students. They agreed to allow the oldest of them
(a) 18.60 (b) 22.50
to enjoy some extra space in the fiat. Manu is two
(c) 20.6i (d) 25.00
months older than Sravan, vvho is three months
[EC (Set~2), ME (Set-3) 2016, 2 Marks]
younger than Trideep. Pavan is one month older
262 ® Reasoning & Aptitude fflRDE ERS'!:::1
than Sravan. Who should occupy the extra space to Ponly sometimes IV. R always loses to 0. Which
in the flat? of the following can be logically inferred from the
!
(a) Manu (b) Sravan above statements? (i) Pis likely to beat all the 1
l
(c) Trideep (d) Pavan three other players (ii) Sis the absolute worst player lI
[EC (Set-3) & IN 2016, 2 Marks] in the set l
!
154. Find the area bounded by the lines 3x + 2y == 14,
(a) (i) only (b) (ii) only ''I
j
(c) (i) and (ii) (d) neither (i) nor (ii) ll
2x - 3y = 5 in the first quadrant.
(a) 14.95 (b) 15.25
[CE, CS 2016, 2 Marks (Set-1)] I
(c) 15.70 (d) 20.35 159. If f(x) = 2x7 + 3x - 5, which of the following is a I
[EC (Set-3) & IN 2016, 2 Marks] factor f(x) ?
155. A straight line is fit to a data set (ln x, y). This line (a) (x3 + 8)
(c) (2x- 5)
(b)(x-1) I
intercepts the abscissa at lnx == 0.1 and has a slope (d) (x + 1)
of -0.02. What is the value of y at x = 5 from the [CE, CS 2016, 2 Marks (Set-1))
fit?
160. In a process, the number of cycles to failure
(a) -0.030 (b) -0.014
decreases exponentially with an increase in load.
(c) 0.014 (d) 0.030
At a load of 80 units, it takes 100 cycles for failure.
[EC (Set-3) & IN 2016, 2 Marks]
When the load is halved, it takes 10000 cycles for
156. A cube is built using 64 cubic blocks of side one failure. The load tor which the failure will happen
unit. After it is built, one cubic block is removed in 5000 cycles is
from every corner of the cube. The resulting (a) 40.00 (b) 46.02
surface area of the body (in square units) after (c) 60.01 (d) 92.02
the removal is _ _ __ [CE, CS 2016, 2 Marks (Set-1 )]
(a) 56 (b) 64
(c) 72 (d) 96 161. In a quadratic function, the value of the product
[CE, CS 2016, 1 Mark (Set-1 )] of the roots (o:, ~) is 4. Find the value of
Quarter/
162. Pick the odd one from the following options.
Elegance Smooth Soft Executive
Product (a) CADBE (b) JHKIL
'
01 27300 20009 17602 9999 (c) XVYWZ (d) ONPMQ
02 25222 19392 18445 8942 [EE (Set-1 ), CS (Set-2) 20'16, 1 Mark]
03 28976 22429 19544 10234
04 21012 18229 16595 10109
163. Among 150 faculty members in an institute, 55
are connected with each other through Facebook®
Which product contributes the greatest fraction and 85 are connected through WhatsApp® 30
to the revenue of the company in that year? faculty members do not have Facebook® or
(a) Eleganct:l (b) Executive WhatsApp® accounts. The number of faculty
(c) Smooth (d) Soft members connected only through Facebook®
[CE, CS 2016, 2 Marks (Set-1 )] accounts is
158. Consider the following statements relating to the (a) 35 (b) 45
level of poker play of four players P, Q, Rand S. I. (c) 65 (d) 90
P always beats O 11. R always beats S 111. S loses [EE (Set-1), CS (Set-2) 2016, 2 Marks]
I
?
@ Previous Years GATE Solved Questions I 263
I I I I i 4
(a) 21 (b) 27
If the amounts invested in the companies, P and
~~ ~)~
Q in 2006 are in the ratio 8 : 9, then the amounts
[EE (Set-1), CS (Set-2) 2016, 2 Marks]
received after one year as interest from companies
165. ChoosA thp, correct expression for f(x) givon in tho P Elnd O would be the I aliu
graph. (a) 2: 3 (b) 3: 4
(c)6:7 (d)4:3
[CE 2016, 2 Marks (Set-2)]
(c) f(x) =2 - Ix_ 11 (rl) f(x) = 2 + Ix_ 11 the slant height is half of the perirr1eter. What is
[EE (Set-1 ), cs (Set-2) 2016, 2 Marks] the lateral surface area of the pyramid?
(a) x2 (b) 0.75x2
166. (x% of y) + (y<'lo of x) is equivalent to __ . (c) 0.50x2 (d) 0.25x2 r,·
(a) ~% ot xy (b) 2% of (xy/100) [CE 2016, 2 Marks (Set-2)]
(c) xy% of 100 (d) 100% of xy
[CE 2016, 1 Mark (Set-2)] 171. Ananth takes 6 hours and Bharath takes 4 hours
to read a book. Both started reading copies of
167. The sum of the digits of a two digit number is
the book at the same time. After how many hours
12. If the new number formed by reversing the
is the number of pages to be rad by Ananth,
digits is greater than the original number by 54,
twice that to be read by Bharath? Assume Ananth
find the original number.
and Bharath read all the pages with constant
(a) 39 (b) 57
pace.
(c) 66 (d) 93
(a) i (b) 2
[CE 2016, 1 Mark (Set-2)]
(c) 3 (d) 4
168. Two finance companies, P and O declared fixed [CE 2016, 2 Marks (Set-2)]
annual rates of interest on the amounts invested
172. Pick the odd one out in the following:
with them. The rates of interest offered by these
13,23,33,43,53
companies may differ from year to year. Year-
(a) 23 (b) 33
wise annual rates of interest offered by these
(c) 43 (d) 53
companies are shown by the line graph provided
[EE 2016, 1 Mark (Set-2)]
below.
264 I ® Reason·ing & Aptitude fflRDE ERS~
'82. Which of the following curve represents the 184. Consider the following sentences:
function y ::: ln(IJsin (lxl)] I) All benches are beds. No bed is a bulb. Some
bulbs are lamps.
for Ix I< 2 n?
Which of the following can be inferred?
Here, x represents the abscissa and yrepresents
(i) Some beds are lamps.
the ordinate.
(ii) Some lamps are beds.
(a) Only (i) (b) Only (ii)
(c) Both (i) and (ii) (d) Neither (i) nor (ii)
[CE20i7, i Mark(Set-1)]
20 C]Bed
18 C=:]Table
-2re -re 0 re 2n
E16 j;-jChair
~ 14
Cl)
:i 12
:t:'.
[ME 2016, 2 Marks (Set-2)] E 10
2
'o,_ 8
183. The following sequence of numbers is arranged Cl)
.n 6
in increasing order: 1, x, x, x, y, y, 9, 16, 18. Given E
:,
z 4
that the mean and median are equal, and are also
2
equal to twice the mode, the value of y is
0
(a) 5 (b) 6 Ci C2 C3 C4 C5
(c) 7 (d) 8 Carpenter ( C)
' I I
' I I
3 r---+-----!-<·Xl<l·*'-+----+--f---:,-+-----<,>-· ~l0----+----+-1
+--:
188. Students applying for hostel rooms are alloted ' '
'
'
I
2r----+---,--,-~--~,-,--T~-~:----+---+-~
rooms in order of seniority. Students already '
''
staying in a room will move if they get a room
· in their preferred list. Preferences of lower ranked o
applicants are ig11ored during allocation. 0 25 10 15 20 25 30 35 40 45 50 55 60
Time (min)
Given the data below, which room will Ajit stay in?
Which of the following statements are correct?
Room
Student Current (i) The elevator never moves directly from any
Names Seniority preference
room
list non-ground floor to another non-ground floor
Amar 1 p R,S,Q over the one hour period.
Akbar 2 None R,S (ii) The elevator stays on the fourth floor for the
Anthony 3 Q p longest duration over the one hour period.
Ajit 4 s Q,P, R (a) Only (i) (b) Only (ii)
(c) Both (i) and (ii) (d) Neither (i) nor (ii)
(a) p (b) Q [CE, IN 2017, 2 Marks (Set-2)]
(c) R (d) S
193. Budhan covers a distance of i9 km in 2 hours ;I
[CE 2017, 2 Marks (Set-1)] by cycling one fourth of the time and walking the
189. The last digit of (2i7i) 7 + (2172) 9 + (2173) 11 + rest. The next day he cycles (at the same speed
(2174) 13 is as before) for half the time and walks the rest ( at
(a) 2 (b) 4 the same speed as before) and covers 26 km in
(c) 6 (d) 8 2 hours. The speed in km/h at which Budhan
walks is
[CE 2017, 2 Marks (Set-1)]
(a) 1 (b) 4
(c) 5 (d) 6
190. Two dice are thrown simultaneously. The probability
[CE, IN 2017, 2 Marks (Set-2)]
that the product of the numbers appearing on the
top faces of the dice is a perfect square is 194. A map shows the elevations of Darjeeling.
Gangtok, Kalimpong, Pelling and Siliguri.
1 2
(a) ·-- (b) - Kalimpong is at a lower elevation than Gangtok.
9 9
Pelling is at a lower elevation than Gangtok.
1 4 Pelling is at a higher elevation than Siliguri.
(c) (d) - Darjeeling is at a higher elevation than Gangtok.
3 9
Which of the following statements can be inferred
[CE, IN 2017, 1 Mark (Set-2)]
from the paragraph above?
® Previous Years GATE Solved Questions I 267
(i) Pelling is at a higher elevation that Kalimpong. (a) 120 (b) 144
(ii) Kalimpong is at a lower elevation than (c) 160 (d) 212
Darjeeling. [EC 2017, 1 Mark (Set-1)]
(iii) Kalimpong is at a higher elevation thc;l.n Siliguri.
199. Trucks (10 rn long) and cars (5 m long) go on a
(iv) Siliguri is at a lower elevation than Gangtok.
single lane bridge. There must be a gap of at least
(a) Only (ii) (b) Only (ii) and (iii)
20 m after each truck and a gap of at least 15 m
(c) Only (ii) and (iv) (d) Only (iii) and (iv)
after each car. Trucks and cars travel at a speed
[CE, IN 2017, 2 Marks (Set-2)]
of 36 km/h. If cars and trucks go alternately. What
195. P, Q, R, S, T and U are seated around a circular is the maximum number of vehicles that can use
table. R is seated two places to the right of 0, the bridqe in one hour?
Pis seated three places to the left R. Sis seated (a) 1440 (b) 1200
opposite U. If P and U now switch seats, which (c) 720 (d) 600
of the following must necessarily be true? [EC 2017, 2 Marks (Set-i)]
(a) P is immediately to the right of R
200. A contour line joins locations having the same
(b) Tis immediately to the left of P
height above the mean sea level. The following is
(c) T is immediately to the left of P or P is
a contour plot of a geographical region. Contour
immediately to the right of Q
lines are shown at 25 m intervals in this plot.
(d) U is immediately to the right of R or P is
immediately to the left of T
[CE, IN 2017, 2 Marks (Set-2)]
203. A rule states that in order to drink beer, one Which of the following is the steepest path leaving
must be over i8 years old. In a bar there are from P?
4 people. Pis 16 years old, Q is 25 years old, (a) Pto Q (b) Pto R
Ris drinking milkshake and Sis drinking a beer. (c) Pto S (d) Pto T
What must be checked to ensure that the rule [EC 2017, 2 Marks (Set-2)]
is being followed?
207. 1200 men and 500 women can build a bridge
(a) Only P's drink
in 2 weeks, 900 men and 250 women will take
(b) Only P's drink and S's age 3 weeks to build the same bridge. How many
(c) Only S's age men will be needed to build the bridge in one
(d) Only P's drink. Q's drink and S's age week?
[EC 2017, 1 Mark (Set-2)] (a) 3000 (b) 3300
(c) 3600 (d) 3900
204. Fatima starts from point P, goes North for 3 km
[EC 2017, 2 Marks (Set-2)]
and then East for 4 km to reach point 0. She
then turns to face point P and goes 15 km in 208. Each of P, 0. R. S, W, X, Y and Z has been
that direction. She then goes North for 6 km. married at most once. X and Y are married and
How far is she from point P and in which direction have two children P and 0. Z is the grandfather
should she go to reach point P? of the daughter S of P. Further, Z and W are
(a) 8 km, East (b) 12 km, North married and are parents of R. Which one of the l
(c) 6 km, East (d) 10 km, North following must necessarily be FALSE? I
[EC 2017, 1 Mark (Set-2)] (a) Xis the mother-in-law of R I
1
(b) P and Rare not married to each other
205. 500 students are taking one or more courses out
of Chemistry, Physics and Mathematics.
(c) Pis a son of Xand Y tI
(d) 0 cannot be married to R i
Registration records indicate course enrolment
as follows: Chemistry (329), Physics ( 186),
Mathematics (295), Chemistry and Physics (83),
[EC 2017, 2 Marks (Set-2)]
(c) Srinivas and Murali 217. There are 3 red socks, 4 green socks and 3 blue
(d) Srinivas and Rahul socks. You choose 2 socks. The probability that
[EE, CS 20i7, 1 Mark (SeH)] they are of the same colour is
(a) i/5 (b) 7/30
2i 3. A contour line joins locations having the s8me
(c) 1/4 (d) 4/15
height above the mean sea level. The following
[EE, CS 2017, 1 Mark (Set-2)]
is a contour plot of a geographical region. Contour
lines are shown at 25 m intervals in this plot. If 218. There are five buildings called V, W. X, Y and Z
in a flood, the water level rises to 525 m, which in a row (not necessarily in that order). V is to
of the villages P, Q, R, S, T get submerged? the West of W. Z is to the East of X and the West
of V. W is to the West of Y. Which is the building
in the middle?
(a) V (b) W
(c) X (d) y
[EE, CS 2017, 1 Mark (Set-2)]
l
(c) R (d) S (c) 3 (d) Cannot be determined
[EE, CS 2017, 2 Marks (Set-2)] [ME, 2017, i Mark (Set-1)]
It
222. Xis a 30 digit number starting with the digit 4 226. In a company with iOO employees, 45 earn Rs.
followed by the digit 7. Then the number X3 will 20000 per month, 25 earn Rs. 30000, 20 earn Rs.
have 40000, 8 earn Rs. 60000, and 2 earn Rs. 150000.
(a) 90 digits (b) 91 digits The median of the salaries is I
(c) 92 digits (d) 93 digits
[EE, CS 2017, 2 Marks (Set-2)]
(aj Rs.20000
(c) Rs. 32300
(b) Rs.30000
(d) Rs. 40000 I
,,
223. There are three boxes. One contains apples,
another contains oranges and the last one contains
[ME, 2017, 1 Mark(Set-1)]
ll
227. What is the sum of the missing digits in the
both apples and oranges. All three are known to subtraction problem below? l
be incorrectly labelled. If you are permitted to I
open just one box and then pull out and inspect
5......... .. l
-48 89
only one fruit, which box would you open to
1i1i
determine the contents of all three boxes?
(a) 8 (b) 10
(a) The box labelled 'Apples'
(c) 11 (d) Cannot be determined
(b) The box labelled 'Apples and Oranges'
[ME, 2017, 2 Marks (Set-1)]
(c) The box labelled 'Oranges'
(d) Cannot be determined
[EE, CS 2017, 2 Marks (Set-2)]
228. Let S1 be the plane figure consting of the points
t-- ll'-
[ME, 2017, 2 Marks (Set-2)]
:::, ,
0 0. 3 l-+----+-+-1-+---'.,.,<-+--+-+---<--<-A--+-+-+--+---+---1-+--+-
l' 235. There are 4 women P, 0, R, Sand 5 men V, W,
o.. 0 .2 l-!--e-+-!_,._,,(-1-J-f--+--h,-,A-+-4-+-+-f--l---'--+-!-t-J
ILJ iVJ X, Y, Z in a group. We are required to form pairs
0 .1 "'
l-l---l--1---IL..,i:n:./------1,..~
each consisting of one woman and one man. P
~ r a. . ~H:fi::t:t=t=tt1ttttt::t=1::::i-- is not to be paired with Z, and Y must necessarily
0 20 40 60 80 100 120 140 160 180 200 be paired with someone. In how many ways can
Time (min)
4 such pairs be formed?
(a) 74 (b) 76
Consider the following statements based on
(c) 78 (d) 80
thA data shown above:
[ME, 2017, 2 Marks (Set-2)]
(i) The growth in bacterial population stops earlier
at 37°C as compared to 25°C 236. In the graph below, the concentration of a
(ii) The time taken for curd formation at 25°C is particular pollutant in a lake is plotted over
twice the time taken at 37°C (alternate) days of a month in winter (average
Which one of the following option is correct? temperature i 0°C) and a month in summer
(a) only i (b) only ii (average temperature 30°C)
11 r!
(c) both i and ii (d) neither i nor ii
I I I I I I I I I I I I i I
i \
I
' i J ' -¢-
~~
i I
[ME, 2017, 2 Marks (Set-1)] 10 -El-- Winter
' !~ II / ..
-9: 8 ! '
of the following must always be even? C
,g 7 f-
i
' /1 ,µ i:J-i;f
(a)ab (b)a 2 +b2 +1
[::
1: 6
I-
I vrr
(!)
( C) a2 +b+1 (d) ab - b g 5 - I!
[ME, 2017, 1 Mark (Set-2)]
0
u 4
~
,r ,.
c V
ill 3
~~~vr
2
32. P looks at Q while Q looks at R, P is married, ~ 2 I-,~
R is not. The number of pairs of peopte in which ~r
P.,:r r . -
0
a married person is looking at an unmarried person L-J~LiJ___L_l__J_ I
LJ
0 2 4 6 8 10 12 14 16 18 20 22 24 26 28 30
is
Day of the month
272 ® Reasoning & Aptitude fflRDE ERS!:j
(a) deteriorating Had he sold it for Rs. i08 more, he would have
(b) arguing
made a i 0% gain. What is the loss in Rupees
(c) departing (d) splitting
incurred by the fruit seller?
[ME, 2017, 2 Marks (Set-2)]
(a) 48 (b) 52
239. The temperature Tin a room varies as a function (c) 60 (d) 108
of the outside temperature T0 and the number of [CE, 2018, 2 Marks (Set-1 )]
persons in the room p, according to the relation 244. Each of the letters arranged as below represents
T = K(0p + T0 ), where 0 is Kare constants. What a unique from 1 to 9. The letters are positioned
would be the value of 0 given the following data? in the figure such that (A x B x C), (B x G x E)
e Previous Years GATE Solved Questions I 273
E
249. a+ a+ a+ ... + a= a2b and
F
ntimes
(a) 4 (b) 5
(c) 6 (d) 9 b+ b+ b+ ... + b = ab2, where a, b, n and m
[CE, 2018, 2 Marks (Set-1)] mtimes
245. The price of a wire made of a superalloy material are natural numbers. What is the value of
is proportional to the square of its length. I he
price of IO m length of the wire is Rs. 1600. What (
m+ m+ m+ ... + m)(n + n+ n+ ... + n)?
nt1mes mt1mes
would be the total price (in Rs.) of two wires of
(a) 2cib2 (b) Eftf
lengths 4 m and 6 m?
(c) ab(a + b) (d) a2 + b 2
(a) 768 (b) 832
[CE, 2018, 1 Mark (Set-2)]
(c) 1440 (d) 1600
[CE, 2018, 2 Marks (Set-1)] 250. In manufacturing industries, loss is usually taken
to be proportional to the square of the deviation
246. What of the following function(s) in an accurate
from a target. If the loss is Rs. 4900 for a deviation
description of the graph for tr,e range(s) indicated?
of 7 units, what would be the loss in Rupees for
a deviation of 4 units from the target?
: : y : :
I I l
----r---r---1--j ----r---r---1---
l I l
(a) 400 (b) 1200
1 ! I ! I I
I ! I ! I I (c) 1600 (d) 2800
----1----L--- --- ----1----L----l.---
: : I 2 : [ : [CE, 2018, 2 Marks (Set-2)]
I I
____ 1____ L_ _ lI ____ ____ 1I __ _
I I
l I I I
I I
I
I
I
251. Each of the letters in the figure below represents
- :
I
1
I
=1r-- ~~---
I I I I
X
a unique integer from 1 to 9. The letters are
----~- -~---+--- ----~---~---+---
: : : -1 : : : positioned in the figure such that each of
I J I l I I
--- ---L---~--- ----L---L---~--- (A+B+C), (C+D+E), (E+F+G) and (G+H+K) is
:t :I -2 :I :I :t
----:----: ---:--- ----: ---: --- :--- equal to 13. Which integer does E represent?
: : : -3 : :
l I I I
(a) y = 2x + 4 for -3 s x s - 1 IA B C
D
(b) y = Ix - 1I for - 1 s x s 2
E F G
(c) y = x 1I for -1 s x s 2
11 I - H
K
(d) y = i for 2 s x s 3 -
(a) (i), (ii) and (iii) only (b) (i), (ii) and (iv) only (a) 1 (b) 4
(c) (i) and (iv) only (d) (ii) and (iv) only (c) 6 (d) 7
[CE, 2018, 2 Marks (Set-1 )] [CE, 2018, 2 Marks (Set-2)]
247. A three-member committee has to be formed 252. The annual average rainfall in a tropical city is
from a group of 9 people. How many such distinct 1000 mm. On a particular rainy day
committees can be formed? (24-hour period), the cumulative rainfall
(a) 27 (b) 72 experienced by the city is shown in the graph.
(c) 81 (d) 84 Over the 24-hour period. 50% of the rainfall falling
[CE, 2018, 1 Mark (Set-2)] on a rooftop, which had an obstruction-free area
of 50 m 2 , was harvested into a tank. What is the
248. For non-negative integers, a, b, c, what would be
total volume of water collected in the tank in
the value of a + b + c if log a + log b + log
liters?
c == 0?
27 4 I ® Reasoning & Aptitude fflRDE ERSl!:B
I
I
I
I
I
I
----:----~---t---~----:----
I
l
I
l
I
I
I
1
4 2
(G I I l I I I I
~
•-
zoo ----:----~---+---
I l l
l
l 1
---~---{---
1 I
SQ : : : : : : : 1 i 1
:::i 150 ----1----1----+- --;----1----1----+---
E : : : : : : :
258. In pqr t:- O and p-' = - , q-·Y = - , r 2 = -p, what
:J I I I I I I I q r
() 100 ----:----~--- l ---~----:----~---1---
I I I I I l I
I I I I I I is the value of the product xyz?
50 ---~- -~---t---~----~---~---f---
1 l l I I t
I I I l l 1
I l I l I I
1
0 3 6 9 12 15 18 21 24 (a) -1 (b) pqr
Hours
(a) 25,000 (b) 18,750 (c) 1 (d) pqr
x*y * z, what is the value of the product POR? citizen. Altogether, there were 300 senior citizens
eligible for this gift. However, only 8/9 th of the
(a)0 (b)1
eligible men and 2/3 rd of the eligible women
(c) xyz (d) rnxyz
claimed the gift. How much money (in Rupees)
[CE, 2018, 2 Marks (Set-2)]
did the philanthropist give away in total?
254. A faulty wall clock is known to gain 15 minutes (a) i ,50,000 (b) 2,00,000
every 24 hours. It is synchronized to the correct (c) 1,75,000 (d) 1,51,000
time at 9 AM on 11 th July. What will be the correct [CS, 2018, 2 Marks]
time to the nearest minute when the clock shows
260. In the figure below, LDEC + LBFC is equal to
2 PM on 15th July of the same year?
(a) 12:45PM (b) 12:58PM
(c) 1:00 PM (d) 2:00 PM E
[CE, 2018, 2 Marks (Set-2)]
256. What would be the smallest natural number which (a) LBCD - L'.BAD (b) LBAD + LBCF
when divided either by 20 or by 42 or by 76 (c) LBAD + L'.BCD (d) LCBA + LADC
leaves a remainder of 7 in each case? [CS, 2018, 2 Marks]
(a) 3047 (b) 6047
261. A six sided unbiased die with four green faces
(c) 7987 (d) 63847 and two red faces is rolled seven times. Which
[CS, 2018, 1 Mark] of the following combinations is the most likely
257. The area of a square is d. What is the area of outcome of the experiment?
the circle which has the diagonal of the square (a) Three green faces and four red faces.
as its diameter? (b) Four green faces and three red faces.
® Previous Years GATE Solved Questions I 275
(c) Five green faces and two red faces. which offers 10% annual rate of interest, if the
(d) Six green faces and one red face. interest was compounded annually?
[CS, 2018, 2 Marks] (a) 5,00,000 (b) 6,21,000
(c) 6,66,667 (d) 7,50,000
~62. In a party, 60% of the invited gu0sts are male
[EC, 2018, 2 Marks]
and 40% are female. If 80% of the invited guests
attended the party a~d if all the invited female 268. A cab was involved in a hit and run accident at
guests attended, what would be the ratio of males night You are given the following data about the
to females among the attendees in the party? cabs in the city and the accident.
(a) 2 : 3 (b) 1 : 1 (i) 85% of cabs in the city are green and the
(c) 3 : 2 (d) 2 : 1 remaining cabs are blue.
[CS, 2018, 2 Marks] (ii) A witness identified the cab involved in the
accident as blue.
. 1 1 1 1 (iii) It is known that a witness can correctly identify
~63. What Is the value of 1+ - + - + - +- + .... ?
4 16 64 256 the cab colour only 80% of the time.
Which of the following options is closest to the
7
(a) 2 (b) 4 probability that the accident was caused by a
blue cab?
3 4 (a) 12% (b) 15%
(c) -
2
(d) 3
(c) 41% (d) 80%
[EC, 2018, 1 Mark] [EC, 2018, 2 Marks]
~64. A 1.5 m in tall person is standing at a distance 269. A coastal region with unparalleled beauty is home
of 3 m from a lamp post. The light from the lamp to many species of animals. It is dotted with coral
at the top of the post casts her shadow. The reefs and unspoilt white sandy beaches. It has
length of the shadow is twice her height. What remained inaccessible to tourists due to poor
_ _ _-.:i-=-s---=t_:_:_he::.___:_h.:. .: e. .:. ,ig2-..~.:. .: 1t---=o~f:_-=-th:_:_e::. __:__:la: .:. .m:. :.ip=----i.: p_:: _o-=---st'-----'--'--in'-'---'-m=e~te~rs~?.~_______ --connectivLty.--@ncl-~-aGk--G-f---aGc0rRm0€1-a-tien;--/J,.-----
(a) 1.5 (b) 3 company has spotted the opportunity and is
(c) 4.5 (d) 6 planning to develop a luxury resort with helicopter
[EC, 2018, 1 Mark] service to the nearest major city airport.
Environmentalists are upset that this would lead
~65. If the number 715? 423 is divisible 3 (? denotes
to the region becoming crowded nnd polluted like
the missing digit in the thousandths place), then
any other major beach resorts.
the smallest whole number in the place of? is
Which one of the following statements can be
logically inferred from the information given in the
(a) O (b) 2
above paragraph?
(c) 5 (d) 6
(a) The culture and tradition of the local people
[EC, 20i 8, 1 Mark]
will be influenced by the tourists.
266. Two alloys A and B contain gold and copper in (b) The region will become crowded and polluted
the ratios of 2 : 3 and 3 : 7 by mass, respectively. due to tourism.
Equal masses of alloys A and B are melted to (c) The coral reefs are on the decline and could
make an alloy C. The ratio of gold to copper in soon vanish.
alloy C is _ _ (d) Helicopter connectivity would lead to an
(a) 5 : 10 (b) 7 : 13 increase in tourists coming to the region.
(c) 6 : 11 (d) 9 : 13 [EC, 2018, 2 Marks]
[EC, 2018, 2 Marks]
270. The Cricket Board has long recognized John's
267. Leila aspires to buy a car worth Rs., 0,00,000 potential as a leader of the team. However, his
after 5 years. What is the minimum amount in on-field Temper has always been a matter of
Rupees that she should deposit now in a bank concern for them since his junior days. While this
276 @ Reasoning & Aptitude fflROE ERS!::i
aggression has filled stadia with die-hard fans, 274. In a certain code AMCF is written as EOGJ and
it has taken a toll on his own batting. Until recently, NKUF is written as ROYJ. How will DHLP be
it appeared that he found it difficult to convert written in the code?
his aggression into big scores. Over the past (a) RSTN (b) TLPH
three seasons though, that picture of John has (c) HLPT (d) XSVR
been replaced by a cerebral, calculative and [EE, 2018, 2 Marks]
successful batsman-captain. After many years, 275. An e-mail password must contain three characters.
it appears that the team has finally found a The password has to contain one numeral from
complete captain. o to 9, one upper case and one lower case
Which of the following statements can be logically character from the English alphabet. How many
inferred from the above paragraph? distinct passwords are possible?
(i) Even as a junior cricketer, John was (a) 6,760 (b) i3,520
considered a good captain. (c) 40,560 (d) 1,05,456
(ii) Finding a complete captain is a challenge. [EE, 2018, 2 Marks]
(iii) Fans and the Cricket Board have differing
276. A designer uses marbles of four different colours
views on what they want in a captain.
for his designs. The cost of each marble is the
(iv) Over the past three seasons John has
same, irrespective of the colour. The table below
accumulated big scores.
shows the percentage of marbles of each colour
(a) (i). (ii) and (iii) only
used in the current design. The cost of each
(b) (iii) and (iv) only
marble increased by 25%. Therefore, the designer
(c) (ii) and (iv) only
decided to reduce equal numbers of marbles of
(d) (i), (ii), (iii) and (v) each colour to keep the total cost unchanged.
[EC, 2018, 2 Marks] What is the percentage of blue marbles in the
new design?
. (k+2) 2
271. For what values of k given below Is k- an
Yellow
3 Blue Black Red
278. A class of twelve children has two more boys 283. In a detailed study of annual crow births in India,
than girls. A group of three children are randomly it was found that there was relatively no growth
picked from this class to accompany the teacher during the period 2002 to 2004 and a sudden
on a field trip. What is the probability that the spike from 2004 to 2005. In another unrelated
group accompanying the teacher contains more study, it was found that the revenue from cracker
girls than boys? sales in India which remained fairly flat from 2002
to 2004, saw a sudden spike in 2005 before
325
(a) O (b) 864 declining again in 2006. The solid line in the
graph below refers to annual sale of crackers and
525 5 the dashed line refers to the annual crow births
(c) 864 (d) 12 in India. Choose the most appropriate inference
from the above data.
[EE, 2018, 2 Marks]
(\J
~ ~ . ~ , - ,- ,- ,- ,- ,~ )>
279. For O:S:x::::: 2n, sinx and cosx are both decreasing - .I. - -!- - "- - -1 - - I- - -I - - 1- - s
_!;; C
functions in the interval _ __ ~ - ~
<l)
:,,::
(.)
~
(.)
280. Arrange the following three-dimensional objects birth and cracker sales.
(b) Cracker usage increases crow birth rate.
in the descending order of their volumes:
(i) A cuboid with dimensions iO cm, 8 cm and (c) If cracker sale declines, crow birth will decline.
6 cm (d)-·trrcreasect·btrth-ratcot~TOws-wrll'~-ause-an
(ii) A cube of side 8 cm increase in the sale of crackers.
(iii) A cylinder with base radius 7 cm and height [IN, 2018, 2 Marks]
7 cm
(iv) A sphere of radius 7 .cm 284. If x 2 + x - 1 = 0 what is the value of x +~?
4
(a) (i), (ii), (iii), (iv) (b) (ii), (i), (iv), (iii) X
(c) (iii), (ii), (i), (iv) (d) (iv), (iii), (ii), (i) (a) 1 (b) 5
[IN, 2018, 1 Mark] (c) 7 (d) 9
[IN, 2018, 2 Marks]
281. The area of an equilateral triangle is ✓3. What
is the perimeter of the triangle? 285. To pass a test, a candidate needs or answer at
(a) 2 (b) 4 least 2 out of 3 questions correctly. A total of
(c) 6 (d) 8 6,30,000 candidates appeared for the test.
[IN, 2018, 1 Mark] Question A was correctly answered by 3,30,000
candidates. Question B was answered correctly
282. An automobile travels from city A to city B and
by 2,50,000 candidates. Question Cwas answered
returns to city A by the same route. The speed
correctly by 2,60,000 candidates. Both questions
of the vehicle during the onward and return
A and B were answered correctly by 1,00,000
journeys were constant at 60 km/h and 90 km/h,
candidates. Both questions B and C were
respectively. What is the average speed in km/h
answered correctly by 90,000 candidates. Both
for the entire journey?
questions A and C were answered correctly by
(a) 72 (b) 73
80,000 candidates. If the number of students
(c) 74 (d) 75
[IN, 2018, 2 Marks] answering all questions correctly is the same as
278 ® Reasoning & Aptitude fflRDE EASY
the number answering none, how many candidates Which of the following statements can be logically
failed to clear the test? inferred from the above statements?
(a) 30,000 (b) 2,70,000 (a) If (i) is true and (ii) is false, then (iii) is false.
(c) 3,90,000 (d) 4,20,000
[IN, 2018, 2 Marks]
(b) If (i) is true and (ii) is false, then (iii) is true.
(c) If (i) and (ii) are true, then (iii) is true.
iI
286. A set of 4 parallel lines intersect with another set
(d) If (i) and (ii) are false, then (iii) is false. I
[ME, 2018, 2 Marks (Set-1 )]
of 5 parallel lines. How many parallelograms are
formed?
(a) 20
(c) 60
(b) 48
(d) 72
292. Which of the following functions describe the
graph shown in the below figure.
II
[IN, 2018, 2 Marks] : : : y : : :
--t---:---i--3 ---i---t---:--- t!
- - : - - -:- - - : - - - -- : - - :- - -: - -
1 I I I I I
1
l
--i---:--- !-=1
!
I
I
---!
!
I
1 l
--;---i--
! f
___ ! _ _ _ _ _ _ ! _ _ _ _ _ I __ 1 ___ 1 __
I!
I
original rectangle in square meters? : :-2 : : :l
I I I I
--j--7---r-- ---,--~---1--
(a) 1125 (b) 2250 : : :-3 : : :
(c) 2924 (d) 4500
[ME, 2018, 1 Mark (Set-1)]
I
I
di
1. (d) 52. (d) 103. (d) 154. (b) 205. (d) 256. (c)
2. (c) 53. (a) 104. (4536) 155. (a) 206. (b) 257. (d)
3. (d) 54. ( 16) 105. (d) 156. (d) 207. (c) 258. (c)
4. (b) 55. (d) 106. (a) 157. (b) 208. (b) 259. (b)
5. (b) 56. (b) 107. (c) 158. (d) 209. (c) 260. (a)
6. (b) 57. (d) 108. (c) 159. (b) 210. (c) 261. (c)
7. (d) 58. (4) 109. (b) 160. (b) 211. (d) 262. (b)
8. (a) 59. (20000) 110. (8) 161. (a) 212. (c) 263. (d)
9. (d) 60. (0.81) 111. (a) 162. (d) 213. (c) 264. (b)
10. (c) 61. (a) 112. (b) 163. (a) 214. (b) 265. (b)
11. (a) 62. (495) 113. (a) 164. (c) 215. (d) 266. (b)
12. (b) 63. (c) 114. (a) 165. (c) 216. (a) 267. (b)
13. (c) 64. (b) 115. (32) 166. (a) 217. (d) 268. (c)
14. (d) 65. (b) 116. (c) 167. (a) 218. (a) 269. (b)
15. (c) 66. (22) 117. (c) 168. (d) 219. (b) 270. (c)
16. (a) 67. (b) 118. (a) 169. (b) 220. (c) 271. (c)
17. (b) 68. (96) 119. (c) 170. (d) 221. (c) 272. (d)
18. (b) 69. (d) 120. (c) 171. (c) 222. (a) 273. (a)
19. (c) 70. (850) 121. (3) 172. (b) 223. (b) 274. (c)
20. (a) 71. (48) 122. (d) 173. (c) 224. (d) 275. (c)
21. (d) 72. (6) 123. (b) 174. (a) 225. (a) 276. (c)
22. (a) 73. (b) 124. (d) 175. (7) 226. (b) 277. (c)
23. (c) 74. (c) 125. (b) 176. (120) 227. (d) 278. (b)
24. (d) 75. (d) 126. (800) 177. (c) 228. (c) 279. (b)
25. (a) 76. (140) 127. (a) 178. (b) 229. (c) 280. (d)
26. (d) 77. (a) 128. (c) 179. (c) 230. (a) 281. (c)
27. (a) 78. (c) 129. (b) 180. (c) 231. (d) 282. (a)
28. (d) 79. (c) 130. (c) 181. (a) 232. (b) 283. (a)
29. (b) 80. (d) 131. (2.064) 182. (c) 233. (c) 284. (c)
30. (a) 81. (c) 132. (b) 183. (d) 234. (d) 285. (d)
31. (c) 82. (c) 133. (b) 184. (d) 235. (c) 286. (c)
32. (d) 83. (c) 134. (280) 185. (c) 236. (b) 287. (b)
33. (c) 84. (1300) 135. (c) 186. (a) 237. (a) 288. (b)
34. (b) 85. (d) 136. (b) 187. (c) 238. (d) 289. (b)
35. (b) 86. (b) 137. (c) 188. (b) 239. (b) 290. (d)
36. (c) 87. (180) 138. (a) 189. (b) 240. (22.22) 291. (c)
37. (c) 88. (d) 139. (c) 190. (b) 241. (d) 292. (b)
38. (d) 89. (b) 140. (c) 191. (b) 242. (c) 293. (a)
39. (b) 90. (25) 141. (d) 192. (d) 243. (c) 294. (d)
40. (a) 91. (a) 142. (c) 193. (d) 244. (b) 295. (b)
41. (a) 92. (a) 143. (c) 194. (c) 245. (b) 296. (a)
42. (16) 93. (d) 144. (c) 195. (c) 246. (b) 297. (c)
43. (d) 94. (c) 145. (b) 196. (d) 247. (d) 298. (b)
44. (b) 95. (0.4896) 146. (d) 197. (b) 248. (a) 299. (*)
45. (560) 96. (b) 147. (d) 198. (b) 249. (b) 300. (c)
46. (d) 97. (c) 148. (c) 199. (a) 250. (c) 301. (c)
47. (b) 98. (4.54) 149. (b) 200. (c) 251. (b) 302. (d)
48. (b) 99. (b) 150. (c) 201. (a) 252. (c)
49. (45) 100. (b) 151. (a) 202. (a) 253. (b)
50. (c) 101. (b) 152. (d) 203. (b) 254. (b)
51. (163) 102. (a) 153. (c) 204. (a) 255. (b)
fflRDE ERS!::l! 111 Previous Years GATE Solved Questions I 281
1. (d)
(731)8
Using the set theory formula
n(A) : Number of people who play hockey= 15 +(672)8
n(B): Number of people who play football= 17 (1623)8
n (An B): Persons who play both hockey and
Hence the overall problem was based on
football = 10
identyfying base, which was 8, and adding
n (A u 8): Persons who play either hockey or
football or both number on base 8.
Using the formula 'J
w. (d)
n (Au B) == n(A) + n(B) - n(A n B) 1
Per day work or rate of 5 skilled workers=
n(AuB)= 15+ 17-10= 22 20
⇒ Per day work or rate of one skill worker
Thus people who play neither hockey nor football
1
= 25 22 = 3 =--=-
5x20 100
Alternative Method Similarly Per day work or rate of 8 semiskilled
i
Refer to Venn diagram given below: workers= -
25
⇒ Per day work or rate of one semi-skill worker
Football i 1
(17) =--=-
8 X 25 200
And per day work or rate of 10 unskilled workers
=
30
Neither hockey ⇒ Per day work or rate of one semi-skill worker
nor football
Both 1 1
(3) =---=
10x30 300
Thus total per day work of 2 skilled,
Number of people playing nither of the two 6 semiskilled and 5 unskilled workers
games is equal to 3.
= -2+ -6- +5- =
i2+18+10
2. (c) ----
! 00 200 300 600
137 + 276 =435
This an addition on base 8. 40 1
=-=-
600 i5
Hence, 73i + 672(8) = i623
Alternative Method Thus time to complete the work is i 5 days.
Alternative Method
7 and 6 added is becoming five means the
Let one day work of skilled semi-skilled and
given two numbers are added on base 8.
unskilled worker be a, b, c units respectively.
(137)8 Sa x 20 = 8b + 25 = 10c x 30 = Total unit of
+(276)8 work
1ooa = 200b = 300c
(435)8
a= 2b = 3c
Hence we have to add the another two given
a a
set of numbers also on base 8. ⇒ b = - and c =~
2 3
282 I ® Reasoning & Aptitude fflRDE ERSY
( 2a + 3a + ~a) x = Sa x
20 (~: = 3numbers are possible)
(b) Using 2, 2, 4 ⇒ 224, 242, 422
20a
- - x = Sax 20
3
( ~: = 3 numbers are possible)
x = 15 days
(c) Using 2, 3, 3 ⇒ 233, 323, 332
4. (b)
We have to make 4 digit numbers, so the ( ~: = 3 numbers are possible)
number should be start with 3 or 4, two cases (d) Using 2, 3, 4 ⇒ 234, 243,324,342,
possible; 423,432
Case (1) thousands digit is 3
( ~: = 3 numbers are possible)
Now other three digits may be any of 2, 2, 3, 3,
4, 4, 4, 4. (e) Using 2, 4, 4 ⇒ 244, 424, 442
(a) Using 2, 2, 3
( ~: = 3 numbers are possible)
⇒ 223, 232, 322 -------
Alternate Solution 1 80 85
Sx--+---
or TC.== 4q+
100 5x+y
n -- - - -- 3 3 _-2
q 5 5 5
14. (d)
2 20. (a)
~ ~] = - 1O which is negative AD, CG, FK, JP are given terms. The first letter in
q q=5
each term A, C, F, J are respectively 1, 3, 6, 10
Hence maximum profit wilt happen at letter of alphabetical sequence.
q=5 A = 1; i st letter of alphabetical sequence
C = 1 + 2 = 3 ; 3rd letter of alphabetical
i 7. (b)
sequence
y = 2x-0.ix2
F = 1 + 2 + 3 = 6 ; 6th letter of alphabetical
For y (height) to be maximum dy = 0 sequence
dx G = 1 + 2 + 3 + 4 = 10; 10th letter of alphabetical
dy .
- = 2x-2x= 0 sequence
dx The next term should begin with
⇒ X= 10 1 + 2 + 3 + 4 + 5 = 15th letter of alphabetical
d2y
dx2
l x= 10
=-0.2 (-1ve),
. sequence. So next term should begin with 0.
Also in the series AD, CG, FK, JP, two letters in
each term are separated by 2, 3, 4, 5 letters in
Hence maximum height will be at x = 1O between them respectively. Hence next term
y = 2 X 10-0.1 X (10) 2 = 10 should be OV as there should be a gap of 6
Maximum height= 10 meters. alphabets between O and V.
Hence answer is OV.
18. (b)
Let total 100 shock absorbes are supplied 21. (d)
so X supplies = 60 ( 40% of 100) (1.001 ) 1259 = 3.52 and (1.001 )2062 = 7.85
Y supplies = 40 (60% of 100) 3321
(1.001) 1259 X (1.001) = (1.001)
Reliable supply by X = 96% of 60 = 57.6 j, j,
Reliable supply by Y = 72% of 40 = 28.8
3.52 x 7.85 = (1.001) 3321 (as am x an= am+n)
So the required probability that random chosen
reliable shock absorber is made by Y is Hence (1.001 )3321 = 27.632"' 27.64
Required probability
22. (a)
28.8 0 33 3 Let the number of~ 20 notes be 'a' and~ 1O notes
== 57.6+ 28.8 = .
be 'b' respectively.
~ 0.334
So, a+ b = 14
19. (c) 20 a + 1 0 b = 230
Adding each and every observation by constant on solving a = 9, b = 5
increases the arithmetic mean by same constant. Hence number of <'10 notes are 5.
Also multiplying each and every observation by Alternative Solution:
constant mutiplies the arithmetic mean by same If we use option (a)
constant. While the standard deviation will not Number of ten <' notes are 5.
show similar effect <10 X 5 = <50
Alternative Solution: The remaining nine notes have to be Z 20 notes
<20 X 9
:::: 180
and hence 50 + 180 = 230 (total money value)
Hence option (a) number of ten~ notes are 5.
23. (c)
Probability that A & B will meet will be given by ·
the graphical representation where shaded region
Hence only P, Rare correct.
represents favorable area
@ Previous Years GATE Solved Questions I 287
unfavorabie
favorable
28 - 81 ➔ 4 weghing required
favorable
area
and the process will go so on.
60 .--1-..--~~,,,. 4~~~,-.,1-,
B 45
t 30~'/.m~
3----.. As there are eight objects. Minimum number of
weighings required will be only 2.
i 5 ~tfri"t/rl>,,/r/,"'i<--"'t---,
111 111 28. (d)
0 LA.4.A.O"-----'---I--J % of raw material are salary and wages
15 30 45 60
-11> A unfavorable = 6240- 5200 X I 00 == 20 %
area
5200
Required probability
% of raw material and research and development
fabourable area 7
-
total area 16 = 26400 - 22000 X I OO = 20 %
22000
24. (d)
29. (b)
Category Amount
Food 4000
I 4x - 11 = 5
Clothing 1200
4x - 7 =5
and 4x 7 = .5
Rent 2000
⇒ X = 3
Savings 1500
Others 1800
and x = 0.5
Total 10500 21xl - I-XI= 2 x 3 - 3 =3
i i i
So total expenses are i0,500. and 2fxl - I-xi = 2x- - -
2 2
=-2
Expenses excluding savings are 9,000.
= 1 3
Hence required percentage
~------=-+~:
2'
which are divisible by 7 are (14, 21, ... 98) only From eq. (ii)- eq. (i)
13 numbers. Th- M = 6
Also Th= i.iSM
1.i5M-M=6
⇒ 15M = 6
So, the required probability that the number is not ⇒ M = 40
divisible by Th = i .15 m
⇒ Th = 1.15x40
1-~ = 77
90 90 ⇒ Th == 46
i Js-1-Joo]
........ [ ✓Sx.foox Js-1-Joo s a + ar + ar 2......... ar n-1)
Which can be resolved as [A a(rn -i)
=--- for r>1
[ ✓2 - ✓1] + [ ✓3 - ✓2] ..... '''' '[✓81 - ✓80] (r - i)
(as denominator in each term will become 1)
Now cancelling like terms we will be left with
and i + 3 + 5 ............ (2n- i) = n
( Sum of odd natural numbers =n
2
2
l
)Si_ ✓1 = (9 - i) =8
9(9n - 1)
35. (b)
So S = - - - + n2
8
4b + 12 == 348
b= 84
Sum of smallest odd no ( a) and second largest
55. (d)
P=A<R<M
P is surely less than M only in option (d) else in
I
l
even no (b + 4) will be any other option it cannot be conveyed surely. ll
= (a)+ (b + 4) l
56. (b)
52. (d)
== 75 + (84 + 4) = 163
7 G, 1 i K, 13 M, _ _
Next term will be Q which is 17 letter of English
I
i
'
For maximum revenue per kg. Let us solve through alphabet.
i
iI
options Among option only (b) (i.e. 17 Q) is correct.
A
20% of 250
ltem 2"' 20% of 500 = 0.5
57. (d) l
I
On seeing the given figure in question
23% of 250 i, ii and iii can be concluded.
8. Item 3 = % of = 0.6
19 500
58. Solution :
19% of 250 Let the velocity of stream is 'V' time taken during
C. Item 6 = i6% of 500 = 0.59
downstream is 't' and distance is 'x'.
20% of 250 X
D. Item 5 = % of = 0.83 t = - ... (i)
12 500 8+V
Clearly item 4 is largest. During upstream
X
53.(a) 3t = - ... (ii)
8-V
Drainage pipe drains half of the tank in 30 minutes
Dividing equation (ii) by (i), we get
or full tank is 68 minutes. Now the tank have to be
V = 4 m/s
filled in 10 minutes (when both drainage and filling
pipes are in action) 59. Solution: l
i
I
Drainage pipe would drained 1/6th of capacity of Since,
tank in 10 minutes. So filling pipe have to fill 1/2 i 5% = 4,50,000
of capacity of 1/6 of capacity more i.e. 2/3rd of OO% :::: 4, 50,000 X 100
the tank in 1O minutes. or complete tariR is 15 1
15
minutes. That's why the filling tank should have = 30,00,000
rate 4 times the draining rate. Selling price of purifier
Alternatively :
30,00,000 + 10,00,000
Drainage pipe can drain complete tank in 60 min. =
200
Let the filling pipe can fill the tank in 'x' min. Now
both are operational and tank is half filled. Also = 20,000
remaining half needs to get filled in 10 minute.
60. Solution:
rn[J_ __i Jl = _:1_
5
X 60 2 Probability of bulb being defective =
100
x == 15 minutes
So filling pipe needs to have rate be times = 0.05
drainage pipe's rate. Probability of bulb being non defective
= 1 -0.05
54. Solution: = 0,95
81, 54,36,24,_ Probability that the batch is accepted
Alternate terms are perfect squares 92, 6 2 , 4 2 and = None of the four bulb
in between terms are products 9 x 6, 6 x 4m _ _ being defective
and so on missing terms is = (0.95) 4 = 0.81450625
42 = 16 "'0.81
fflRDE ERS~ ® Previous Years GATE Solved Questions I 291
4,50,000x100
69. (d)
= Given ax 2 + bx + c = 0
15
roots are real and positive
= 30,00,000
292 ® Reasoning & Aptitude fflRDE ERS~
= 10 ['1 + 2 + 3 + .. · + 19]
If x < 0 }
Ix I= - x x2 + 5x + 6 = O 19
= 10 ['19 x 20] = mo
(x + 2) (x + 3) = O x = -2, -3 are roots 19 X 2
So if x 2 - 5x + 6 = O has two real roots
I I
then x 2 - 5 I x I + 6 = 0 has four real roots 7 4. (c)
ax 2 + bx + c = O has real roots
then ax 2 + b Ix I + c = O has 4 real roots.
Let. Y= )12+ ✓12+ ✓12+ .. ,
70. (850 to 850) So, y = ✓12+ y
One way fare of single person = 100
Squaring y2 = i 2 + y
Undiscounted round trip fare of singe person
y2 - y - 12 = 0
= 200
⇒ (y - 4) (y + 3) = 0
Undiscounted round trip fare of five persons
y=4
= 1000
Option (c) ➔ 4.000
Round trip will result in a discount of 10%. Also
additional discount of 5% on total fare for 4 or 75. (d)
more person. Given x is real
So a term of 5 person will be eligible for overall
discount of (10% + 5%) = 15% discount Ix2 - 2x + 3 I = 11
Discounted round trip for 5 persons
x2 2x - 8 = 11
1000 x 0.85 = 850 Rs.
x2 - 2x - 8 = O
71. (48 to 48) (x - 4) (x + 2) = 0
Following table can be redrawn: = 4, -2
X
or x2 - 2x + 3 = - 11
x2 2x + 14 = O
Will not give real roots as D = b2 - 4ac < O
I- x3 + x x I= I8 + 4 + 2 I
2
- = 14
76. (140)
Given ratio of male to female is 2.5 in year 2008.
So % of people not owing a scooter So let us assume male as 500 and female as 200.
[(
74
+
70
) X 100] ::::
144
X 100 = 48% So ratio ~ in 2008 = 2.5
300 300
• Previous Years GATE Solved Questions I 293
78. (c)
2, 5, 10, i 7, 26, 37, 50, 64.
82. (c)
2 = 12 + 1 , 5 = 22 + 1 , 10 = 3 2 + 1, (7526) - (Y)8 = (4364)8
17 = 42 + 1, 26 = 52 + 1, 3 7 = 62 + 1, So, Y = (7526) 8-(4364)8
50 = 72 + 1, 64 = B2 = (7526-4364)8
So 64 is the incorrect term. = (31420)8
79. ( C) 83. (c)
Answered Answered Not Total
O.No. Marks dy
Correctly Wrongly Attemped Students Slope = · · = 10x
1 2 21 17 6 44 dx
2 3 15 27 2 44
3 2 23 18 3 44 dy!
dx (o,3) =0
Total marks scored
Average = Equation of tangent
Total number of students
(y-3) = 0(x-0)
= 21x2+15x3+23x2 = 3 _022727
44 y=3
'.::::: 3.02 Line parallel to x-axis.
294 ® Reasoning & Aptitude fflADE EASY
90. Solution:
91. (a)
⇒ 10 = 5( 1+ 100
20 r
⇒ 2 = (i.2t = ✓3
log2 95. Solution:
⇒
log1 .2 == n
+/ 95% correct .
⇒ n = 3.8 year 10% HIV
93. (d) ~ 5% incorrect
97. (c)
Let f(x:) = Ax:+ b (linear function) (2, 0)
at x = -2,f(x) = 29
⇒ + b = 29
--2a ... (i)
X = 3, f(x) = 39
⇒ 3a + b = 39 ... (ii)(a = 2, b = 33) (0,-1) - - - - " " ' \ . .
Giving f(x) = 2x + 33
⇒ f(S) = 43 Option (c) is not possible.
x = 0 for +ve value of y in
graph.
98. Solution Option (a), through valid for +ve values of y, is
Combined percentage increase in imports and not correct for -ve values of 'y'.
exports for year 2001 Option ( d) is also not true since x is -ve for +ve
values of y which is contradictory to the given
- (60+ 50)-(50+ 40) 100 - 0
- + X - 22.221/o graph.
50 40
Combined percentage increase in imports and 100. {b)
Option (a) is not correct as R is given defence.
exports for year 2002
As given in question R wants only Finance or ______
- (70+60)-(60+50) 100 - 0
Home.
- 60+S0 X - 18.181/o Option (d) is not corect as R is alloted Telecom
(same reason as above).
Combined percentage increase in imports and
Option (c) is not possible because S & U cannot
exports for year 2003
be together as per question.
So only option (b) is possible.
= (70+60)-(60+70)x 100 =0
70+60 101. (b)
Before drawing third card, only two kings remain
Combined percentage increase in imports and
among the remaining 50 cards.
exports for year 2004
- (70+90)-(70-80) 100 - 0
- 70+ 80 X - 6.671/o
&VNE is equilateral & Hence both statements will be required for finding
So VE= 6 out gallons of solution in A and B respectively.
Also VP = VE - PE = 6 - 4 = 2
i06. (a)
VP= 2, PW= 2
= 2✓2
a 0
6
103. (d)
Type I:
=C 1
~~
75
)x100 =53.3% 5
C
Type II:
85 -144) n=6
=( x100 = -40.97% EM
144
Type Ill: 40
= 11,200
Vol. in 8 occupied = 40% of 20000 = 8000
fflRDE ERS!::11 ® Previous Years GATE Solved Questions I 297
115. (32)
As ON= 1, OA = AN= -J2.1 = PB Let the total number of students in electrical be
Te. Male students in electrical is 40.
So.MB= ( 4+2-~) 5
So, -T0 == 40 ⇒ Te= 72
9
andPB= ( ~ ) Also, let total number of students (overall)= T
Electrical is 20% of T
⇒ 20% of T = 72 ⇒
T == 360
PM= ✓PB2 +BM2 = 5.34
Now number of female students in Civil
,,M 4
/
/
/
/
/
⇒ 30% Of 360 X g = 48
/
/
/
/
/
/
/
/
/
-----------------
Difference = 48 - 16 = 32
p 1 8
✓2 116. (c)
r q
According to question
In llPQR ⇒ Ar/\PQR = Ar(llOPS) + l\r(IiPSR) 75% = .a + b + c + (x + y + z) + t
Also, x + y + z + t = 50%
~rxqxsin120° = lrxPSxsin60°+~qxPSsin60°
X + y+ Z = 40%
so, t = 10%
2r x qx sin(i 80- 60°) = 2 PS sin60°(r + q) so, (iii) is correct.
2 2 .
and a + b + c = 25 %
2r
2
x q x sin60° = 2 PS sin60°(r + q)
2
Also, m + c +ch= (a+ x + y+ t) + (b + x + z
+ t) + ( C + y + Z + t)
m + c + ch= (a + b + c) +2 (x + y + z) + 3t 25
PS= [rxq]
r+q
27
+2(40)+3x 10= i35% =
113. (a) 30
From statement A, if height of each step is known,
117. (c)
we can get the number of steps as height of
Marks are alloted only for correct answer
building is given in the question.
Total marks
114. (a) Average marks= N
o. o f st udens
t
A= {2, 3, 4, 5}, B = (11, 12, 13, 14, 15]
Sample space will contain 4 x 5 = 20 events 21 X 2 + i 5 X 3 + 11 X 1+ 23 X 2 + 31 X 5
=
favourable events [2, 14](3, 13][4, 12)[5, 11) for 21 + 17 + 6
sum 16
99
4 == :? == 6.795
required probability=
20
= 25 = 0.2 44
298 ® Reasoning & Aptitude fflRDE ERS~
⇒
X = (if N
0
45
75
25
45
13/25 = 0.52
20/30:::: 0.66
( 66 - 6)
66 + 6
X( 66 + 6) :::: 1
66-6
: : i i)
X ( +; X ( ¾) = :! : : ±
124. (d)
120. (c)
Both the conclusions I and II follow.
Number of faces visible == 9 x 6 = 54
(9 on each face x 6 faces) 125. (b)
We know,
(a+ b + c) 2 = a2 + b2 + cl-+ 2[ab + be+ ca]
(a+ b + c) 2 -
1 == 2[ab + be+ ca]
[ as a2 + b2 + c2- = 1]
For ab+ be+ ca to be min. (a+ b + c) = 0
. 1
Min. (ab+ be+ ca)= -
Total faces == 27 x 6 2
Not visible = Total face - visible faces [an putting a+ b + c = 0]
:::: 27 X 6- 9 X 6 = I 08
Only option (b) has min. value as -~.
Visible 54 2
Required ratio = - - - - = - =-i
Not visible i 08 2
126. (800)
121. (3) Tiger is 50 leap of its own behind deer. Given tiger's
one leap is 8 m. So initial separation of Tiger and
sum)
Look at first row middle term is ( - - of terms deer == 50 x 8 = 400 m/min
2
Tiger goes 5 leap in a minute which is equal to
6 4
an e1t. her s1'd e .1t + = 5 . 5 x8 = 40 m/min.
X
Deer goes 4 leap in a minute which is equal to
Second row middle term is ( s~) of terms an 4 x5 = 20 m/min.
Now relative distance of 400 m have to be covered
7+4+2+1 = 20 m/min.
either side of it
2
=7 is equal to middle with (40-20)
i 30. (c)
log tani 0 + log tan2° + ...... + log tan89°
= !og(tan1° x tan2° x • • • tan89°)
As [logm + logn = log mn]
= log(tani O x tan2° • • • tan (90° -2°)) tan(90° - 1°) AreaAOTS= 2so+OTsin90°
2
0
= log(tan 1° x tan2° x • • •tan45° x cot44 ••. cot2° coti )
=log(tan45°) 1
20:: - XX X 2y X 1
=log(i)=0 2
As tan(90° - 0) = cote and xy= 20
tan1° x·coff' =tan2° x cot2° =·· .. 1
S1:0 0
Arc length
of shaded
S2:
2rcR' = ~ 2rc x 30 R' = 27
10
orN /0)
fil ~ 27 Fig. (a)
~
~
~
Fig. (b)
E/
300 ® Reasoning & Aptitude fflRDE EASY
35
4 X ( -9x10)
- X (10x11)
- - = 9900. 0.5 X 0.7= 0.35 z 10Q = 35%
2 2
fflRDE ER5Y w Previous Years GATE Solved Questions I 301
a log q = log r,
b log r = log s, iO 2
clog s = log q
logr logs logq
So ax bx c = -- x -- x -- = 1
logq logr logs
143. (c)
Q can do work in 25 x 12 = 300 hrs
R can do work in 50 x i2 = 600 hrs Mirror image of i : 30 is i O : 30
So we can say Q is twice efficient as R 10: 30 was the time two and quarter hour back
Now O worked only for 5 days@ 12 hrs/day. So so time now will be i 2 : 45
he will do 60 units of his work (Total work for Q
148. (c)
~--He-wi+h:te---( ~
66}=-(e:--crruf-hrs-work------
i 126
So required ratio
5 . 600
120: 126
N
20: 21
See the adjoining figure for solution
145. {b)
M works with twice efficiency as E but worked MM = 5-fj_ + 5 + 2✓2 == 5 + 7✓2
for half as many days. So in this repsect they
NM= 10+5✓2-2✓2 = 10+3✓2
will do equal work if their shifts would have been
for same timings. But Ms shift is for hrs, whil8 MN = ✓(MM')2 + (NM) 2
E's shift for i 2 hrs. Hence Ewill do twice the work
as M. = ✓(5+ 712)+(10+312.)2 ~ 20.61
Ratio of contribution of M : E in work, i : 2
149. (b)
146. (108)
Let the two parts length is x and y
so x + y = 340 mm ... (1)
The part which has length x is used in making
a square so each side will be length x/4, and the
part which has length y is used in making a
rectangle with sides in ratio i : i so sides have
length (y/6 : y/3)
Area of square is x 2/i 6
302 ® Reasoning & Aptitude fflRDE EASY
(0, 7)
Since the combined area should be minimum so
using equation i in 2, we get
!!_(x
dx 16
2
+ (340 -
18
x)2 l_ - O
(0, 1) B
-------'-'-'--+-~-x
0
so [ i i- ~88°] = O
+
or x = 160 mm 2x-3y = 5
and length of side of square is
X Required area = Area of ~ABC + Area of
- = 40 mm
4 trapezoid BCDO
150. (c) 1 1
= - X 4 X 6 + - X (4 + 2.5) XI
97 is not a perfect square. 2 2
= 12 + 3.25 = 15.25 sq. units
152. (d)
The odometer reading increases from starting 155. (a)
point to end point Straight line equation y = mx + c
Magnitude of area of the given diagram = Odometer m = slope=-0.02 (lnx,y)
If ln x = X, then set (x, y)
reading
Magnitude of area of the velocity and time graph
t t
0.1 0
per second y =
mX+ C
o = -0.02 x 0.1 + C
1 1
i st sec ⇒ triangle = x 1x 1 = C = 0.002
2 2
2nd sec ⇒ square= 1 x 1 = 1
y = mX+ C
3 rd sec ⇒
y = -0.02 x logx + C
at X =5
1 1
square+ triangle= 1+ 1+ - x 1x 1 = 1- y = -0.02 x log 5 + 0.002
2 2 = -0.030
1
4th sec ⇒ triangle =- x 1x 2 =1 156. (d)
2
Original surface area = 6 (4) 2 = 96
5th sec ⇒ straight line = 0
If corner cubes are removed, three exposed
1 1
6th sec ⇒ triangle = -- x 1x 1 = - surfaces are removed which will create 3 new
2 2 surfaces in miginal large cube. So surface area
· 1 1 will remain unchanged, i.e. 96.
7th sec ⇒ triangle= - x 1x 1 = -
2 2
Total Odometer reading at 7 seconds 157. (b)
Elegance
1 1 1 1
= -+1+1-+i+0+-+-=5 (27300 + 25222 + 28976 + 21012) x 48 = A
2 2 2 2
Executive
153. (c) (999 + 8942 + i0234 + 10234 + ·10109) x 173 = B
Manu's age = Sravan's age + 2 months Smooth
Manu's age = Trideep's age - 3 months (20009 + 9392 + 22429 + 18229) x 63 = C
Pavan 's age = Sr avan's age + 1 month Soft
From this Trideep's age> Manu > Pavan > Sravan (17602 +18445 + 19544 + 16595) x 78 = 0
:. Trideep can occupy the extra spane in the flat. Which is highest for B (executive).
® Previous Years GATE Solved Questions I 303
160. (a)
f(1)= 2(1) 7 + 3 - 5 = 5 - 5 =0 -~------'"'P~U~t V = ~L(~r -_;X~= 4
=[=---------
-=~--QLJIL-;2-=-=
put y = 1, i2 - ~ = _2
3 3
174. (a)
60
Probability of free throw= = 0.6
given, - 100
Probability of NOT free throw= 1 - 0.6 = 0.4
162. (d) So required probability of exactly 6 throws in ·1 O
attempts will be given by
C A D B E
~
10
-2, +3, -2, 3 C5(0.6)6 x (0.4)4 = 0.2508
Same pattern follows in all options A, 8, C only
(d) doesn't follow this pattern , Hence odd number 175. Sol.
out (D). Unit digit of 211 870 + 146 127 x 3424 is 1 + 6 x 1 == 7
a a
a
a a
T = 150 a
304 • Reasoning & Aptitude fflRDE ERS!::I
8
2 2 8 1214 16 20 22 24 26
4 6 8
186. (a)
P 30 min x 2 = 60 min
Q 20 min X 3 = 60 min
R 60min X 1 = 60 min
s 15 min X 4 = 60min
\\ M1 I p II Q I = 2 hrs
-2n 21t X M2 I R II s I = 2 hrs
fflRDE ERSW 111 Previous Years GATE Solved Questions I 305
190. (b)
Sample space = 6 x 6 = 36 events = Total Q Q
chances Given in question P & U interchange then new
Product of numbers on 2 dice have to perfect diagram can be drawn.
square = Favourable chances T T
Put (x = -"1)-···
s1[~:J 2 =~
32
34 x2 4
=32 x22 = 144
196. (d)
Let His householrl consumption and P be other
consumption.
1~l H x 0.8 + P x 1.7 = (H + P) x 0.75
LsJ
(According to given condition)
On putting (x == -1)
:. from here ratio is negative.
⇒ LHS = RHS
Hence option (b) is correct.
197. (b)
192. (d) B
193. (d)
Let cycling speed =C
and walking speed == W
c(2l+w(~l
\_2) \2)
c-
-·
19
198. (b) X
40
40% of 360° = -x360° = 144°
100
So, the angle subtended on pie chart will be
= 144°.
199. (a)
Length of Truck+ gap required = 1O + 20 = 30 m T
Length of Car + gap required = 5 + i 5 = 20 m Only one such arrangement can be drawn.
Alternative pairs of Truck and Car needs The person on third to the left of V is X.
30 + 20 = 50 m
203. (b)
Let 'n' be the number of repetition of (Truck +
Car) in i hour (3600 secs) 204. (a)
Given speed= 36 km/hr = 1O m/sec 205. (d)
50mxn
3600 secs = 36 km/hr 206. (b)
1~r
Down Up Down
Down-Up-Down satisfies.
probability is ( or 0.7K
201. (a)
Subgroups containing only Indians 2ii. (d)
3 3 3 y x 162 = Perfect cube
= C1 + C2 + C3 = 3 +3 + 1 = 7
3 3 3
is a perfect cube
3
Chinese= C2 [ C1 + C2 + C3 ]=21
Hence the answer is. y = 36
Subgroups containing three Indian and remaining
212. (c)
3 3 3 3
Chinese= C3 [ C1 + C 2 + C3 ]=7 Following seating arrangement can be drawn
Total number of subgroups
= 7 + 2i + 21 + 7 = 56. S~A
202. (a)
Following circular seating arrangement can be R~M
214. (b)
omarried ➔ Runmarried
2p ➔ i50K
Median will be arithmatic mean 50th and 5ist only one pair in which married person is looking
term, so both are in 30K bracket. @ unmarried.
So average or arithmatic mean of, 2. Let Q be unmarried
pmarried ➔ ounmarried
30000 + 30000 ounmarried ➔ Runmarried
= Rs. 30000
2 Then also only one pair exists.
So, only one pair exists in which married person
227. (d)
is looking @ unmarried.
The following Two possibiles exists.
237. (a)
5 0 1 0 0 5 0 0 0 0
- 4 8 9 8 9 and - 4 8 8 8 9 x ➔ Bullocks
0 1 1 1 1 0 1 1 1 1 y ➔ Tractors
Sum=1+9=10 Sum::: 0 + 8 = 8
228. (c)
(x + y)8 = (j+2y )s
E 5x
8x + 8y= +i0y
(5, 3)
2
⇒ 5.5x = 2y or y = 2.75x
Now, x bullocks are alone working or 'd' days.
A (0, 1) B xx d = (x + y)8
(-1, -1)
xx d= (x + 2.75x)8
(-1, 0) (0, 0) (3, 0) x x d = 3.75x x 8
d = 30 days
238. (d)
Cleave - means to divide/split
D ~_ _,__ _ _ ___, C
(0, -5)
239. (b)
32.4 = K (20 + 25) ... (i)
IY + 21 ~ 3 X - y ~ - 2 42 = K (50 + 30) ... (ii)
X:S:: 3 y :s; i e= 1
x~-1 -5:S::y
240. (22.22)
Total area= Area of rectangle ABCD
+ Area of L1ABE
i A
= 4x6+--(4x4)=32 E
2 T
EM
20m
230. (a)
We can see graph only (i) follows.
0
0)
l p
70 m
~~
C
l
232. (b) 100 m - - - - - - - 1
P➔ Married tiAME "" tiAPC
R ➔ Unmarried AM ME
---
Q ➔ Status Not given AP PC
P ➔ Looks@ Q 20 ME
⇒ ---
O ➔ Looks@ R 90 100
Now two possibility arise, ⇒ ME= 22.22
fflRDE ERS!::il ® Previous Years GATE Solved Questions I 309
12.5 a+b+c=1+1+1=3
So loss 108 x - - = 60
22.5 249. (b)
244. (b) a+a+a+ ... +a = na = 8 2b
ntimes
A x B x C = B x G x E = D x E x F = 72
⇒ n= ab "' (i)
8 X 9 X 1= 9 X 2 X 4 = 3 X 4 X 6 = 72
Any of A, B, C, D, E, F, G cannot be 5. b+b+b+ ... +b = mb = b 2a
,nlirnes
of correct clock
E + F + G = 13 ... (iii)
G + H+ K = 13 ... (iv) (24 + 150) hours of IC
6
Adding [(i) + (ii) + (iii) + (iv)]
= 24 hours of correct clock
A+B+C+O+ E + F + G + H + K +
96
(C + E + G) = 13 x 4 = 52 ... (v) 1 hour of IC = hours of correct clock
97
Also A, B, C, D, E, F, G, H & K represents natural
numbers from (1 to 9) 96
101 hour of IC = - x 101 hours of correct clock
97
= 99.958 hours of correct clock
There sum will be given by n(n+ i) = 45
2 = 99 hours+ 0.95876 x 60 minutes
Substituting (iv) C + E + G = 7 ... (vi) of correct clock
Now, 0 will become equal to 10 (which is not 2 PM, 11 th july + (99 hours and 58 minutes)
1
(z = p q(R) =3
x logp = logq
n= 7
y logq = logr
z logr = logp Option (1),P(G = 3)=
7
¼(~J3(ir
logq
x=-···- 35x~ 35x~
YP =
(3)7 - (3)7
yr
y == logq
logp
(2),
P(G = 4) =
7
C4 x(~r x(iY
Z= ~·--.
yr
xxyxz=1
59. {b)
Male + Female = 300 (3),
". (i)
8 2
Total money=
9 Mx750+ 3F x moo
312 I ® Reasoning & Aptitude fflRDE ERSl!::I
265. (b)
(4), 715 ? 423
0
We know that the divisibility rule for 3 is sum of
7x26 28x24 all digits should be divisible by 3.
=
= (3)7 (3)7 7 + 1 + 5 + 4 + 2 + 3 = 22
Option 3 is maxin:ium value. So, the next number after sum 22 which are
divisible by 3 are 24. 27, 30 etc.
So, five green faces and two red faces.
So, 22 + 2 = 24
262. (b) 22 + 5 = 27
Let total number of people are 100. But according to minimum condition 2 is right
100 answer.
266. (b)
(M)60/I~ 40(W)
80 (Attend)
Alloy A contains Gold and Copper.
Let 2x : 3x
(M)/ ~ 40(W) Which is same as 4x : 6x
Alloy B contains Copper 3x : 7x
So, M must be 80 - 40 = 40
As masses of Alloy A is equal to Alloy of mass B.
Ratio of male to female
i.e.
40: 40
Gold Copper
1 : 1
i.e. Alloy 1 4x 6x ⇒ 10x (mass)
263. (d) Alloy 2 3x 7x ⇒ 10x (mass)
This is a infinite Geometric progression with first :. Ratio of Gold to Copper is,
term (a) = 1 Gold from alloy 1 + Gold of alloy 2
1 Copper from alloy 1 + Copper of alloy 2
and common ratio (r) =
4 4x+3x 7x 7
The formula for finding the sum of infinite G.P = --=---
6x+ 7x 13x 13
a
is given by - - 267. (b)
1-r
1 1 4
1-2 - 3/4
=
3 A= P(1+-r )n
100
4
5
264. (b) 1000000 = P(1 + _2g_)·
iOO
A
5
Height of D
1000000 = p(_!2)
10
lamp post (h)
B
I..._
1.5
E
3 ___,...,...__ 3 -I
1000000 =
115
P(--105 l
ABC and DEC are similar triangles, 105 1011
p = 1000000 X --
AB BC 1 115
DE EC = 620921.323:ce62i000
h 6 268. (c)
- --
1.5 - 3 Let us suppose total number of cabs are i 00.
h 85 are green and rest 15 are blue.
-- = 2
i .5 Witness is correct 80% of times in identifying.
h = 3 meter Total number of blue cabs identified correctly.
IDRDE ERS~ @ Previous Years GATE Solved Questions I 313
80 272. (d)
(80% of 15) ⇒ X 15 = 12
100 f(x) = 0
Witness is incorrect 20% of times X = {-2, 0, 3}
That incorrectness should happen for green f(-2) = 0 , f(0) = 0 , f(3) = 0
f(x - 3) = 0
20 1, 3, 6
(20% of 85) ⇒ - , x85 = 17
100 f(1 - 3) = f(-2) = 0
Witness identifies 12 + 17 = 29 Cabs in total. f(3 - 3) = f(O) = 0
f(6 -- 3) = f(3) = 0
Required 12 12
Probability = Total = 12 + 17 = 29 273. (a)
=41.3%:co41% F(a, b) = (a - b) 2
270. (c)
Statement (i) is not true as nowhere it is mentioned
that John was a captain in junior team. The
introductory line emphasizes on the board
recognizing John's potential (Latent quality/
possIbfflty) as leader of the team.
Statement (iii) also manipulates the facts
mentioned in the argument.
DHLP HLPT
The 3 rd statement of the argument while this +3 I
aggression has filled stadia with die-hard fans +3 I
+3
does not indicate fans expectations from John
+3
as a caption.
Statement (ii) The concluding statement of the
275. (c)
para suggests that finding a completer captain
Numeral can be selected in 1 O ways (O - 9).
is a tough task as it took John many years to
Each of upper case and lower case alphabhats
become a successful and calculative batsman
can be done in 26 ways each.
- captain.
All three choosen (1 numeral and 2 alphabat) can
Statement (iv) can be explicitly concluded from
be arrange in 3! ways.
the last 4 lines of the para.
So, total number of ways will be 1 O x 26 x 26
?71. (c) x 3! = 40560 ways
25
iOO
So, option (c) ⇒ 4, 8, 28
= - x 100 = 80 marbles
125
3i 4 I III Re,asoning & Aptitude fflRDE ERS~
278. (b) From the curve it is clear that sinx and cosx both
B + G = 12
B=G+2 are decreasing in the interval (i, 1t).
⇒ 8=7
G=5 280. (d)
7 Boys and 5 Girls are there is 12 students. (i) Cuboid volume :;;: 8 x 1O x 6 ;::; 480 cm 3
Among 3 students selected boys have to be (ii) Cube volume ;::; 8 x 8 x 8 = 512 cm 3
more than girls. (iii) Cylinder volume = nr 2 h
So only two cases arise. 22 x 7 x 7 x 7 == 1078
. .3
G B = --
7 . . .cm
2 i
(iv) Sphere volume
3 0
As case (i)
4 .3 4 3 . 3
5 5 5 . 3 .. ;:::~nx(7)
""~nr 3 .... · ="1436.75cm
.
GGG-1 x x
12 12 12
Hence, the descending order of their volumes is
case (ii)G
GGB} ➔ B G 3x
5
x_§_x!_
(iv), (iii), (li), (i).
B G G 12 12 12 281. (c)
Area of equilateral triangle = ✓3
3 2
Required prob.= ( 2. )
\12
+ 3(2)
12
(1-)
12
x
650 325
= --=-
1728 864 a2 = 4
= 0.3761574 ~ 0.376 a== 2
Alternative Method : Perimeter = 3a = 3 x 2 : : : : 6
Th€3re are 7B and 5G
Throu.gh as question is stating 3 students are 282. (a)
taken at random. Total distance == x
This can be a possible way 'fo approach it. (Onward journey) $ 1 =;: 60 km/h
G B (Return journey) S2 = 90 km/h
2 1 Total distance
Average speed
3 0 Total time
fflRDS EASY ® Previous Years GATE Solved Questions I 315
A
(3,3Q,OQO) ---- \ ~ R
(2,50,0QO)
X + Y = 9 ". (i)
10x + y - 45=: iOy + X
or x-5=y
x-y=5 "' (ii)
100000 -· y
150000 + y 60000 + y
Adding (i) and (ii)
x=7
Subtracting (i) and (ii)
y=2
80,000 Therefore the number is 72.
90,000
---
6,30,000;:;; 2y + i ,50,000 + 100000 + 80000 +
60000 + 90000 + 90000
1 machine ➔ i toy ➔ 7 minutes
Because one machine takes 7 minute for making
i toy.
So, 100 machines will take 7 minute for making
6,30,000 - 5.70,000 = 2y 100 toys.
y = 30000
Students who failed to clear the test = 150000 290. (d)
+ 60000 + 90000 + 4y Given, a and b are integer
= 300000 + 4 X 300QQ a + a2 b3 is odd
Students who failed to clear the test = 420000 a(i +ab3 ) is odd
316 ® Reasoning & Aptitude fflRDE EASY
~=
2
use of 9 will be preferred over use of -Jg , which 16. A wall, rectangular in shape, has a perimeter of
72 m. If the length of its diagonal is 18 m, what
should be used minimally. How many times
is the area of the wall?
would Jg have to be used? (a) 224 ni 2 (b) 486 m 2
(a) 6 (b) 5 (c) 572 m2 (d) 606 m 2
(c) 4 (d) 3 [ESE i->re-2018 : 2 Marks]
[ESE Pre-2018 : 2 Marks]
17. To isolate an enclosed area for conservation, an
12. In a particular test, the marks scored by open traverse is run kE::eping close to (but outside
4 candidates •- A, B, C and Oare as follows: of) the exterior boundary of the area through
@ Marks ohtAined by A and B add to 100;
ground points A ➔ B ➔ C ➔ D ➔ E ➔ F ➔ G
• Marks obtained by C and O add up to those ➔ towards H (to be eventually located). AB is
scored by A;
80° to the East of the North line at A. Deflection/
• B scores 4 times of O;
Interior angles at B, C, D, E, Fare indicated.
• D scores 1O marks less than C.
What would be the magnitude of the deflection
The marks obtained by C will be
angle at G (as marked) so that GH many run
(a) 30 (b) 15
parallel to BA? (Lengths are immaterial in this
(c) 20 (d) 25
case.)
[ESE Pre-2018 : 2 Marks]
b
80° 60°
1300 900
kso:_.~B ~ ,D
---
(a) x 2 + 2x + 3
jgl_~~ + 2x - 3
(b) x 2 - 2x + 3
(dLx.~_:::::_2x_= .::s~-- To: ~ i o n ( a n g l e ~ --=--=-~----'-------
14. What is the maximum value of z, if (a) 190° (b) 210°
z = 10x+ 6y subject to t~e constraints (c) 200° (d) 230°
3x + y :s; 12, 2x + Sy :s; 34, x 2 0, y 2 07 [ESE Pre~2018: 2 Marks]
(a) 56 (b) 52
(c) 50 (d) 40 18. The objective function z =
3x 1 + 5x 2 is to be
[ESE Pre-20i 8 : 2 Marks j maximized subjected to constraints.
x 1 + 2x 2 S:: 200
15. Let the sum of the squares of successive integers X 1 +x2 S150
o, 1, 2, ... , n - i, n be denoted by S. Let the x1, X 2 2 0
sum of the cubes of the same integers be denoted The values of x 1 and x 2 in this context are,
C . , respectively
by C. It is desirable that
5 , as n increases 1n
(a) 100 and 75 (b) 125 and 75
steps of 'unity' from 'zero', is given by the series:
(c) 100 and 50 (d) 125 and 50
0 3 9 18 30 [ESE Pre~2018 : 2 Marks]
-,-,-,-,-, ... (for n = 0, 1, 2, 3, 4 .... ). What
135 7 9
19. Consider the length Qf a room is 15 m and width
will this ratio be for n = 8? is 10 m. If the sum,Qf,tQ~;~r~,s of the floor and
108 103 ceiling ls equal J· ,~, "cthe areas of the
(a) fl (b) i7 four walls, then tffe,
103 100 (a) 900 m 3
(c) 15 Id\1 -
' i5 (c) 1200 m3
[ESE Pre-20i 8 : 2 Marks]
320 ® Reasoning & Aptitude fflADE EASY
20. A small production unit now works 6 days per an 8% increase in weekly wages. How much
change in the hourly production would mean parity
week with 3 2 hours of first shift every one of the in the agreement for both management ar-id
3
employees?
6 days and 3 hours of second shift for each of (a) 3.68% (b) 2.15%
the first 5 days. Wage negotiations led to an (c) 1.82% (d) 1.33%
agreement to work on 5 days a week with both [ESE Pre-2018 : 2 Marks]
10. (c) 11. (b) 12. (d) 13. (a) 14. (a) 15. (a) 16. (b) 17. (a) 18. (c)
1. (d) Alt 1: If all the notes are Rs. 50 notes, total amount
Since the number of passengers in Black car is = 324 x SO= Rs. 16200 which is 16200- 12450
3, it leaves 300 - 3 = 297 passengers to be = 3750
accommodated in White and Silver coloured cars 3750/(50 - 20) = 125 notes of Rs. 20.
which are equal in number (W = S). Alt 2: This question can also be solved by putting
We can form a linear equation: options.
(6 + 5)W = 297 giving W = 27 as also S = 27
4. (d)
Total number of cars being 27 + 27 + 1 = 55
5 men in 20 days = i 00 man days
2. (a) 8 women in 25 days = 200 woman days
Let 3x, 4x, 5x be the present age of 3 brothers. 10 boys in 30 days = 300 boy days
After 10 years, sum of their age is 12x + 30 = 78 which means 100 MO = 200 WD = 300 80 or 1
which gives x = 4 man day = 3 boy days; 1 woman day = 1.5 Boy
Hence the present age of three brothers is 12, 16 days 2 men, 6 women and 5 boys = 2 x 3 + 6
and 20 years. x 1.5 + 5 = 20 boys deployed to complete the
Alt: Can be directly solved using the given options task which will get completed in 300/20 = i 5 days
since the sum of their present age = 78 - 30 = Alt: [2(1/iO0) + 6(1/200) + 5(1/300)]x = 1
48 which is satisfied by option 'A' alone. Which gives x = i 5
3. (c) 5. (c)
We can form 2 linear equations taking the number The remaining percentage after expenditure 20%
of Rs. 20 and Rs. 50 notes as T and F which equals Rs. 2000 leading to total monthly
T + F = 324 ... ( 1) income = Rs. 10000.
20T + 50 F = 12450 ... (2)
6. (d)
Solving the 2 equations, we get T = 125
If' W' is the original number of workers estimated
mRDE ERS~ @ Previous Years GATE Solved Questions I 321
(25) n
2
25 15. (a)
cm; Area of circle B = or
4 n
C;
Circumference of circle C = Circumference of circle Ignoring i st term;
A + Circumference of circle B = 4n + 5n = 9rc
322 I ® Reasoning & Aptitude fflRDE IERSY
19. (a)
Perimeter = 72 m Th
(a + b) x 2 = 72 m .;. (i)
a + b = 36 m
1
a2 + b2 = c?- ~
⇒ a2 + b 2 = 182 = 324 ... (ii) I 1s m
Area = a x b ... (iii) ..,_____1om-------....~
we know, (a + b) 2 = a2 + b 2 + 2ab
Given information is
⇒ 362 = 324 + 2 x (Area)
Area of floor + Area of ceiling = Sum of area of
⇒ Area= 486 m2
four walls
17. (a} ⇒ (10xi5)+(10 x i5) = (i5x h x2) + (iO x h x 2)
⇒ 300 = 50h
E
N
⇒ h=6m
Volume of room = i O m x i5 m x6 m
130° = 900 m3
B
D
A t=...-L-:..::...,
10'
G
----- 20. (a)
HA F In the first case, total number of hours per week
= 3.5 x 6 + 3 x 5 = 36 hours
L'.FGH = 190° In the second case, total number of hours= 75 x 5
Note: Line FG will be horizontal and not inclined = 37.5 hours
as given in question. With increased wages, the effective hourly rate is
36 08
( xi_· ) = i .0368 Imp
· I,YlnQ
· 3 .68°/co as the
37 5
increase in expected productivity.
111111111111!11!1!!1